You are on page 1of 127

ART.

VII
EXECUTIVE DEPARTMENT
A. EXECUTIVE POWER (Sec. 1) Commented [1]:
Section 1. The executive power shall be vested in the
Section 1. The executive power shall be vested in the President of the Philippines. President of the Philippines.

1. POWER TO ADMINISTER AND ENFORCE LAWS

Ople V. Torres 293 Scra 141 (1998)Petitioner Ople prays that invalidation of
Administrative Order No. 308 entitled "Adoption of a National Computerized Identification
Reference System" on two important constitutional grounds, viz: one, it is a usurpation of
the power of Congress to legislate, and two, it impermissibly intrudes on our citizenry's
protected zone of privacy.

Issue: Whether or not Administrative Order No. 308 is not a mere administrative order but
a law and hence, beyond the power of the President to issue.

***While Congress is vested with the power to enact laws, the President executes
the laws. The executive power is vested in the Presidents. It is generally defined as
the power to enforce and administer the laws. It is the power of carrying the laws
into practical operation and enforcing their due observance
.
As head of the Executive Department, the President is the Chief Executive. He
represents the government as a whole and sees to it that all laws are enforced by the
officials and employees of his department. He has control over the executive
department, bureaus and offices. This means that he has the authority to assume
directly the functions of the executive department, bureau and office or interfere
with the discretion of its officials. Corollary to the power of control, the President
also has the duty of supervising the enforcement of laws for the maintenance of
general peace and public order. Thus, he is granted administrative power over
bureaus and offices under his control to enable him to discharge his duties
effectively.

Administrative power is concerned with the work of applying policies and


enforcing orders as determined by proper governmental organs. It enables the
President to fix a uniform standard of administrative efficiency and check the official
conduct of his agents. To this end, he can issue administrative orders, rules and
regulations.

***Prescinding from these precepts, we hold that A.O. No. 308 involves a subject that
is not appropriate to be covered by an administrative order. An administrative
order is:

Sec. 3.Administrative Orders. — Acts of the President which relate to particular


aspects of governmental operation in pursuance of his duties as administrative head
shall be promulgated in administrative orders.

It cannot be simplistically argued that A.O. No. 308 merely implements the
Administrative Code of 1987. It establishes for the first time a National
Computerized Identification Reference System. Such a System requires a delicate
adjustment of various contending state policies — the primacy of national security,
the extent of privacy interest against dossier-gathering by government, the choice of
policies, etc. Indeed, the dissent of Mr. Justice Mendoza states that the A.O. No. 308
involves the all-important freedom of thought.

As said administrative order redefines the parameters of some basic rights of our
citizenry vis-a-vis the State as well as the line that separates the administrative
power of the President to make rules and the legislative power of Congress, it ought
to be evident that it deals with a subject that should be covered by law.
Nor is it correct to argue as the dissenters do that A.D. No. 308 is not a law because it
confers no right, imposes no duty, affords no protection, and creates no office. Under
A.O. No. 308, a citizen cannot transact business with government agencies delivering
basic services to the people without the contemplated identification card. No citizen
will refuse to get this identification card for no one can avoid dealing with
government. It is thus clear as daylight that without the ID, a citizen will have
difficulty exercising his rights and enjoying his privileges. Given this reality, the
contention that A.O. No. 308 gives no right and imposes no duty cannot stand.

KMU vs. Director General 487 SCRA 623 (2006) In April 13, 2005, President Gloria
Macapagal – Arroyo issued Executive Order 420 requiring all government agencies and
government-owned corporations to streamline and harmonize their Identification Systems.
The purposes of the uniform ID data collection and ID format are to reduce costs, achieve
efficiency and reliability and ensure compatibility and provide convenience to the people
served by government entities.
Petitioners allege that EO420 is unconstitutional because it constitutes usurpation of
legislative functions by the executive branch of the government. Furthermore, they allege
that EO420 infringes on the citizen’s rights to privacy.

ISSUE: IS EO420 a valid exercise of executive power?

***A unified ID system for all these government entities can be achieved in either of
two ways. First, the heads of these existing government entities can enter into a
memorandum of agreement making their systems uniform. If the government
entities can individually adopt a format for their own ID pursuant to their regular
functions under existing laws, they can also adopt by mutual agreement a uniform
ID format, especially if the uniform format will result in substantial savings, greater
efficiency, and optimum compatibility. This is purely an administrative matter, and
does not involve the exercise of legislative power.

Second, the President may by executive or administrative order direct the


government entities under the Executive department to adopt a uniform ID data
collection and format. Section 17, Article VII of the 1987 Constitution provides that
the “President shall have control of all executive departments, bureaus and offices.”
The same Section also mandates the President to “ensure that the laws be faithfully
executed.”

THE PRESIDENT’S CONSTITUTIONAL POWER OF CONTROL IS SELF-


EXECUTING AND DOES NOT NEED ANY IMPLEMENTING LEGISLATION.
Certainly, under this constitutional power of control the President can direct all
government entities, in the exercise of their functions under existing laws, to adopt a
uniform ID data collection and ID format to achieve savings, efficiency, reliability,
compatibility, and convenience to the public.

Of course, the President’s power of control is limited to the Executive branch of


government and does not extend to the Judiciary or to the independent
constitutional commissions. Thus, EO 420 does not apply to the Judiciary, or to the
COMELEC which under existing laws is also authorized to issue voter’s ID cards.
This only shows that EO 420 does not establish a national ID system because
legislation is needed to establish a single ID system that is compulsory for all
branches of government.

The Constitution also mandates the President to ensure that the laws are faithfully
executed. There are several laws mandating government entities to reduce costs,
increase efficiency, and in general, improve public services. The adoption of a
uniform ID data collection and format under EO 420 is designed to reduce costs,
increase efficiency, and in general, improve public services. Thus, in issuing EO
420, the President is simply performing the constitutional duty to ensure that the
laws are faithfully executed.
Legislative power is the authority to make laws and to alter or repeal them. In
issuing EO 420, the President did not make, alter or repeal any law but merely
implemented and executed existing laws. EO 420 reduces costs, as well as insures
efficiency, reliability, compatibility and user-friendliness in the implementation of
current ID systems of government entities under existing laws. Thus, EO 420 is
simply an executive issuance and not an act of legislation.

Difference in AO 308 re: Oplevs Torres:

***EO 420 applies only to government entities that already maintain ID systems and
issue ID cards pursuant to their regular functions under existing laws. EO 420 does
not grant such government entities any power that they do not already possess
under existing laws. In contrast, the assailed executive issuance in Ople v. Torres
sought to establish a “National Computerized Identification Reference System,”[19]
a national ID system that did not exist prior to the assailed executive issuance.
Obviously, a national ID card system requires legislation because it creates a new
national data collection and card issuance system where none existed before.

In the present case, EO 420 does not establish a national ID system but makes the
existing sectoral card systems of government entities like GSIS, SSS, Philhealth and
LTO less costly, more efficient, reliable and user-friendly to the public. Hence, EO
420 is a proper subject of executive issuance under the President’s constitutional
power of control over government entities in the Executive department, as well as
under the President’s constitutional duty to ensure that laws are faithfully executed.

Review Center vs. Executive Secretary 583 SCRA 428 (2009)There was a report that
handwritten copies of two sets of 2006 Nursing Board examination were circulated during
the examination period among examinees reviewing at the R.A. Gapuz Review Center and
Inress Review Center.The examinees were provided with a list of 500 questions and
answers in two of the examinations’ five subjects, particularly Tests III (Psychiatric
Nursing) and V (Medical-Surgical Nursing). The PRC later admitted the leakage and traced
it to two Board of Nursing members. Exam results came out but Court of Appeals
restrained the PRC from proceeding with the oath-taking of the successful examinees.

Subsequently, President GMA ordered for a re-examination and issued EO 566 which
authorized the CHED to supervise the establishment and operation of all review centers
and similar entities in the Philippines.
On 3 November 2006, the CHED, through its then Chairman Carlito S. Puno (Chairman
Puno), approved CHED Memorandum Order No. 49, series of 2006 (IRR).

In a letter dated 24 November 2006, the Review Center Association of the Philippines
(petitioner), an organization of independent review centers, asked the CHED to “amend, if
not withdraw” the IRR arguing, among other things, that giving permits to operate a review
center to Higher Education Institutions (HEIs) or consortia of HEIs and professional
organizations will effectively abolish independent review centers.
On 26 October 2007, petitioner filed a petition for Prohibition and Mandamus before this
Court praying for the declaration of EO 566 as invalid and unconstitutional, and the
prohibition against CHED from implementing the same.

ISSUE/S:
1. Whether EO 566 is an unconstitutional exercise by the Executive of legislative power as
it expands the CHED’s jurisdiction; and

2. Whether the RIRR is an invalid exercise of the Executive’s rule-making power.

***The scopes of EO 566 and the RIRR clearly expand the CHED’s coverage under RA
7722. The CHED’s coverage under RA 7722 is limited to public and private
institutions of higher education and degree-granting programs in all public and
private post-secondary educational institutions. EO 566 directed the CHED to
formulate a framework for the regulation of review centers and similar entities.
The definition of a review center under EO 566 shows that it refers to one which
offers “a program or course of study that is intended to refresh and enhance the
knowledge or competencies and skills of reviewees obtained in the formal school
setting in preparation for the licensure examinations” given by the PRC. It also
covers the operation or conduct of review classes or courses provided by
individuals whether for a fee or not in preparation for the licensure examinations
given by the PRC.

A review center is not an institution of higher learning as contemplated by RA 7722.


It does not offer a degree-granting program that would put it under the jurisdiction
of the CHED. A review course is only intended to “refresh and enhance the
knowledge or competencies and skills of reviewees.” A reviewee is not even
required to enroll in a review center or to take a review course prior to taking an
examination given by the PRC. Even if a reviewee enrolls in a review center,
attendance in a review course is not mandatory. The reviewee is not required to
attend each review class. He is not required to take or pass an examination, and
neither is he given a grade. He is also not required to submit any thesis or
dissertation. Thus, programs given by review centers could not be considered
“programs x xx of higher learning” that would put them under the jurisdiction of the
CHED.

Further, the “similar entities” in EO 566 cover centers providing “review or tutorial
services” in areas not covered by licensure examinations given by the PRC, which
include, although not limited to, college entrance examinations, Civil Services
examinations, and tutorial services. These review and tutorial services hardly
qualify as programs of higher learning.

***The President has no inherent or delegated legislative power to amend the


functions of the CHED under RA 7722. Legislative power is the authority to make
laws and to alter or repeal them,] and this power is vested with the Congress under
Section 1, Article VI of the 1987 Constitution

ADMINISTRATIVE AGENCIES EXERCISE THEIR QUASI-LEGISLATIVE OR RULE-


MAKING POWER THROUGH THE PROMULGATION OF RULES AND
REGULATIONS.The CHED may only exercise its rule-making power within the
confines of its jurisdiction under RA 7722. The RIRR covers review centers and
similar entities which are neither institutions of higher education nor institutions
offering degree-granting programs.

Hence, both the EO 566 and RIRR are unconstitutional.

2. RESIDUAL POWERS
Marcos v. Manglapuz, 177 SCRA 668 (1989) and178 SCRA 760 (1989) This case involves
a petition of mandamus and prohibition asking the court to order the respondents
Secretary of Foreign Affairs, etc. to issue a travel documents to former Pres. Marcos and the
immediate members of his family and to enjoin the implementation of the President's
decision to bar their return to the Philippines. Petitioners assert that the right of the
Marcoses to return in the Philippines is guaranteed by the Bill of Rights, specifically
Sections 1 and 6. They contended that Pres. Aquino is without power to impair the liberty
of abode of the Marcoses because only a court may do so within the limits prescribed by
law. Nor can the President impair their right to travel because no law has authorized her to
do so.

Issue: The issue is basically one of power: whether or not, in the exercise of the powers
granted by the Constitution, the President may prohibit the Marcoses from returning to the
Philippines.

*** EXECUTIVE POWER IS MORE THAN THE SUM OF SPECIFIC POWERS SO


ENUMERATED IN THE CONSTITUTION It would not be accurate, however, to state
that "executive power" is the power to enforce the laws, for the President is head of
state as well as head of government and whatever powers inherent in such
positions pertain to the office unless the Constitution itself withholds it.
Furthermore, the Constitution itself provides that the execution of the laws is only
one of the powers of the President. It also grants the President other powers that do
not involve the execution of any provision of law, e.g., his power over the country's
foreign relations.

On these premises, we hold the view that although the 1987 Constitution imposes
limitations on the exercise of specific powers of the President, it maintains intact
what is traditionally considered as within the scope of "executive power."
Corollarily, the powers of the President cannot be said to be limited only to the
specific powers enumerated in the Constitution.

THE POWER INVOLVED IS A RESIDUAL POWER OF THE PRESIDENT TO


PROTECT GENERAL WELFARE OF THE PEOPLE FOUNDED ON ITS DUTY AS
STEWARD OF THE PEOPLE***Admittedly, service and protection of the people, the
maintenance of peace and order, the protection of life, liberty and property, and the
promotion of the general welfare are essentially ideals to guide governmental
action. But such does not mean that they are empty words. Thus, in the exercise of
presidential functions, in drawing a plan of government, and in directing
implementing action for these plans, or from another point of view, in making any
decision as President of the Republic, the President has to consider these principles,
among other things, and adhere to them.

To the President, the problem is one of balancing the general welfare and the
common good against the exercise of rights of certain individuals. The power
involved is the President's residual power to protect the general welfare of the
people. It is founded on the duty of the President, as steward of the people.

More particularly, this case calls for the exercise of the President's powers as
protector of the peace. Rossiter The American Presidency].The power of the
President to keep the peace is not limited merely to exercising the commander-in-
chief powers in times of emergency or to leading the State against external and
internal threats to its existence. The President is not only clothed with
extraordinary powers in times of emergency, but is also tasked with attending to the
day-to-day problems of maintaining peace and order and ensuring domestic
tranquility in times when no foreign foe appears on the horizon. Wide discretion,
within the bounds of law, in fulfilling presidential duties in times of peace is not in
any way diminished by the relative want of an emergency specified in the
commander-in-chief provision. For in making the President commander-in-chief the
enumeration of powers that follow cannot be said to exclude the President's
exercising as Commander-in- Chief powers short of the calling of the armed forces,
or suspending the privilege of the writ of habeas corpus or declaring martial law, in
order to keep the peace, and maintain public order and security.

That the President has the power under the Constitution to bar the Marcose's from
returning has been recognized by members of the Legislature, and is manifested by
the Resolution proposed in the House of Representatives and signed by 103 of its
members urging the President to allow Mr. Marcos to return to the Philippines "as a
genuine unselfish gesture for true national reconciliation and as irrevocable proof of
our collective adherence to uncompromising respect for human rights under the
Constitution and our laws." [House Resolution No. 1342, Rollo, p. 321.1 The
Resolution does not question the President's power to bar the Marcoses from
returning to the Philippines, rather, it appeals to the President's sense of
compassion to allow a man to come home to die in his country.

What we are saying in effect is that the request or demand of the Marcoses to be
allowed to return to the Philippines cannot be considered in the light solely of the
constitutional provisions guaranteeing liberty of abode and the right to travel,
subject to certain exceptions, or of case law which clearly never contemplated
situations even remotely similar to the present one. It must be treated as a matter
that is appropriately addressed to those residual unstated powers of the
President which are implicit in and correlative to the paramount duty residing
in that office to safeguard and protect general welfare. In that context, such
request or demand should submit to the exercise of a broader discretion on the part
of the President to determine whether it must be granted or denied.

Accordingly, the question for the Court to determine is whether or not there exist factual
bases for the President to conclude that it was in the national interest to bar the return of
the Marcoses to the Philippines. If such postulates do exist, it cannot be said that she has
acted, or acts, arbitrarily or that she has gravely abused her discretion in deciding to bar
their return.

It will not do to argue that if the return of the Marcoses to the Philippines will cause
the escalation of violence against the State, that would be the time for the President
to step in and exercise the commander-in-chief powers granted her by the
Constitution to suppress or stamp out such violence. The State, acting through the
Government, is not precluded from taking pre- emptive action against threats to its
existence if, though still nascent they are perceived as apt to become serious and
direct. Protection of the people is the essence of the duty of government. The
preservation of the State the fruition of the people's sovereignty is an obligation in
the highest order. The President, sworn to preserve and defend the Constitution and
to see the faithful execution the laws, cannot shirk from that responsibility.

Province of Cotabato vs. GRP, 568 SCRA 402 (2008)Subject of these consolidated cases is
the extent of the powers of the President in pursuing the peace process. While the facts
surrounding this controversy center on the armed conflict in Mindanao between the
government and the MILF, the legal issue involved has a bearing on all areas in the country
where there has been a long-standing armed conflict. Yet again, the Court is tasked to
perform a delicate balancing act. It must uncompromisingly delineate the bounds within
which the President may lawfully exercise her discretion, but it must do so in strict
adherence to the Constitution, lest its ruling unduly restricts the freedom of action vested
by that same Constitution in the Chief Executive precisely to enable her to pursue the peace
process effectively.

ISSUE: The President cannot delegate a power that she herself does not possess. May the
President, in the course of peace negotiations, agree to pursue reforms that would require
new legislation and constitutional amendments, or should the reforms be restricted only to
those solutions which the present laws allow? The answer to this question requires a
discussion of

*** The extent of the Presidents power to conduct peace negotiations is not
explicitly mentioned in the Constitution, but this does not mean that she has no such
authority to do so.
in Marcos v. Manglapus as cited in Sanlakas v. Executive
(…unstated residual powers which are implied from the grant of executive
power and which are necessary for her to comply with her duties under the
Constitution. The powers of the President are not limited to what are expressly
enumerated in the article on the Executive Department and in scattered
provisions of the Constitution)Thus, the President's authority to declare a
state of rebellion springs in the main from her powers as chief executive and,
at the same time, draws strength from her Commander-in-Chief powers.
***Similarly, the Presidents power to conduct peace negotiations is implicitly
included in her powers as Chief Executive and Commander-in-Chief. As Chief
Executive, the President has the general responsibility to promote public peace, and
as Commander-in-Chief, she has the more specific duty to prevent and suppress
rebellion and lawless violence.

***THE PRESIDENT IS UNIQUELY VESTED WITH THE POWER TO CONDUCT


PEACE NEGOTIATITIONS W/ REBEL GROUPS,SHE MUST THEN BE GIVEN
LEEWAY TO EXPLORE SOLUTIONS THAT MAY REQUIRE CHANGES TO THE
CONSTITUTION. If the President is to be expected to find means for bringing this
conflict to an end and to achieve lasting peace in Mindanao, then she must be given
the leeway to explore, in the course of peace negotiations, solutions that may
require changes to the Constitution for their implementation. Being uniquely vested
with the power to conduct peace negotiations with rebel groups, the President is in
a singular position to know the precise nature of their grievances which, if resolved,
may bring an end to hostilities.

*** THE PRESIDENT MAY SUBMIT PROPOSALS FOR CONSTITUTIONAL CHANGE


TO CONGRESS IN A MANNER THAT DOES NOT INVOLVE THE ARROGATION OF
CONSTITUENT POWERS. Even lacking constituent powers, the President in the
course of conducting peace negotiations may validly consider implementing even
those policies that require changes to the Constitution, but she may not unilaterally
implement them without the intervention of Congress, or act in any way as if the
assent of that body were assumed as a certainty.

3. IMMUNITY FROM SUITS


Soliven v. Makasiar, 167 SCRA 393 (1988) President Aquino sued Beltran for libel for
having written that the President hid under the bed during the attempted coup. Petitioner
Beltran argues that ‘the reasons which necessitate presidential immunity from suit impose
a correlative disability to file a suit.” This would be an indirect way to defeat her privilege
of immunity from suit, as by testifying on the witness stand, she would be exposing herself
to possible contempt of court or perjury. Whether or not the President of the Philippines,
under the Constitution, may initiate criminal proceedings against the petitioners through the
filing of a complaint affidavit?

***Yes. The president of the Philippines may initiate criminal proceedings against
petitioners through filing of a complaint affidavit. Privilege of immunity may only be
invoked by the holder of the office. Thus, the accused in a criminal case of which the
President is a complainant cannot raise the presidential privilege as a defense to
prevent the case from proceedings against the accused. There is nothing in our laws
that prohibits the president from waiving her presidential privilege of immunity
from suits. The choice of whether to exercise the privilege or to waive it is solely to
the President’s prerogative. No other person can assume and waive it for him/her.

Gloria v. CA&Icsiano, 338 SCRA 5 (2000) Dr. BienvenidoIcasiano was appointed by


President Aquino as Schools Division Superintendent, Division of City Schools in QC. He
was reassigned as a superintendent of the Marikina Institute of Science and Technology as
recommended y DECS Sec. Ricardo Gloria on June 17, 1994. In October 1994 Director
Roxas informed Icasiano of the reassignment but Icasiano requested reconsideration of the
assignment. Request was denied and CA granted a TRO. CA found that the reassignment as
violative of Icasiano’s security of tenure. However, Sec. Gloria and Director Roxas contend
that the prohibition is improper because it attacks as act of the President in violation of the
Doctrine of Presidential immunity from suit.

***The contention is untenable for the simple reason that the petition is directed
against the petitioners and not against the President. The questioned acts are those
of the petitioners and not of the President. Furthermore, Presidential decisions may
be questioned before the courts where there is grave abuse of discretion or that the
President has acted without or in excess of jurisdiction.

Estrada v. Desierto, 353 SCRA 452 (2001)

Estrada Vs. Desierto 356 SCRA 108 Following the aborted impeachment trial and the
resignation of majority of the members of the cabinet and the defection of the military and
police at about noon of Jan. 20, 2001, Vice-President Arroyo was sworn into Office as
President of the Philippines, while President Estrada with his family left Malacañang. In the
days that followed, various criminal complaints were filed against Estrada before the
Ombudsman for preliminary investigation.Is Estrada, a non-sitting President, still immune
from suit?
*** No. immunity of the president from suit is concurrent only with his tenure and
not his term. THERE IS NO DECISION LICENSING THE PRESIDENT TO COMMIT
CRIMINAL ACTS AND WRAPPING HIM WITH POST-TENURE IMMUNITY FROM
LIABILITY The rule is that unlawful acts of public officials are not acts of the State
and the officer who acts illegally is not acting as such but stands in the same footing
as any other trespasser. The cases filed against Estrada are criminal in character.
They involve plunder, bribery and graft and corruption. By no stretch of the
imagination, can these crimes, especially plunder, which carries the death penalty,
covered by the alleged mantle of immunity of a non-sitting President.It will be
anomalous to hold that immunity is an inoculation from liability for unlawful acts
and omissions.

ESTRADA V. DESIERTO 353 SCRA 424 (THE EXTENT OF THE IMMUNITY)Petitioner


Estrada makes two submissions: first, the cases filed against him before the respondent
Ombudsman should be prohibited because he has not been convicted in the impeachment
proceedings against him; and second, he enjoys immunity from all kinds of suit, whether
criminal or civil. [Estrada vs. Desierto, 353 SCRA 452(2001)]

*** We reject his argument that he cannot be prosecuted for the reason that he must
first be convicted in the impeachment proceedings. The impeachment trial of
petitioner Estrada was aborted by the walkout of the prosecutors and by the events
that led to his loss of the presidency. Indeed, on February 7, 2001, the Senate passed
Senate Resolution No. 83 “Recognizing that the Impeachment Court is Functus
Officio”109 Since the Impeachment Court is now functus officio, it is untenable for
petitioner to demand that he should first be impeached and then convicted before
he can be prosecuted. The plea if granted, would put a perpetual bar against his
prosecution. Such a submission has nothing to commend itself for it will place him in
a better situation than a non-sitting President who has not been subjected to
impeachment proceedings and yet can be the object of a criminal prosecution. To be
sure, the debates in the Constitutional Commission make it clear that when
impeachment proceedings have become moot due to the resignation of the
President, the proper criminal and civil cases may already be filed against him,
incumbent Presidents are immune from suit or from being brought to court during
the period of their incumbency and tenure” but not beyond. Considering the
peculiar circumstance that the impeachment process against the petitioner has been
aborted and thereafter he lost the presidency, petitioner Estrada cannot demand as
a condition sine qua non to his criminal prosecution before the Ombudsman that he
be convicted in the impeachment proceedings. His reliance on the case of Lecaroz vs.
Sandiganbayan112 and related cases113 are inapropos for they have a different
factual milieu.

We now come to the scope of immunity that can be claimed by petitioner as a non-sitting
President.

***The cases filed against petitioner Estrada are criminal in character. They involve
plunder, bribery and graft and corruption. By no stretch of the imagination can
these crimes, especially plunder which carries the death penalty, be covered by the
alleged mantle of immunity of a non-sitting president. Petitioner cannot cite any
decision of this Court licensing the President to commit criminal acts and wrapping
him with post-tenure immunity from liability. It will be anomalous to hold that
immunity is an inoculation from liability for unlawful acts and omissions. The rule is
that unlawful acts of public officials are not acts of the State and the officer who acts
illegally is not acting as such but stands in the same footing as any other trespasser.

Indeed, a critical reading of current literature on executive immunity will reveal a


judicial disinclination to expand the privilege especially when it impedes the search
for truth or impairs the vindication of a right.

David vs.Macapagal-Arroyo, 489 scra 160 These seven (7) consolidated petitions for
certiorari and prohibition allege that in issuing Presidential Proclamation No. 1017 (PP
1017) declaring state of national emergency and General Order No. 5 (G.O. No. 5),
President Gloria Macapagal-Arroyo committed grave abuse of discretion. Petitioners
contend that respondent officials of the Government, in their professed efforts to defend
and preserve democratic institutions, are actually trampling upon the very freedom
guaranteed and protected by the Constitution. Hence, such issuances are void for being
unconstitutional.

***The Court rules that PP 1017 is CONSTITUTIONAL insofar as it constitutes a call


by President Gloria Macapagal-Arroyo on the AFP to prevent or suppress lawless
violence. However, the provisions of PP 1017 commanding the AFP to enforce
laws not related to lawless violence, as well as decrees promulgated by the
President, are declared UNCONSTITUTIONAL. In addition, the provision in PP
1017 declaring national emergency under Section 17, Article VII of the
Constitution is CONSTITUTIONAL, but such declaration does not authorize the
President to take over privately-owned public utility or business affected with
public interest without prior legislation.

G.O. No. 5 is CONSTITUTIONAL since it provides a standard by which the AFP and
the PNP should implement PP 1017, i.e. whatever is "necessary and appropriate
actions and measures to suppress and prevent acts of lawless violence." Considering
that "acts of terrorism" have not yet been defined and made punishable by the
Legislature, such portion of G.O. No. 5 is declared UNCONSTITUTIONAL. The
warrantless arrest of Randolf S. David and Ronald Llamas; the dispersal and
warrantless arrest of the KMU and NAFLU-KMU members during their rallies, in the
absence of proof that these petitioners were committing acts constituting lawless
violence, invasion or rebellion and violating BP 880; the imposition of standards on
media or any form of prior restraint on the press, as well as the warrantless search
of the Tribune offices and whimsical seizure of its articles for publication and other
materials, are declared UNCONSTITUTIONAL.

Rubrico vs. Arroyo, 613scra233 (Writ of amparo case.)


Issue: WHETHER OR NOT the [CA] committed reversible error in dismissing [their]
Petition and dropping President GMA as party respondent.Petitioners first take issue on
the President’s purported lack of immunity from suit during her term of office. The 1987
Constitution, so they claim, has removed such immunity heretofore enjoyed by the chief
executive under the 1935 and 1973 Constitutions.
***Petitioners are mistaken. The presidential immunity from suit remains preserved
under our system of government, albeit not expressly reserved in the present
constitution. Addressing a concern of his co-members in the 1986 Constitutional
Commission on the absence of an express provision on the matter, Fr. Joaquin
Bernas, S.J. observed that it was already understood in jurisprudence that the
President may not be sued during his or her tenure. The Court subsequently made it
abundantly clear in David v. Macapagal-Arroyo, a case likewise resolved under the
umbrella of the 1987 Constitution, that indeed the President enjoys immunity
during her incumbency, and why this must be so:

*** IT IS IMPORTANT THAT HE BE FREED FROM ANY FORM OF


HARASSMENT, HINDRANCE OR DISTRACTION TO ENABLE HIM TO
FULLY ATTEND TO THE PERFORMANCE OF HIS OFFICIAL DUTIES AND
FUNCTIONS. Settled is the doctrine that the President, during his tenure of
office or actual incumbency, may not be sued in any civil or criminal case, and
there is no need to provide for it in the Constitution or law. It will degrade
the dignity of the high office of the President, the Head of State, if he can be
dragged into court litigations while serving as such. Unlike the legislative and
judicial branch, only one constitutes the executive branch and anything
which impairs his usefulness in the discharge of the many great and
important duties imposed upon him by the Constitution necessarily impairs
the operation of the Government.10 x xx
And lest it be overlooked, the petition is simply bereft of any allegation as to
what specific presidential act or omission violated or threatened to violate
petitioners’ protected rights.
Commented [2]:
2. QUALIFICATIONS
natural-born citizen of the Philippines,
registered voter
Literate
40 years of age
B. QUALIFICATIONS, TERM OF OFFICE, ELECTION, PROCLAMATION OF 10 year residency
3. VICE PRESIDENT
PRES. AND VP, ASSUMPTION OF OFFICE (Sec. 2-7) There shall be a Vice-President who shall have the same
qualifications and term of office and be elected with, and in the
same manner, as the President. He may be removed from
office in the same manner as the President.
Section 2. No person may be elected President unless he is a natural-born citizen of the
Philippines, a registered voter, able to read and write, at least forty years of age on the day The Vice-President may be appointed as a Member of the
Cabinet. Such appointment requires no confirmation.
of the election, and a resident of the Philippines for at least ten years immediately 4.
preceding such election. The President and the Vice-President shall be elected by
direct vote of the people for a term of six years which shall
Section 3. There shall be a Vice-President who shall have the same qualifications and term begin at noon on the thirtieth day of June next following the
day of the election and shall end at noon of the same date, six
of office and be elected with, and in the same manner, as the President. He may be removed years thereafter.
from office in the same manner as the President. The President shall not be eligible for any re-election. No
person who has succeeded as President and has served as
The Vice-President may be appointed as a Member of the Cabinet. Such appointment such for more than four years shall be qualified for election to
the same office at any time.
requires no confirmation. No Vice-President shall serve for more than two successive
terms. Voluntary renunciation of the office for any length of
Section 4. The President and the Vice-President shall be elected by direct vote of the time shall not be considered as an interruption in the
continuity of the service for the full term for which he was
people for a term of six years which shall begin at noon on the thirtieth day of June next elected.
following the day of the election and shall end at noon of the same date, six years Unless otherwise provided by law, the regular election for
President and Vice-President shall be held on the second
thereafter. The President shall not be eligible for any re-election. No person who has Monday of May.
succeeded as President and has served as such for more than four years shall be qualified The returns of every election for President and Vice-President,
duly certified by the board of canvassers of each province or
for election to the same office at any time. city, shall be transmitted to the Congress, directed to the
President of the Senate. Upon receipt of the certificates of
No Vice-President shall serve for more than two successive terms. Voluntary renunciation canvass, the President of the Senate shall, not later than thirty
of the office for any length of time shall not be considered as an interruption in the days after the day of the election, open all the certificates in
the presence of the Senate and the House of Representatives
continuity of the service for the full term for which he was elected. in joint public session, and the Congress, upon determination
of the authenticity and due execution thereof in the manner
Unless otherwise provided by law, the regular election for President and Vice-President provided by law, canvass the votes.
shall be held on the second Monday of May. The person having the highest number of votes shall be
proclaimed elected, but in case two or more shall have an
The returns of every election for President and Vice-President, duly certified by the board equal and highest number of votes, one of them shall forthwith
be chosen by the vote of a majority of all the Members of both
of canvassers of each province or city, shall be transmitted to the Congress, directed to the Houses of the Congress, voting separately.
The Congress shall promulgate its rules for the canvassing of
President of the Senate. Upon receipt of the certificates of canvass, the President of the the certificates. The Supreme Court, sitting en banc, shall be
Senate shall, not later than thirty days after the day of the election, open all the certificates the sole judge of all contests relating to the election, returns,
and qualifications of the President or Vice-President, and may
in the presence of the Senate and the House of Representatives in joint public session, and promulgate its rules for the purpose.
the Congress, upon determination of the authenticity and due execution thereof in the 5.
Before they enter on the execution of their office, the
manner provided by law, canvass the votes. President, the Vice-President, or the Acting President shall
take the following oath or affirmation:"I do solemnly swear (or
The person having the highest number of votes shall be proclaimed elected, but in case two affirm) that I will faithfully and conscientiously fulfill my duties
or more shall have an equal and highest number of votes, one of them shall forthwith be as President (or Vice-President or Acting President) of the
Philippines, preserve and defend its Constitution, execute its
chosen by the vote of a majority of all the Members of both Houses of the Congress, voting laws, do justice to every man, and consecrate myself to the
separately. service of the Nation. So help me God." (In case of
affirmation, last sentence will be omitted.)
The Congress shall promulgate its rules for the canvassing of the certificates. 6. The President shall have an official residence. The salaries
The Supreme Court, sitting en banc, shall be the sole judge of all contests relating to the of the President and Vice-President shall be determined by
law and shall not be decreased during their tenure. No
election, returns, and qualifications of the President or Vice-President, and may promulgate increase in said compensation shall take effect until after the
its rules for the purpose. expiration of the term of the incumbent during which such
increase was approved. They shall not receive during their
tenure any other emolument from the Government or any
Section 5. Before they enter on the execution of their office, the President, the Vice- other source.
President, or the Acting President shall take the following oath or affirmation: 7. The President-elect and the Vice President-elect shall
assume office at the beginning of their terms.
"I do solemnly swear (or affirm) that I will faithfully and conscientiously fulfill my duties as
President (or Vice-President or Acting President) of the Philippines, preserve and defend
its Constitution, execute its laws, do justice to every man, and consecrate myself to the
service of the Nation. So help me God." (In case of affirmation, last sentence will be
omitted.)
Section 6. The President shall have an official residence. The salaries of the President and
Vice-President shall be determined by law and shall not be decreased during their tenure.
No increase in said compensation shall take effect until after the expiration of the term of
the incumbent during which such increase was approved. They shall not receive during
their tenure any other emolument from the Government or any other source.
Section 7. The President-elect and the Vice President-elect shall assume office at the
beginning of their terms.
If the President-elect fails to qualify, the Vice President-elect shall act as President until the
President-elect shall have qualified.
If a President shall not have been chosen, the Vice President-elect shall act as President
until a President shall have been chosen and qualified.
If at the beginning of the term of the President, the President-elect shall have died or shall
have become permanently disabled, the Vice President-elect shall become President.
Where no President and Vice-President shall have been chosen or shall have qualified, or
where both shall have died or become permanently disabled, the President of the Senate or,
in case of his inability, the Speaker of the House of Representatives, shall act as President
until a President or a Vice-President shall have been chosen and qualified.
The Congress shall, by law, provide for the manner in which one who is to act as President
shall be selected until a President or a Vice-President shall have qualified, in case of death,
permanent disability, or inability of the officials mentioned in the next preceding
paragraph.

Pormento vs. Estrada, 629 SCRA 530 (2010)What is the proper interpretation of the
following provision of Section 4, Article VII of the Constitution: [t]he President shall not be
eligible for any reelection?***moot and academic

Brillantes vs. COMELEC, 432 SCRA 269 (2004)(Unofficial Quick Count is VOID)

ISSUE: Whether or not Resolution No. 6712 dated April 28, 2004 issued by the COMELEC in
authorizing the use of election funds in consolidating the election results for the May 10,
2004 elections should be declared VOID, as it is unconstitutional.

***Yes, Resolution No. 6712 preempts the sole authority of the Congress to canvass
the votes of the election returns for the President and the Vice-President. The
assailed resolution usurps, under the guise of an "unofficial" tabulation of election
results based on a copy of the election returns, the sole and exclusive authority of
Congress to canvass the votes for the election of President and Vice-President.
Article VII, Section 4 of the Constitution provides in part:

Article VII, Section 4The returns of every election for President and Vice-
President duly certified by the board of canvassers of each province or city,
shall be transmitted to the Congress, directed to the President of the Senate.
Upon receipt of the certificates of canvass, the President of the Senate shall,
not later than thirty days after the day of the election, open all the certificates
in the presence of the Senate and the House of Representatives in joint public
session, and the Congress, upon determination of the authenticity and due
execution thereof in the manner provided by law, canvass the votes.

*** Senate President Drilon reiterated his position emphasizing that "any quick
count to be conducted by the Commission on said positions would in effect
constitute a canvass of the votes of the President and Vice-President, which not only
would be pre-emptive of the authority of Congress, but would also be lacking of any
constitutional authority. Nonetheless, in disregard of the valid objection of the
Senate President, the COMELEC proceeded to promulgate the assailed resolution.
Such resolution directly infringes the authority of Congress, considering that Section
4 thereof allows the use of the third copy of the Election Returns (ERs) for the
positions of President, Vice-President, Senators and Members of the House of
Representatives, intended for the COMELEC, as basis for the encoding and
transmission of advanced precinct results, and in the process, canvass the votes for
the President and Vice-President, ahead of the canvassing of the same votes by
Congress.

*** The contention of the COMELEC that its tabulation of votes is not prohibited by
the Constitution and Rep. Act No. 8436 as such tabulation is "unofficial," is puerile
and totally unacceptable. If the COMELEC is proscribed from conducting an official
canvass of the votes cast for the President and Vice-President, the COMELEC is, with
more reason, prohibited from making an "unofficial" canvass of said votes.

Defensor- Santiago v. Ramos, 253 SCRA 559 (1996)The protestant, Miriam Defensor-
Santiago ran for presidency and lost in the May 1992 election. In her Motion on the 16th
day of August in the year 1995, reiterated in her comment of the 29th of August of the same
year, protestant Defensor-Santiago prayed that the revision in the remaining precincts of
the pilot areas be dispensed with and the revision process in the pilot areas be deemed
computed.The Court deferred action on the motion and required, instead, the protestant
and protestee to submit their respective memoranda. Hence, this petition.

Issue:
Whether or not the election protest filed by Defensor-Santiago is moot and academic by her
election as a Senator in the May 1995 election and her assumption of office as such on the
30th of June in the year 1995.

***YES. IN ASSUMING THE OFFICE OF SENATOR, THE PROTESTANT HAS


EFFECTIVELY ABANDONED OR WITHDRAWN HER ELECTION PROTEST,
THEREBY MAKING IT MOOT. The protestant abandoned her “determination to
protest and pursue the public interest Involved in the matter of who is the real
choice of the electorate.Moreover, the dismissal of this protest would serve public
interest as it would dissipate the aura of uncertainty as to the results of the 1992
presidential elections, thereby enhancing the all too crucial political stability of the
nation during this period of national recovery.

*** the protestant abandoned her election protest when she waived the
revision of the remaining ballots and failed to inform the tribunal whether she
still intends to present additional evidence after the completion of the
revision of the ballots from the pilot areas. Also, the PET issued a resolution
ordering the protestant to inform the PET within 10 days if after the completion of
the revision of the ballots from her pilot areas, she still wishes to present evidence.
Since DS has not informed the Tribunal of any such intention, such is a manifest
indication that she no longer intends to do so.

Legarda vs. De Castro, 542 SCRA 125 (2008)


*** In the case at bar, protestants tenure in the Senate coincides with the term of the
Vice-Presidency 2004-2010, that is the subject of her protest. In Defensor-Santiago v.
Ramos, the protestant’s tenure in the Senate also coincided with the term of the
Presidency she was vying for. Like the protestant in the aforementioned case, the
protestant in the case at bar filed her certificate of candidacy for the Senate,
campaigned for the office, assumed office after election, and discharged the duties
and functions of said office. Thus, we agree concerning the applicability of the
Defensor-Santiago case as a precedent in the resolution of the present protest,
though they differ in that Defensor-Santiagos case involves the Presidency while
Legardas protest concerns only the Vice-Presidency.
***On the matter of the alleged spurious ER copies, we agree with the protestee that
the protestant had not adequately and convincingly rebutted the presumption that
as public documents, the Congress-retrieved ER copies, used for the proclamation of
the protestee by the NBC, are authentic and duly executed in the regular course of
official business. The evidence adduced by protestee to show that the supposed
security features and markings in the Congress-retrieved ERs and the
COMELEC/NAMFRELs copies are different, did not categorically establish that the
Congress-retrieved ERs are fake and spurious. To overcome the presumption of
regularity, there must be evidence that is clear, convincing and more than merely
preponderant. Absent such convincing evidence, the presumption must be
upheld.[41] In fact, the records show that even the witnesses presented by the
protestant testified that they were able to discern security features and markings in
the Congress-retrieved ERs. The records also show that witnesses were not made to
examine all Congress-retrieved ERs in making observations relative to security
features and markings, but only a sample set thereof was utilized, resulting in grave
insufficiency in the evidence presented by protestant.

***As to the alleged break-in in Congress, which allegedly facilitated the switching of
ERs, no conclusive evidence has been given. One of the protestants own witnesses,
Atty. ArtemioAdasa, Deputy General for Legislative Operations of the House of
Representatives, categorically denied that a break-in and a switching of ERs had
occurred in Congress. At any rate, as pointed out by protestee, even assuming
arguendo that all the votes in the 497 precincts included in the pilot areas for the
First Aspect with approximately 99,400 votes are considered in favor of protestant,
still the protestant would not be able to overcome the lead of the protestee.

Tecson v. COMELEC GR. 161434 (2004) Petitioners Tecson, et al., invoke the provisions of
Article VII, Section 4, paragraph 7, of the 1987 Constitution in assailing the jurisdiction of
the COMELEC when it took cognizance of SPA No. 04-003 and in urging the Supreme Court
to instead take on the petitions they directly instituted before it. The Constitutional
provision asserted reads:

"The Supreme Court, sitting en banc, shall be the sole judge of all contests
relating to the election, returns, and qualifications of the President or Vice-
President, and may promulgate its rules for the purpose."

*** The provision is an innovation of the 1987 Constitution. The omission in the
1935 and the 1973 Constitution to designate any tribunal to be the sole judge of
presidential and vice-presidential contests, has constrained this Court to declare, in
Lopez vs. Roxas, as not (being) justiciable controversies or disputes involving
contests on the elections, returns and qualifications of the President or Vice-
President. The constitutional lapse prompted Congress create PET through Republic
Act 1793 designating the Chief Justice and the Associate Justices of the Supreme
Court to be the members of the tribunal for V/PET contests. Although the
subsequent adoption of the parliamentary form of government under the 1973
Constitution might have implicitly affected Republic Act No. 1793, the statutory set-
up, nonetheless, would now be deemed revived under the present Section 4,
paragraph 7, of the 1987 Constitution.

***ORDINARY USAGE WOULD CHARACTERIZE A "CONTEST" IN REFERENCE TO


A POST-ELECTION SCENARIO.Election contests consist of either an election
protestORa quo warrantowhich, although two distinct remedies, would have one
objective in view, i.e., to dislodge the winning candidate from office. A perusal of the
phraseology in Rule 12, Rule 13, and Rule 14 of the "Rules of the Presidential
Electoral Tribunal," promulgated by the Supreme Court en banc on 18 April 1992,
would support this premise -

Rule 12.Jurisdiction. - The Tribunal shall be the sole judge of all contests
relating to the election, returns, and qualifications of the President or Vice-
President of the Philippines.
Rule 13. How Initiated. - An election contest is initiated by the filing of an
election protest or a petition for quo warranto against the President or Vice-
President. An election protest shall not include a petition for quo warranto. A
petition for quo warranto shall not include an election protest.

Rule 14.Election Protest. - ONLY THE REGISTERED CANDIDATE FOR


PRESIDENT OR FOR VICE-PRESIDENT OF THE PHILIPPINES WHO
RECEIVED THE SECOND OR THIRD HIGHEST NUMBER OF VOTES MAY
CONTEST THE ELECTION OF THE PRESIDENT OR THE VICE-
PRESIDENT,as the case may be, by filing a verified petition with the Clerk of
the PET within (30) days after the proclamation of the winner.

The rules categorically speak of the jurisdiction of the tribunal over contests relating
to the election, returns and qualifications of the "President" or "Vice-President", of
the Philippines, and not of "candidates" for President or Vice-President.

A quo warranto proceeding is generally defined as being an action against a


person who usurps, intrudes into, or unlawfully holds or exercises a public office. In
such context, the election contest can only contemplate a post-election scenario. In
Rule 14, only a registered candidate who would have received either the second or
third highest number of votes could file an election protest. This rule again
presupposes a post-election scenario. It is fair to conclude that the jurisdiction of the
Supreme Court, defined by Section 4, paragraph 7, of the 1987 Constitution, would
not include cases directly brought before it, questioning the qualifications of a
candidate for the presidency or vice-presidency before the elections are
held.Dismissed for lack of jurisdiction.

Poe Jr. v. Macapagal 454 SCRA 142 (2005) Refusing to concede defeat, the second-placer
in the elections, Mr. FPJ, filed an election protest before PET against Mrs. GMA. On
December 14, 2004, the FPJ died of cardio-pulmonary arrest. Susan Roces, his widow then
filed a motion to intervene. Proper? No.
***We are not unaware that a contest before election tribunals has two aspects.
First, it is in pursuit of ones right to a public office, and second, it is imbued with
public interest.

***THE INTEREST WHICH ALLOWS A PERSON TO INTERVENE IN A SUIT MUST


BE IN THE MATTER OF LITIGATION AND OF SUCH DIRECT AND IMMEDIATE
CHARACTER THAT THE INTERVENOR WILL EITHER GAIN OR LOSE BY THE
EFFECT OF THE JUDGMENT. In this protest, Mrs. FPJ will not immediately and
directly benefit from the outcome should it be determined that the declared
president did not truly get the highest number of votes. We fully appreciate counsels
manifestation that movant/intervenor herself claims she has no interest in
assuming the position as she is aware that she cannot succeed to the presidency,
having no legal right to it. Yet thus far, in this case, no real parties such as the vice-
presidential aspirants in the 2004 elections, have come forward to intervene, or to
be substituted for the deceased protestant. In our view, if persons not real parties in
the action could be allowed to intervene, proceedings will be unnecessarily
complicated, expensive and interminable and this is not the policy of the law

BANAT v. COMELEC, GR No. 177508, Aug. 7, 2009


Macalintal v. PET, GR No. 191618, June 7, 2011 petitioner reiterates that the constitution
of the PET, with the designation of the Members of the Court as Chairman and Members
thereof, contravenes Section 12, Article VIII of the Constitution, which prohibits the
designation of Members of the Supreme Court and of other courts established by law to any
agency performing quasi-judicial or administrative functions.
***the overarching framework affirmed in Tecson v. Commission on Elections is that
the Supreme Court has original jurisdiction to decide presidential and vice-
presidential election protests whileconcurrently acting as an independent
Electoral Tribunal.
1) Verbalegis dictates that wherever possible, the words used in the
Constitution must be given their ordinary meaning except where technical
terms are employed, in which case the significance thus attached to them
prevails. What it says according to the text of the provision to be construed
compels acceptance and negates the power of the courts to alter it, based on
the postulate that the framers and the people mean what they say. Thus
these are cases where the need for construction is reduced to a minimum.

2) However, where there is ambiguity or doubt, the words of the Constitution


should be interpreted in accordance with the intent of its framers or ratio
legiset anima. A doubtful provision must be examined in light of the history
of the times, and the condition and circumstances surrounding the framing of
the Constitution.

3) utmagisvaleat quam pereat- It is a well-established rule in constitutional


construction that no one provision of the Constitution is to be separated from
all the others, to be considered alone, but that all the provisions bearing upon
a particular subject are to be brought into view and to be so interpreted as to
effectuate the great purposes of the instrument. In other words, the court
must harmonize them, if practicable, and must lean in favor of a construction
which will render every word operative, rather than one which may make
the words idle and nugatory.

***The Supreme Court’s method of deciding presidential and vice-presidential


election contests, through the PET, IS ACTUALLY A DERIVATIVE OF THE
EXERCISE OF THE PREROGATIVE CONFERRED BY THE AFOREQUOTED
CONSTITUTIONAL PROVISION.PET was constituted in implementation of Section
4, Article VII of the Constitution, and it faithfully complies – not unlawfully defies –
the constitutional directive. The adoption of a separate seal, as well as the change in
the nomenclature of the Chief Justice and the Associate Justices into Chairman and
Members of the Tribunal, respectively, was designed simply to highlight the
singularity and exclusivity of the Tribunal’s functions as a special electoral court.

***It is also beyond cavil that when the Supreme Court, as PET, resolves a
presidential or vice-presidential election contest, it performs what is essentially a
judicial power. In the landmark case of Angara v. Electoral Commission,47 Justice
Jose P. Laurel enucleated that "it would be inconceivable if the Constitution had not
provided for a mechanism by which to direct the course of government along
constitutional channels." In fact, Angara pointed out that "[t]he Constitution is a
definition of the powers of government." And yet, at that time, the 1935 Constitution
did not contain the expanded definition of judicial power found in Article VIII,
Section 1, paragraph 2 of the present Constitution.

With the explicit provision, the present Constitution has allocated to the Supreme
Court, in conjunction with latter’s exercise of judicial power inherent in all courts,48
the task of deciding presidential and vice-presidential election contests, with full
authority in the exercise thereof. The power wielded by PET is a derivative of the
plenary judicial power allocated to courts of law, expressly provided in the
Constitution. On the whole, the Constitution draws a thin, but, nevertheless, distinct
line between the PET and the Supreme Court.

If the logic of petitioner is to be followed, all Members of the Court, sitting in the
Senate and House Electoral Tribunals would violate the constitutional proscription
found in Section 12, Article VIII. Surely, the petitioner will be among the first to
acknowledge that this is not so. The Constitution which, in Section 17, Article VI,
explicitly provides that three Supreme Court Justices shall sit in the Senate and
House Electoral Tribunals, respectively, effectively exempts the Justices-Members
thereof from the prohibition in Section 12, Article VIII. In the same vein, it is the
Constitution itself, in Section 4, Article VII, which exempts the Members of the Court,
constituting the PET, from the same prohibition.
We have previously declared that the PET is not simply an agency to which
Members of the Court were designated. Once again, the PET, as intended by the
framers of the Constitution, is to be an institution independent, but not separate,
from the judicial department, i.e., the Supreme Court. McCulloch v. State of
Maryland49 proclaimed that "[a] power without the means to use it, is a nullity."
The vehicle for the exercise of this power, as intended by the Constitution and
specifically mentioned by the Constitutional Commissioners during the discussions
on the grant of power to this Court, is the PET. Thus, a microscopic view, like the
petitioner’s, should not constrict an absolute and constitutional grant of judicial
power.

C. SUCCESSION (Sec. 8-12)


Section 8. In case of death, permanent disability, removal from office, or resignation of the
President, the Vice-President shall become the President to serve the unexpired term. In
case of death, permanent disability, removal from office, or resignation of both the
President and Vice-President, the President of the Senate or, in case of his inability, the
Speaker of the House of Representatives, shall then act as President until the President or
Vice-President shall have been elected and qualified.
The Congress shall, by law, provide who shall serve as President in case of death,
permanent disability, or resignation of the Acting President. He shall serve until the
President or the Vice-President shall have been elected and qualified, and be subject to the
same restrictions of powers and disqualifications as the Acting President.
Section 9. Whenever there is a vacancy in the Office of the Vice-President during the term
for which he was elected, the President shall nominate a Vice-President from among the
Members of the Senate and the House of Representatives who shall assume office upon
confirmation by a majority vote of all the Members of both Houses of the Congress, voting
separately.
Section 10. The Congress shall, at ten o'clock in the morning of the third day after the
vacancy in the offices of the President and Vice-President occurs, convene in accordance
with its rules without need of a call and within seven days, enact a law calling for a special
election to elect a President and a Vice-President to be held not earlier than forty-five days
nor later than sixty days from the time of such call. The bill calling such special election
shall be deemed certified under paragraph 2, Section 26, Article V1 of this Constitution and
shall become law upon its approval on third reading by the Congress. Appropriations for
the special election shall be charged against any current appropriations and shall be
exempt from the requirements of paragraph 4, Section 25, Article V1 of this Constitution.
The convening of the Congress cannot be suspended nor the special election postponed. No
special election shall be called if the vacancy occurs within eighteen months before the date
of the next presidential election.
Section 11. Whenever the President transmits to the President of the Senate and the
Speaker of the House of Representatives his written declaration that he is unable to
discharge the powers and duties of his office, and until he transmits to them a written
declaration to the contrary, such powers and duties shall be discharged by the Vice-
President as Acting President.
Whenever a majority of all the Members of the Cabinet transmit to the President of the
Senate and to the Speaker of the House of Representatives their written declaration that
the President is unable to discharge the powers and duties of his office, the Vice-President
shall immediately assume the powers and duties of the office as Acting President.
Thereafter, when the President transmits to the President of the Senate and to the Speaker
of the House of Representatives his written declaration that no inability exists, he shall
reassume the powers and duties of his office. Meanwhile, should a majority of all the
Members of the Cabinet transmit within five days to the President of the Senate and to the
Speaker of the House of Representatives, their written declaration that the President is
unable to discharge the powers and duties of his office, the Congress shall decide the issue.
For that purpose, the Congress shall convene, if it is not in session, within forty-eight hours,
in accordance with its rules and without need of call.
If the Congress, within ten days after receipt of the last written declaration, or, if not in
session, within twelve days after it is required to assemble, determines by a two-thirds vote
of both Houses, voting separately, that the President is unable to discharge the powers and
duties of his office, the Vice-President shall act as President; otherwise, the President shall
continue exercising the powers and duties of his office.
Section 12. In case of serious illness of the President, the public shall be informed of the
state of his health. The members of the Cabinet in charge of national security and foreign
relations and the Chief of Staff of the Armed Forces of the Philippines, shall not be denied
access to the President during such illness.

Estrada v. Desierto, 353 SCRA 452 (2001)In the May 11, 1998 elections, petitioner Joseph
Ejercito Estrada was elected President whilerespondent GMA was elected VP. In the heat of
people power, Chief Justice Davide administered the oath to respondent Arroyo as
President of the Philippines. At 2:30 p.m., petitioner and his family hurriedly left
Malacanang Palace. He issued the following press statement:
"20 January 2001
STATEMENT FROM PRESIDENT JOSEPH EJERCITO ESTRADA
At twelve o'clock noon today, Vice President Gloria Macapagal-Arroyo took
her oath as President of the Republic of the Philippines. While along with
many other legal minds of our country, I have strong and serious doubts
about the legality and constitutionality of her proclamation as President, I do
not wish to be a factor that will prevent the restoration of unity and order in
our civil society.
It is for this reason that I now leave Malacañang Palace, the seat of the
presidency of this country, for the sake of peace and in order to begin the
healing process of our nation. I leave the Palace of our people with gratitude
for the opportunities given to me for service to our people. I will not shirk
from any future challenges that may come ahead in the same service of our
country.
I call on all my supporters and followers to join me in to promotion of a
constructive national spirit of reconciliation and solidarity.
May the Almighty bless our country and beloved people.
MABUHAY!
(Sgd.) JOSEPH EJERCITO ESTRADA"

On January 22, the Monday after taking her oath, respondent Arroyo immediately
discharged the powers the duties of the Presidency On February 6, respondent Arroyo
nominated Senator TeofistoGuingona, Jr., as her Vice President. Petitioner Joseph Ejercito
Estrada alleges that he is the President on leave while respondent Gloria Macapagal-
Arroyo claims she is the President..

ISSUE:Whether petitioner Estrada is a President on leave while respondent Arroyo is an


Acting President

***RESIGNATION CAN BE ORAL. IT CAN BE WRITTEN. IT CAN BE EXPRESS. IT


CAN BE IMPLIED. Resignation is not a high level legal abstraction. It is a factual
question and its elements are beyond quibble: there must be an intent to resign
and the intent must be coupled by acts of relinquishment. The validity of a
resignation is not governed by any formal requirement as to form. As long as the
resignation is clear, it must be given legal effect.Using this totality test, we hold that
petitioner resigned as President. In sum, we hold that the resignation of the
petitioner cannot be doubted.

***It was confirmed by his leaving Malacanang. In the press release containing his
final statement, (1) he acknowledged the oath-taking of the respondent as President
of the Republic albeit with reservation about its legality; (2) he emphasized he was
leaving the Palace, the seat of the presidency, for the sake of peace and in order to
begin the healing process of our nation. He did not say he was leaving the Palace due
to any kind inability and that he was going to reassume the presidency as soon as
the disability disappears: (3) he expressed his gratitude to the people for the
opportunity to serve them. Without doubt, he was referring to the past opportunity
given him to serve the people as President (4) he assured that he will not shirk from
any future challenge that may come ahead in the same service of our country.
Petitioner's reference is to a future challenge after occupying the office of the
president which he has given up; and (5) he called on his supporters to join him in
the promotion of a constructive national spirit of reconciliation and solidarity.
Certainly, the national spirit of reconciliation and solidarity could not be attained if
he did not give up the presidency.

The press release was petitioner's valedictory, his final act of farewell. His
presidency is now in the past tense. It is, however, urged that the petitioner did not
resign but only took a temporary leave dated January 20, 2001 of the petitioner sent
to Senate President Pimentel and Speaker Fuentebella is cited. Again, we refer to the
said letter, viz:
"Sir.
By virtue of the provisions of Section II, Article VII of the
Constitution, I am hereby transmitting this declaration that I am unable to
exercise the powers and duties of my office. By operation of law and the
Constitution, the Vice President shall be the Acting president.
(Sgd.) Joseph Ejercito Estrada"

To say the least, the above letter is wrapped in mystery. The pleadings filed by the
petitioner in the cases at bar did not discuss, may even intimate, the circumstances
that led to its preparation. Neither did the counsel of the petitioner reveal to the
Court these circumstances during the oral argument. It strikes the Court as strange
that the letter, despite its legal value, was never referred to by the petitioner during
the week-long crisis. To be sure, there was not the slightest hint of its
existencewhen he issued his final press release. It was all too easy for him to tell the
Filipino people in his press release that he was temporarily unable to govern and
that he was leaving the reins of government to respondent Arroyo for the time
bearing. Under any circumstance, however, the mysterious letter cannot negate the
resignation of the petitioner. If it was prepared before the press release of the
petitioner clearly as a later act. If, however, it was prepared after the press released,
still, it commands scant legal significance. Petitioner's resignation from the
presidency cannot be the subject of a changing caprice nor of a whimsical will
especially if the resignation is the result of his reputation by the people.

Issue 2: Was Estrada merely temporarily unable to perform the powers and duties of the
presidency, and hence is a President on leave?

Petitioner postulates that respondent Arroyo as Vice President has no power to


adjudge the inability of the petitioner to discharge the powers and duties of the
presidency. His significant submittal is that "Congress has the ultimate authority
under the Constitution to determine whether the President is incapable of
performing his functions in the manner provided for in section 11 of article VII.
Considering the operative facts:
1. Petitioner, on January 20, 2001, sent the above letter claiming inability to
the Senate President and Speaker of the House;
2. Unaware of the letter, respondent Arroyo took her oath of office as
President on January 20, 2001 at about 12:30 p.m.;
3. Despite receipt of the letter, the House of Representatives passed on
January 24, 2001 House Resolution No. 175
4. Also, House of the Representatives passed House Resolution No. 176
stating that expressed its support to the assumption into office by Vice
President Gloria Macapagal-Arroyo as President of the Republic of the
Philippines,
5. Further, bills were already sent by the Congress to the Office of GMA as
president.
6. Despite the lapse of time and still without any functioning Cabinet, without
any recognition from any sector of government, and without any support
from the Armed Forces of the Philippines and the Philippine National Police,
the petitioner continues to claim that his inability to govern is only
momentary.

***What leaps to the eye from these irrefutable facts is that both houses of Congress
have recognized respondent Arroyo as the President. Implicitly clear in that
recognition is the premise that the inability of petitioner Estrada. Is no longer
temporary. Congress has clearly rejected petitioner's claim of inability. In fine, even
if the petitioner can prove that he did not resign, still, he cannot successfully claim
that he is a President on leave on the ground that he is merely unable to govern
temporarily. That claim has been laid to rest by Congress and the decision that
respondent Arroyo is the de jure, president made by a co-equal branch of
government cannot be reviewed by this Court.
D. PROHIBITIONS (Sec. 13)
Section 13. The President, Vice-President, the Members of the Cabinet, and their deputies
or assistants shall not, unless otherwise provided in this Constitution, hold any other office
or employment during their tenure. They shall not, during said tenure, directly or
indirectly, practice any other profession, participate in any business, or be financially
interested in any contract with, or in any franchise, or special privilege granted by the
Government or any subdivision, agency, or instrumentality thereof, including government-
owned or controlled corporations or their subsidiaries. They shall strictly avoid conflict of
interest in the conduct of their office.
The spouse and relatives by consanguinity or affinity within the fourth civil degree of the
President shall not, during his tenure, be appointed as Members of the Constitutional
Commissions, or the Office of the Ombudsman, or as Secretaries, Undersecretaries,
chairmen or heads of bureaus or offices, including government-owned or controlled
corporations and their subsidiaries.

Bitonio v. COA, 425 SCRA 437 (2004)in 1994, petitioner Bitonio, Jr. was appointed
Director IV of the Bureau of Labor Relations in the DOLE. In a Letter dated May 11, 1995,
Acting Secretary Jose S. Brilliantes of the DOLE designated the petitioner to be the DOLE
representative to the Board of Directors of PEZA. In pursuance to Section 11 of Republic
Act No. 7916, otherwise known as the Special Economic Zone Act of 1995, As
representative of the Secretary of Labor to the PEZA, the petitioner was receiving a per
diem for every board meeting he attended during the years 1995 to 1997. After a post
audit of the PEZA’s disbursement transactions, the COA disallowed the payment of per
diems to the petitioner. The uniform reason for the disallowance was stated in the Notices,
as follows:

“Cabinet members, their deputies and assistants holding other offices in addition to
their primary office and to receive compensation therefore was declared
unconstitutional by the Supreme Court in the Civil Liberties Union vs. Executive
Secretary. Disallowance is in pursuance to COA Memorandum No. 97-038 dated
September 19, 1997 implementing Senate Committee Report No. 509”

Issue: whether or not the COA correctly disallowed the per diems received by the
petitioner for his
attendance in the PEZA Board of Directors’ meetings as representative of the Secretary of
Labor.

*** YES. The COA anchors the disallowance of per diems in the case of Civil Liberties
Union v. Executive Secretary where the Court declared Executive Order No. 284
allowing government officials to hold multiple positions in government,
unconstitutional. Thus, Cabinet Secretaries, Undersecretaries, and their
Assistant Secretaries, are prohibited to hold other government offices or positions
in addition to their primary positions and to receive compensation therefor, except
in cases where the Constitution expressly provides. It must be noted that the
petitioner’s presence in the PEZA Board meetings is solely by virtue of his capacity
as representative of the Secretary of Labor. As the petitioner himself admitted, there
was no separate or special appointment for such position. Since the Secretary of
Labor is prohibited from receiving compensation for his additional office or
employment, such prohibition likewise applies to the petitioner who sat in the
Board only in behalf of the Secretary of Labor.

In Dela Cruz v. Commission on Audit., it was held that"The ex-officio position being
actually and in legal contemplation part of the principal office, it follows that
the official concerned has no right to receive additional compensation for his
services in the said position. The reason is that these services are already paid for
and covered by the compensation attached to his principal office. It should be
obvious that if, say, the Secretary of Finance attends a meeting of the Monetary
Board as an ex-officio member thereof, he is actually and in legal contemplation
performing the primary function of his principal office in defining policy in
monetary banking matters, which come under the jurisdiction of his department.
For such attendance, therefore, he is not entitled to collect any extra compensation,
whether it be in the form of a per diem or an honorarium or an allowance, or some
other such euphemism. By whatever name it is designated, such additional
compensation is prohibited by the Constitution."

Similarly in the case at bar, we cannot allow the petitioner who sat as representative
of the Secretary of Labor in the PEZA Board to have a better right than his principal.
As the representative of the Secretary of Labor, the petitioner sat in the Board in the
same capacity as his principal. Whatever laws and rules the member in the Board is
covered, so is the representative; and whatever prohibitions or restrictions the
member is subjected, the representative is, likewise, not exempted. Thus, his
position as Director IV of the DOLE which the petitioner claims is not covered by the
constitutional prohibition set by the Civil Liberties Union case is of no moment. The
petitioner attended the board meetings by the authority given to him by the
Secretary of Labor to sit as his representative. If it were not for such designation, the
petitioner would not have been in the Board at all.

NAC v. COA, 437 SCRA 655 (2004) Petitioner National Amnesty Commission (NAC) is a
government agency created on March 25, 1994 by then President Fidel V. Ramos tasked to
receive, process, and review amnesty applications. It is composed of seven members: a
Chairperson, three regular members appointed by the President, and the Secretaries of
Justice, National Defense and Interior and Local Government as ex officio members. It
appears that after personally attending the initial NAC meetings, the three ex officio
members turned over said responsibility to their representatives who were paid honoraria
beginning December 12, 1994. However, on October 15, 1997, NAC resident auditor Eulalia
disallowed on audit the payment of honoraria to these representatives

*** COA is correct that there is no legal basis to grant per diem, honoraria or any
allowance whatsoever to the NAC ex officio members' official representatives. In
Civil Liberties Union, we elucidated on the two constitutional prohibitions against
holding multiple positions in the government and receiving double compensation:
(1) the blanket prohibition of paragraph 2, Section 7, Article IX-B on all
government employees against holding multiple government offices, unless
otherwise allowed by law or the primary functions of their positions, and

(2) the stricter prohibition under Section 13, Article VII on the President
and his official family from holding any other office, profession, business or
financial interest, whether government or private, unless allowed by the
Constitution.
The NAC ex officio members’ representatives who were all appointive officials with
ranks below Assistant Secretary are covered by the two constitutional prohibitions.
First, the NAC ex officio members’ representatives are not exempt from the general
prohibition because there is no law or administrative order creating a new office or
position and authorizing additional compensation therefor.

Public Interest vs Elma, 517 SCRA 337 (2007) Elma sought - the reconsideration of the
Decision in the case of Public Interest Center, Inc., et al. v. Magdangal B. Elma, et al. (G.R. No.
138965), promulgated on 30 June 2006 where the Court declared that the concurrent
appointments of the respondent as PCGG Chairman and CPLC were unconstitutional
in violation of Section 7, par. 2, Article IX-B of the 1987 Constitution, since these are
incompatible offices. The duties of the CPLC include giving independent and impartial legal
advice on the actions of the heads of various executive departments and agencies and
reviewing investigations involving heads of executive departments. Since the actions of the
PCGG Chairman, a head of an executive agency, are subject to the review of the CPLC, such
appointments would be incompatible.

The Court also decreed that the strict prohibition under Section 13 Article VII of the
1987 Constitution would not apply to the present case, since neither the PCGG Chairman
nor the CPLC is a secretary, undersecretary, or assistant secretary. However, had the rule
thereunder been applicable to the case, the defect of these two incompatible offices would
be made more glaring. The said section allows the concurrent holding of positions only
when the second post is required by the primary functions of the first appointment and is
exercised in an ex-officio capacity. Although respondent Elma waived receiving
renumeration for the second appointment, the primary functions of the PCGG Chairman
do not require his appointment as CPLC.

Issue: Whether or not the motion for reconsideration be granted.

Ruling: DENIEDRespondent Elma’s concurrent appointments as PCGG Chairman and


CPLC are unconstitutional, for being incompatible offices. This ruling does not
render both appointments void. Following the common-law rule on incompatibility
of offices, respondent Elma had, in effect, vacated his first office as PCGG
Chairman when he accepted the second office as CPLC.

There also is no merit in the respondents’ motion to refer the case to the Court en
banc. What is in question in the present case is the constitutionality of respondent
Elma’s concurrent appointments, and not the constitutionality of any treaty, law or
agreement.[2] The mere application of constitutional provisions does not
require the case to be heard and decided en banc. Contrary to the allegations of
the respondent, the decision of the Court in this case does not modify the ruling in
Civil LibertiesUnion v. Executive Secretary. It should also be noted that Section 3 of
Supreme Court Circular No. 2-89, dated 7 February 1989clearly provides that the
Court en banc is not an Appellate Court to which decisions or resolutions of a
Division may be appealed.

Funa vs. Ermita, 612 SCRA 308 (2010)The sole issue to be resolved is whether or not the
designation of respondent Bautista as OIC of MARINA, concurrent with the position of
DOTC Undersecretary for Maritime Transport to which she had been appointed, violated
the constitutional proscription against dual or multiple offices for Cabinet Members and
their deputies and assistants.

*** Undersecretary Bautista’s designation as MARINA OIC falls under the stricter
prohibition under Section 13, Article VII of the 1987 Constitution.

SEC. 13. The President, Vice-President, the Members of the Cabinet, and
their deputies or assistants shall not, unless otherwise provided in this
Constitution, hold any other office or employment during their tenure. They
shall not, during said tenure, directly or indirectly practice any other profession,
participate in any business, or be financially interested in any contract with, or in
any franchise, or special privilege granted by the Government or any subdivision,
agency, or instrumentality thereof, including government-owned or controlled
corporations or their subsidiaries. They shall strictly avoid conflict of interest in the
conduct of their office.

Section 7, paragraph (2), Article IX-B reads:


Unless otherwise allowed by law or the primary functions of his
position, no appointive official shall hold any other office or
employment in the Government or any subdivision, agency or
instrumentality thereof, including government-owned or controlled
corporations or their subsidiaries.

***Respondent Bautista being then the appointed Undersecretary of DOTC, she was
thus covered by the stricter prohibition under Section 13, Article VII and
consequently she cannot invoke the exception provided in Section 7, paragraph 2,
Article IX-B where holding another office is allowed by law or the primary functions
of the position. Neither was she designated OIC of MARINA in an ex-officio capacity,
which is the exception recognized in Civil Liberties Union.
***The prohibition against holding dual or multiple offices or employment under
Section 13, Article VII of the 1987 Constitution was held inapplicable to posts
occupied by the Executive officials specified therein, without additional
compensation in an ex-officio capacity as provided by law and as required by the
primary functions of said office. The reason is that these posts do not comprise any
other office within the contemplation of the constitutional prohibition but are
properly an imposition of additional duties and functions on said officials.Apart
from their bare assertion that respondent Bautista did not receive any
compensation when she was OIC of MARINA, respondents failed to demonstrate
clearly that her designation as such OIC was in an ex-officio capacity as required by
the primary functions of her office as DOTC Undersecretary for Maritime Transport.
Powers of the President
A. APPOINTING POWER (Sec. 14-16)
Section 14. Appointments extended by an Acting President shall remain effective, unless
revoked by the elected President, within ninety days from his assumption or reassumption
of office.
Section 15. Two months immediately before the next presidential elections and up to the
end of his term, a President or Acting President shall not make appointments, except
temporary appointments to executive positions when continued vacancies therein will
prejudice public service or endanger public safety.
Section 16. The President shall nominate and, with the consent of the Commission on
Appointments, appoint the heads of the executive departments, ambassadors, other public
ministers and consuls, or officers of the armed forces from the rank of colonel or naval
captain, and other officers whose appointments are vested in him in this Constitution. He
shall also appoint all other officers of the Government whose appointments are not
otherwise provided for by law, and those whom he may be authorized by law to appoint.
The Congress may, by law, vest the appointment of other officers lower in rank in the
President alone, in the courts, or in the heads of departments, agencies, commissions, or
boards.

1. MIDNIGHT APPOINTMENTS

De Rama v. CA, 353 SCRA 94 (2001)Conrado L. de Rama wrote a letter dated July 13, 1995
to the Civil Service Commission (or CSC), seeking the recall of the appointments of fourteen
(14) municipal employees alleging that said employees were midnight appointments of the
former mayor, Ma. Evelyn S. Abeja, done in violation of Article VII, Section 15 of the 1987
Constitution.
*** there is no law that prohibits local elective officials from making appointments
during the last days of his or her tenure. The CSC correctly ruled, however, that the
constitutional prohibition on so-called midnight appointments, specifically those
made within two (2) months immediately prior to the next presidential elections,
applies only to the President or Acting President.

2. REGULAR APPOINTMENTS

Sarmiento v. Mison, 156 SCRA 549 (1987)Petitioners seek to enjoin the respondent
Salvador Mison from performing the functions of the Office of Commissioner of the Bureau
of Customs and the respondent Guillermo Carague, as Secretary of the Department of
Budget, from effecting disbursements in payment of Mison's salaries and emoluments, on
the ground that Mison's appointment as Commissioner of the Bureau of Customs is
unconstitutional by reason of its not having been confirmed by the Commission on
Appointments.
*** four groups of officers whom the President shall appoint.
1) heads of the executive departments, ambassadors, other public ministers and
consuls, officers of the armed forces from the rank of colonel or naval
captain, and other officers whose appointments are vested in him in this
Constitution;
2) all other officers of the Government whose appointments are not otherwise
provided for by law
3) those whom the President may be authorized by law to appoint;
4) Officers lower in rank 4 whose appointments the Congress may by law vest
in the President alone.

*** The first group of officers is clearly appointed with the consent of the CA.
Appointments of such officers are initiated by nomination and, if the nomination is
confirmed by the CA, the President appoints.

***In this case however, the position of Commissioner of the Bureau of Customs (a
bureau head) is not one of those within the first group of appointments where the
consent of the Commission on Appointments is required. As a matter of fact, as
already pointed out, while the 1935 Constitution includes "heads of bureaus" among
those officers whose appointments need the consent of the Commission on
Appointments, the 1987 Constitution on the other hand, deliberately excluded the
position of "heads of bureaus" from appointments that need the consent
(confirmation) of the Commission on Appointments.

Quintosv. Commission ,177 SCRA 259 (1989)On April 6, 1988, petitioner and three others
were appointed Sectoral Representatives by the President pursuant to Article VII, Section
16, paragraph 2 and Article XVIII, Section 7 of the Constitution. In the May 12, 1988
meeting of the Committee on Appointments ruled against the position of petitioner Deles.
Petitioner TeresitaQuintos-Deles contends that her appointment as Sectoral
Representative for Women by the President does not require confirmation by the
Commission on Appointments to qualify her to take her seat in the House of
Representatives.

ISSUE: Whether or not the Constitution requires confirmation by CA in the appointment of


sectoral representatives to the House of Representatives.

***Yes. The first group of people that may be appointed by the president, as
previously stated in the Sarmiento v. Mison case, are “the heads of the executive
departments, ambassadors, other public ministers and consuls, or officers of the
armed forces from the rank of colonel or naval captain, and other officers whose
appointments are vested in him in this Constitution.” Since the seats reserved for
sectoral representatives in paragraph 2, Section 5, Art. VI may be filled by
appointment by the President by express provision of Section 7, Art. XVIII of the
Constitution, it is indubitable that sectoral representatives to the House of
Representatives are among the “other officers whose appointments are vested
in the President in this Constitution,” referred to in the first sentence of Section
16, Article VII (or the first group of people who may be appointed) whose
appointments are subject to confirmation by the Commission on Appointments.
Thus, appointments by the President of sectoral representatives require the consent
of the Commission on Appointments in accordance with the first sentence of Section
16, Art. VII of the Constitution. More to the point, petitioner Deles' appointment was
issued not by virtue of Executive Order No. 198 but pursuant to Art. VII, Section 16,
paragraph 2 and Art.XVIII, Section 7 of the Constitution which require submission to
the confirmation process.

Bautista v. Salonga, 172 SCRA 160 (1989)n August 27, 1987, President Cory Aquino
appointed petitioner Bautista as permanent Chairman of the Commission on Human
Rights (CHR). Bautista took her oath of office on December 22, 1988 to Chief Justice
Marcelo Fernan and immediately acted as such.

On January 9, 1989, the Secretary of the CA wrote a letter to Bautista requesting for her
presence along with several documents at the office of CA on January 19. Bautista refused
to be placed under CA's review hence this petition filed with the Supreme Court.

While waiting for the progress of the case, President Aquino appointed Hesiquio R. Mallillin
as "Acting Chairman of the Commission on Human Rights" but he was not able to sit in his
appointive office because of Bautista's refusal to surrender her post. Malilin invoked EO
163-A which provides that the tenure of the Chairman and the Commissioners of the CHR
should be at the pleasure of the President thus stating that Bautista shall be subsequently
removed as well.

ISSUES: 1) Whether or not the President's appointment is considered constitutional; 2)


Whether or not Bautista's appointment is subject to CoA's confirmation; 3) Whether or not
President should extend her appointment on January 14, 1989.
*** THE PRESIDENT IS AUTHORIZED BY LAW TO APPOINT, WITHOUT
CONFIRMATION OF CA, SEVERAL GOVERNMENT OFFICIALS it is within the
authority of the President, vested upon her by the Constitution, that she appoint
Executive officials. The second sentence of the provision Section 16, Article VII
provides that the President is authorized by law to appoint, without confirmation of
CoA, several government officials. The position of Chairman of CHR is not among the
positions mentioned in the first sentence of Sec. 16, Art VII of the 1987 Constitution,
which provides the appointments which are to be made with the confirmation of
CoA. It therefore follows that the appointment of the Chairman of CHR by the
President is to be made and finalized even without the review or participation of CA.
Bautista's appointment as the Chairman of CHR, therefore, was already a completed
act onthe day she took her oath as the appointment was finalized upon her
acceptance, expressly stated in her oath.

Furthermore, the Court held that the provisions of EO 163-A is unconstitutional and
thus cannot be invoked by Mallillin. The Chairman of CHR cannot be removed at the
pleasure of the President for it is constitutionally guaranteed that they must have a
term of office. In view of the foregoing, the petition is thus GRANTED and the
restraining order for Mallillin was made permanent.

Calderon v. Carale ,208 SCRA 254 (1992) On 1989, RA 6715 was passed amending PD 442
or the Labor Code. RA 6715 provides that the Chairman, the Division Presiding
Commissioners and other Commissioners [of the NLRC] shall all be appointed by the
President, subject to confirmation by the CA. Appointments to any vacancy shall come from
the nominees of the sector which nominated the predecessor. Pursuant to the law, Cory
assigned Carale et al as the Chairman and the Commissioners respectively of the NLRC, the
appointment was not submitted to the CA for its confirmation.

Calderon questioned the appointment saying that w/o the confirmation by the CA, such an
appointment is in violation of RA 6715. Calderon asserted that RA 6715 is not an
encroachment on the appointing power of the executive contained in Sec16, Art. 7, of the
Constitution, as Congress may, by law, require confirmation by the Commission on
Appointments of other officers appointed by the President additional to those mentioned in
the first sentence of Sec 16 of Article 7 of the Constitution.

ISSUE: Whether or not Congress may, by law, require confirmation by the CA of


appointments extended by the President to government officers additional to those
expressly mentioned in the first sentence of Sec. 16, Art. 7 of the Constitution whose
appointments require confirmation by the CA.

In Sarmiento v. Mison(Bureau of Customs)


it is evident that the position of Commissioner of the Bureau of Customs (a
bureau head) is not one of those within the first group of appointments
where the consent of the Commission on Appointments is required. As a
matter of fact, as already pointed out, while the 1935 Constitution includes
"heads of bureaus" among those officers whose appointments need the
consent of the Commission on Appointments, the 1987 Constitution, on the
other hand, deliberately excluded the position of "heads of bureaus" from
appointments that need the consent (confirmation) of the Commission on
Appointments.

. . . Consequently, we rule that the President of the Philippines acted within


her constitutional authority and power in appointing respondent Salvador
Mison, Commissioner of the Bureau of Customs, without submitting his
nomination to the Commission on Appointments for confirmation. . . .

. . . In the 1987 Constitution, however, as already pointed out, the clear and
expressed intent of its framers was to exclude presidential appointments
from confirmation by the Commission on Appointments, except
appointments to offices expressly mentioned in the first sentence of Sec. 16,
Art. VII. Consequently, there was no reason to use in the third sentence of
Sec. 16, Article VII the word "alone" after the word "President" in providing
that Congress may by law vest the appointment of lower-ranked officers in
the President alone, or in the courts, or in the heads of departments, because
the power to appoint officers whom he (the president) may be authorized by
law to appoint is already vested in the President, without need of
confirmation by the Commission on Appointments, in the second sentence of
the same Sec. 16, Article VII.

In Bautista v. Salonga(CHR Chairman)


Since the position of Chairman of the Commission on Human Rights is not
among the positions mentioned in the first sentence of Sec. 16, Art. VII of the
1987 Constitution, appointments to which are to be made with the
confirmation of the Commission on Appointments, it follows that the
appointment by the President of the Chairman of the CHR is to be made
without the review or participation of the Commission on Appointments. To
be more precise, the appointment of the Chairman and Members of the
Commission on Human Rights is not specifically provided for in the
Constitution itself, unlike the Chairmen and Members of the Civil Service
Commission, the Commission on Elections and the Commission on Audit,
whose appointments are expressly vested by the Constitution in the
president with the consent of the Commission on Appointments. The
president appoints the Chairman and Members of The Commission on
Human Rights pursuant to the second sentence in Section 16, Art. VII, that is,
without the confirmation of the Commission on Appointments because they
are among the officers of government "whom he (the President) may be
authorized by law to appoint." And Section 2(c), Executive Order No. 163, 5
May 1987, authorizes the President to appoint the Chairman and Members of
the Commission on Human Rights.
TeresitaQuintos Deles, et al. v. The Commission on Constitutional Commissions,
et al.,
the power of confirmation of the Commission on Appointments over
appointments by the President of sectoral representatives in Congress was
upheld because:

. . . Since the seats reserved for sectoral representatives in paragraph 2,


Section 5, Art. VI may be filled by appointment by the President by express
provision of Section 7, Art. XVIII of the Constitution, it is indubitable that
sectoral representatives to the House of Representatives are among the
"other officers whose appointments are vested in the President in this
Constitution," referred to in the first sentence of Section 16, Art. VII whose
appointments are subject to confirmation by the Commission on
Appointments.

***DOCTRINES in interpreting Sec 16, Art 7 of the Constitution


1. Confirmation by the Commission on Appointments is required only for
presidential appointees mentioned in the first sentence of Section 16, Article
VII, including, those officers whose appointments are expressly vested by the
Constitution itself in the president (like sectoral representatives to Congress
and members of the constitutional commissions of Audit, Civil Service and
Election).

2. Confirmation is not required when the President appoints other


government officers whose appointments are not otherwise provided for by
law or those officers whom he may be authorized by law to appoint (like the
Chairman and Members of the Commission on Human Rights).

***In this case, the only issue to be resolved by the Court in the present case is
whether or not Congress may, by law, require confirmation by the Commission on
Appointments of appointments extended by the president to government officers
additional to those expressly mentioned in the first sentence of Sec. 16, Art. VII of
the Constitution whose appointments require confirmation by the Commission on
Appointments.
***Indubitably, the NLRC Chairman and Commissioners fall within the second
sentence of Section 16, Article VII of the Constitution, more specifically under the
"third groups" of appointees referred to in Mison, i.e. those whom the President may
be authorized by law to appoint. Undeniably, the Chairman and Members of the
NLRC are not among the officers mentioned in the first sentence of Section 16,
Article VII whose appointments requires confirmation by the Commission on
Appointments. To the extent that RA 6715 requires confirmation by the Commission
on Appointments of the appointments of respondents Chairman and Members of the
National Labor Relations Commission, it is unconstitutional because:

1)it amends by legislation, the first sentence of Sec. 16, Art. VII of the
Constitution by adding thereto appointments requiring confirmation by the
Commission on Appointments; and
2)it amends by legislation the second sentence of Sec. 16, Art. VII of the
Constitution, by imposing the confirmation of the Commission on
Appointments on appointments which are otherwise entrusted only with the
President.

Manalo v. Sistoza, 312 SCRA 329 (1999) On December 13, 1990, Republic Act 6975
creating the Department of Interior and Local Government was signed into law by former
President Corazon C. Aquino. Sections 26 and 31 of RA 6975 provided that the
appointments of PNP Chief, Senior Superintendent to Deputy Director General, and
Director General shall be subject to confirmation by the Commission on Appointments.

In accordance therewith, on March 10, 1992, the President of the Philippines, through then
Executive Secretary Franklin M. Drilon, promoted fifteen (15) respondent police officers,
by appointing them to positions in the Philippine National Police with the rank of Chief
Superintendent to Director. The appointments of respondent police officers were in a
permanent capacity.

Without their names submitted to the Commission on Appointments for confirmation, the
said police officers took their oath of office and assumed their respective positions.
Thereafter, the Department of Budget and Management, under the then Secretary Salvador
M. Enriquez III, authorized disbursements for their salaries and other emoluments.Legal?
*** It is well-settled that only presidential appointments belonging to the first group
require the confirmation by the Commission on Appointments. The appointments of
respondent officers who are not within the first category, need not be confirmed by
the Commission on Appointments. As held in the case of Tarrosa vs. Singson,
Congress cannot by law expand the power of confirmation of the Commission on
Appointments and require confirmation of appointments of other government
officials not mentioned in the first sentence of Section 16 of Article VII of the 1987
Constitution.

Consequently, unconstitutional are Sections 26 and 31 of Republic Act 6975 which


empower the Commission on Appointments to confirm the appointments of public
officials whose appointments are not required by the Constitution to be confirmed.
But the unconstitutionality of the aforesaid sections notwithstanding, the rest of
Republic Act 6975 stands. It is well-settled that when provisions of law declared
void are severable from the main statute and the removal of the unconstitutional
provisions would not affect the validity and enforceability of the other provisions,
the statute remains valid without its voided sections.

IT IS SETTLED THAT THE POLICE FORCE IS DIFFERENT FROM AND


INDEPENDENT OF THE ARMED FORCES AND THE RANKS IN THE MILITARY
ARE NOT SIMILAR TO THOSE IN THE PHILIPPINE NATIONAL POLICE. Thus,
directors and chief superintendents of the PNP, such as the herein respondent police
officers, do not fall under the first category of presidential appointees requiring the
confirmation by the Commission on Appointments.

Soriano v. Lista, 399 SCRA 437 (2004) Public respondents were promoted by PGMA to
different ranks in the Philippine Coast Guard (PCG) on different dates. Petitioner bewails
the fact that despite the non-submission of their names to the Commission on
Appointments (CA) for confirmation, all of the said respondent officers of the PCG had
assumed their duties and functions. According to petitioner, their respective appointments
are illegal and unconstitutional for failure to undergo the confirmation process in the CA.
Thus, they should be prohibited from discharging their duties and functions as such officers
of the PCG.

Issue: WON the appointments of the respondents need the confirmation of the Commission
on Appointments.

***NO. The PCG is under the DOTC and no longer part of the Philippine Navy or the
Armed Forces of the Philippines, the promotions and appointments of respondent
officers of the PCG, or any PCG officer from the rank of captain and higher for that
matter, do not require confirmation by the CA.

***OFFICERS OF THE ARMED FORCES FROM THE RANK OF COLONEL OR NAVAL


CAPTAIN” REFERS TO MILITARY OFFICERS ALONE.The enumeration of
appointments subject to confirmation by the CA under Section 16, Article VII of the
1987 Constitution is exclusive. The clause “officers of the armed forces from the
rank of colonel or naval captain” refers to military officers alone. This is clear from
the deliberations of the Constitutional Commission on the proposed text of said
Section 16, Article VII of the Constitution. Since the promotions and appointments of
respondent officers are not covered by the above-cited provision of the Constitution,
the same need not be confirmed by the CA.

3. AD INTERIM APPOINTMENTS

Matibag v. Benipayo, 380 SCRA 49 (2002) On February 1999, Matibag was appointed
Acting Director IV of the Comelec’s EID by then Comelec Chairperson Harriet Demetriou in
a temporary capacity. On March 2001, respondent Benipayo was appointed Comelec
Chairman together with other commissioners in an ad interim appointment. While on
such ad interim appointment, respondent Benipayo in his capacity as Chairman issued a
Memorandum address transferring petitioner to the Law Department. Petitioner requested
Benipayo to reconsider her relief as Director IV of the EID and her reassignment to the Law
Department. She cited Civil Service Commission Memorandum Circular No. 7 dated April
10, 2001, reminding heads of government offices that "transfer and detail of employees are
prohibited during the election period. Benipayo denied her request for reconsideration on
April 18, 2001, citing COMELEC Resolution No. 3300 dated November 6, 2000, exempting
Comelec from the coverage of the said Memo Circular.

Petitioner appealed the denial of her request for reconsideration to the COMELEC en banc.
She also filed an administrative and criminal complaint with the Law Department against
Benipayo, alleging that her reassignment violated Section 261 (h) of the Omnibus Election
Code, COMELEC Resolution No. 3258, Civil Service Memorandum Circular No. 07, s. 001,
and other pertinent administrative and civil service laws, rules and regulations.

During the pendency of her complaint before the Law Department, petitioner filed the
instant petition questioning the appointment and the right to remain in office of Benipayo,
Borra and Tuason, as Chairman and Commissioners of the COMELEC, respectively.
Petitioner claims that the ad interim appointments of Benipayo, Borra and Tuason violate
the constitutional provisions on the independence of the COMELEC.

ISSUE: WON the assumption of office by Benipayo, Borra and Tuason on the basis of the ad
interim appointments issued by the President amounts to a temporary appointment
prohibited by Section 1 (2), Article IX-C of the Constitution.

*** NO. AN AD INTERIM APPOINTMENT IS A PERMANENT APPOINTMENT


BECAUSE IT TAKES EFFECT IMMEDIATELY AND CAN NO LONGER BE
WITHDRAWN BY THE PRESIDENT ONCE THE APPOINTEE HAS QUALIFIED
INTO OFFICE. The fact that it is subject to confirmation by the Commission on
Appointments does not alter its permanent character. The Constitution itself makes
an ad interim appointment permanent in character by making it effective until
disapproved by the Commission on Appointments or until the next adjournment of
Congress.

In the instant case, the President did in fact appoint permanent Commissioners to
fill the vacancies in the COMELEC, subject only to confirmation by the Commission
on Appointments. Benipayo, Borra and Tuason were extended permanent
appointments during the recess of Congress. They were not appointed or designated
in a temporary or acting capacity, unlike Commissioner HaydeeYorac in Brillantes
vs. Yorac and Solicitor General Felix Bautista in Nacionalista Party vs. Bautista. The
ad interim appointments of Benipayo, Borra and Tuason are expressly allowed by
the Constitution which authorizes the President, during the recess of Congress, to
make appointments that take effect immediately.

While the Constitution mandates that the COMELEC "shall be independent", this
provision should be harmonized with the President’s power to extend ad interim
appointments. To hold that the independence of the COMELEC requires the
Commission on Appointments to first confirm ad interim appointees before the
appointees can assume office will negate the President’s power to make ad interim
appointments. This is contrary to the rule on statutory construction to give meaning
and effect to every provision of the law. It will also run counter to the clear intent of
the framers of the Constitution.

Pimentel Jr. v. Ermita, 472 SCRA 587 (2005) This is a petition to declare unconstitutional
the appointments issued by GMA through Secretary Ermita to respondents as acting
secretaries of their respective departments on August 2004. Congress adjourned on 22
September 2004.

On 23 September 2004, President Arroyo issued ad interim appointments to respondents


as secretaries of the departments to which they were previously appointed in an acting
capacity.

Issue: Is President Arroyo’s appointment of respondents as acting secretaries without the


consent of the Commission on Appointments while Congress is in session, constitutional?

*** Yes. The power to appoint is essentially executive in nature, and the legislature
may not interfere with the exercise of this executive power except in those instances
when the Constitution expressly allows it to interfere. Limitations on the executive
power to appoint are construed strictly against the legislature. The scope of the
legislature’s interference in the executive’s power to appoint is limited to the power
to prescribe the qualifications to an appointive office. Congress cannot appoint a
person to an office in the guise of prescribing qualifications to that office. Neither
may Congress impose on the President the duty to appoint any particular person to
an office.

However, even if the Commission on Appointments is composed of members of


Congress, the exercise of its powers is executive and not legislative. The Commission
on Appointments does not legislate when it exercises its power to give or withhold
consent to presidential appointments.

Petitioners contend that President Arroyo should not have appointed respondents
as acting secretaries because “in case of a vacancy in the Office of a Secretary, it is
only an Undersecretary who can be designated as Acting Secretary.”

THE ESSENCE OF AN APPOINTMENT IN AN ACTING CAPACITY IS ITS


TEMPORARY NATURE. It is a stop-gap measure intended to fill an office for a
limited time until the appointment of a permanent occupant to the office. In case of
vacancy in an office occupied by an alter ego of the President, such as the office of a
department secretary, the President must necessarily appoint an alter ego of her
choice as acting secretary before the permanent appointee of her choice could
assume office.
CONGRESS, THROUGH A LAW, CANNOT IMPOSE ON THE PRESIDENT THE
OBLIGATION TO APPOINT AUTOMATICALLY THE UNDERSECRETARY AS HER
TEMPORARY ALTER EGO. An alter ego, whether temporary or permanent, holds a
position of great trust and confidence. Congress, in the guise of prescribing
qualifications to an office, cannot impose on the President who her alter ego should
be.

The office of a department secretary may become vacant while Congress is in


session. Since a department secretary is the alter ego of the President, the acting
appointee to the office must necessarily have the President’s confidence. Thus, by
the very nature of the office of a department secretary, the President must appoint
in an acting capacity a person of her choice even while Congress is in session. That
person may or may not be the permanent appointee, but practical reasons may
make it expedient that the acting appointee will also be the permanent appointee.

The law expressly allows the President to make such acting appointment. Section
17, Chapter 5, Title I, Book III of EO 292 states that “[t]he President may temporarily
designate an officer already in the government service or any other competent
person to perform the functions of an office in the executive branch.” Thus, the
President may even appoint in an acting capacity a person not yet in the
government service, as long as the President deems that person competent.

Finally, petitioners claim that the issuance of appointments in an acting capacity is


susceptible to abuse. Petitioners fail to consider that acting appointments cannot
exceed one year as expressly provided in Section 17(3), Chapter 5, Title I, Book III
of EO 292. The law has incorporated this safeguard to prevent abuses, like the use of
acting appointments as a way to circumvent confirmation by the Commission on
Appointments.

AD-INTERIM APPOINTMENTS MUST BE DISTINGUISHED FROM


APPOINTMENTS IN AN ACTING CAPACITY. Both of them are effective upon
acceptance. But ad-interim appointments are extended only during a recess of
Congress, whereas acting appointments may be extended any time there is a
vacancy. Moreover ad-interim appointments are submitted to the Commission on
Appointments for confirmation or rejection; acting appointments are not submitted
to the Commission on Appointments. Acting appointments are a way of temporarily
filling important offices but, if abused, they can also be a way of circumventing the
need for confirmation by the Commission on Appointments.

However, we find no abuse in the present case. The absence of abuse is readily
apparent from President Arroyo’s issuance of ad interim appointments to
respondents immediately upon the recess of Congress, way before the lapse of one
year.

B. POWER OF CONTROL (Sec. 17)

The President shall have control of all the executive departments, bureaus, and offices. He
shall ensure that the laws be faithfully executed.

POWER OF CONTROL
Doctrine of Qualified Political Agency
- "all executive and administrative organizations are adjuncts of the Executive
Department; the heads of the various executive departments are assistants and
agents of the Chief Executive; and, except in cases where the Chief Executive is
required by the Constitution or law to act in person or the exigencies of the situation
demand that he act personally, the multifarious executive and administrative
functions of the Chief Executive are performed by and through the executive
departments, and the acts of the secretaries of such departments, performed and
promulgated in the regular course of business, are, unless disapproved or
reprobated by the Chief Executive, presumptively the acts of the Chief Executive."
Supervision and Control

Lacson v. Pano, 21 SCRA 895 (1967) Plaintiff's mainstay is Section 4 of Commonwealth


Act 141. The precept there is that decisions of the Director of Lands "as to questions of facts
shall be conclusive when approved" by the Secretary of Agriculture and Natural Resources.
Plaintiff's trenchment claim is that this statute is controlling not only upon courts but also
upon the President.

***WRONG. The president's duty to execute the law is of constitutional origin. so,
too, is his control of all executive departments. Thus it is, that department heads are
men of his confidence. His is the power to appoint them; his, too, is the privilege to
dismiss them at pleasure. Naturally, he controls and directs their acts. Implicit then
is his authority to go over, confirm, modify or reverse the action taken by his
department secretaries. In this context, it may not be said that the President cannot
rule on the correctness of a decision of a department secretary.

Particularly in reference to the decisions of the Director of Lands, as affirmed by the


Secretary of Agriculture and Natural Resources, the standard practice is to allow
appeals from such decisions to the Office of the President. This Court has
recognized this practice in several cases. In one, the decision of the Lands Director
as approved by the Secretary was considered superseded by that of the President's
appeal. In other cases, failure to pursue or resort to this last remedy of appeal was
considered a fatal defect, warranting dismissal of the case, for non-exhaustion of all
administrative remedies.

Parenthetically, it may be stated that THE RIGHT TO APPEAL TO THE PRESIDENT


REPOSES UPON THE PRESIDENT'S POWER OF CONTROL OVER THE EXECUTIVE
DEPARTMENTS.And control simply means "the power of an officer to alter or
modify or nullify or set aside what a subordinate officer had done in the
performance of his duties and to substitute the judgment of the former for that of
the latter."

***THE CHIEF EXECUTIVE MAY DELEGATE ITS POWER OF CONTROL TO THE


EXECUTIVE SECRETARY. The Office of the Executive Secretary is an auxiliary unit
which assists the President. Under our constitutional setup, the Executive Secretary
acts for and in behalf and by authority of the President and thus has an undisputed
jurisdiction to affirm, modify, or even reverse any order" that the Secretary of
Agriculture and Natural Resources, including the Director of Lands, may issue.-
Ratio: Doctrine of Qualified Political Agency, Hence executive secretary when acting
‘by authority of the President,’ may reverse the decision of department heads.

De Leon v. Carpio, 178 SCRA 457 (1989)Two consolidated cases involving the same issue
against respondent NBI who has refused to reinstate the petitioners in defiance of the CSC
as referred to him by Justice Sec for Implementation.

***It is an elementary principle of our republican government, enshrined in the


Constitution and honored not in the breach but in the observance that all executive
departments, bureaus and offices are under the control of the President of the
Philippines. This precept, first embodied in the Commonwealth Constitution and 11
reiterated in the 1973 Constitution, has been retained in Article VII, Section 17 of
the present Constitution.

The President's power of control is directly exercised by him over the members of
the Cabinet who, in turn and by his authority, control the bureaus and other offices
under their respective jurisdictions in the executive department. The constitutional
vesture of this power in the President is self-executing and does not require
statutory implementation, nor may its exercise be limited, much less withdrawn, by
the legislature.

In Lacson-Magallanes v. Pano,
SC held that a statute making decisions of the department secretaries final and
unappealable does not preclude the President from reviewing and even reversing
such decisions by virtue of his constitutional power of control over the members of
his Cabinet.

***In the case at bar, there is no question that when he directed the respondent to
reinstate the petitioners, Secretary Ordoñez was acting in the regular discharge his
functions as an alter ego of the President. His acts should therefore have been
respected by the respondent Director of the National Bureau of Investigation, which
is in the Department of Justice under the direct control of its Secretary. As a
subordinate in this department, the respondent was (and is) bound to obey the
Secretary's directives, which are presumptively the acts of the President of the
Philippines.

Bermudez v. Torres, 311 SCRA 733 (1999) whether or not the absence of a
recommendation of the Secretary of Justice to the President can be held fatal to the
appointment of respondent ConradoQuiaoit. This question would, in turn, pivot on the
proper understanding of the provision of the

Revised Administrative Code of 1987 (Book IV, Title III, Chapter II, Section9)
to the effect that- All provincial and city prosecutors and their assistants shall be
appointed by the President upon the recommendation of the Secretary.

***No. SC ruled that an “appointment” to a public office is the


unequivocal act of designating or selecting by one having the authority
therefor of an individual to discharge and perform the duties and
functions of an office or trust. The appointment is deemed complete once
the last act required of the appointing authority has been complied with and
its acceptance thereafter by the appointee in order to render it effective.

Indeed, it may rightly be said that the right of choice is the heart of the power
to appoint. In the exercise of the power of appointment, discretion is an
integral part thereof.When the Constitution or the law clothes the President
with the power to appoint a subordinate officer, such conferment must be
understood as necessarily carrying with it an ample discretion of whom to
appoint. It should be here pertinent to state that the President is the head of
government whose authority includes the power of control over all
“executive departments, bureaus and offices.”

It is the considered view of the Court that the phrase “upon recommendation
of the Secretary,” found in Section 9, Chapter II, Title III, Book IV, of the
Revised Administrative Code, SHOULD BE INTERPRETED TO BE A MERE
ADVISE, EXHORTATION OR INDORSEMENT, WHICH IS ESSENTIALLY
PERSUASIVE IN CHARACTER AND NOT BINDING OR OBLIGATORY UPON
THE PARTY TO WHOM IT IS MADE. The President, being the head of the
Executive Department, could very well disregard or do away with the action
of the departments, bureaus or offices even in the exercise of discretionary
authority, and in so opting, he cannot be said as having acted beyond the
scope of his authority.

MMDA v. Viron, 530 SCRA 341 (2007)GMA declared Executive Order (E.O.) No. 179
operational, thereby creating the MMDA in 2003. Due to traffic congestion, the MMDA
recommended a plan to “decongest traffic by eliminating the bus terminals now located
along major Metro Manila thoroughfares and providing more and convenient access to the
mass transport system.” The MMC gave a go signal for the project. Viron Transit, a bus
company assailed the move. They alleged that the MMDA didn’t have the power to direct
operators to abandon their terminals. In doing so they , together with another busline
asked the court to interpret the extent and scope of MMDA’s power under RA 7924. They
also asked if the MMDA law contravened the Public Service Act.
ISSUES: whether 1) the MMDA’s power to regulate traffic in Metro Manila included the
power to direct provincial bus operators to abandon and close their duly established and
existing bus terminals in order to conduct business in a common terminal;

***EO Unconstitutional. Under E.O. 125 A, the DOTC was given the objective of
guiding government and private investment in the development of the country’s
intermodal transportation and communications systems. It was also tasked to
administer all laws, rules and regulations in the field of transportation and
communications.

It bears stressing that under the provisions of E.O. No. 125, as amended, it is the
DOTC, and not the MMDA, which is authorized to establish and implement a project
such as the one subject of the cases at bar. THUS, THE PRESIDENT, ALTHOUGH
AUTHORIZED TO ESTABLISH OR CAUSE THE IMPLEMENTATION OF THE
PROJECT, MUST EXERCISE THE AUTHORITY THROUGH THE
INSTRUMENTALITY OF THE DOTC which, by law, is the primary implementing
and administrative entity in the promotion, development and regulation of
networks of transportation, and the one so authorized to establish and implement a
project such as the Project in question.

BY DESIGNATING THE MMDA AS THE IMPLEMENTING AGENCY OF THE


PROJECT, THE PRESIDENT CLEARLY OVERSTEPPED THE LIMITS OF THE
AUTHORITY CONFERRED BY LAW, rendering E.O. No. 179 ultra vires. There was
no grant of authority to MMDA. It was delegated only to set the policies concerning
traffic in Metro Manila, and shall coordinate and regulate the implementation of all
programs and projects concerning traffic management, specifically pertaining to
enforcement, engineering and education.

In light of the administrative nature of its powers and functions, the MMDA is devoid
of authority to implement the Project as envisioned by the E.O; hence, it could not
have been validly designated by the President to undertake the Project.

MMDA’S MOVE DIDN’T SATISFY POLICE POWER REQUIREMENTS such that:


✓ the interest of the publicgenerally, as distinguished from that of a particular
class, requires its exercise; and
✓ themeans employed are reasonably necessary for the accomplishment of
the purpose and not unduly oppressive upon individuals.
➢ Stated differently, the police power legislation must be firmly grounded
on public interest and welfare and a reasonable relation must exist
between the purposes and the means.

Notably, the parties herein concede that traffic congestion is a public concern that
needs to be addressed immediately. Are the means employed appropriate and
reasonably necessary for the accomplishment of the purpose. Are they not duly
oppressive?

De la Cruz v. Paras- Bus terminals per se do not, however, impede or help impede
the flow of traffic. How the outright proscription against the existence of all
terminals, apart from that franchised to petitioner, can be considered as reasonably
necessary to solve the traffic problem, this Court has not been enlightened

In the subject ordinances, however, the scope of the proscription against the
maintenance of terminals is so broad that even entities which might be able to
provide facilities better than the franchised terminal are barred from operating at
all.

Finally, an order for the closure of respondents’ terminals is not in line with the
provisions of the Public Service Act.
Consonant with such grant of authority, the PSC (now LTFRB)was empowered to
"impose such conditions as to construction, equipment, maintenance, service, or
operation as the public interests and convenience may reasonably require" in
approving any franchise or privilege. The law mandates the LTFRB to require any
public service to establish, construct, maintain, and operate any reasonable
extension of its existing facilities.

2. POWER TO REMOVE
Ang-Angco v. Castillo, 9 SCRA 619 (1963)Civil Service Executive Secretary Power of
Control Tenure A CSC officer bypassed the procedure prescribed by the CSC law. Executive
Sec. Castillo imposed disciplinary actions against the said officer. It must be noted that Art.
XI-B, Sec. 4 states: No officer or employee of the CSC shall be removed or suspended except
for a cause provided by law.

ISSUE: Whether the President through the Executive Secretary may take disciplinary action
against a CSC officer. NO
*** Despite the power of control conferred upon to the President, Section 4 Art. XII
of the Constitution clearly provides that ‘No officer or employee in the CSC shall be
removed or suspended except for cause as provided by law.” The constitution gives
stability to the tenure of office to those who belong to classified service. To hold
otherwise would demoralize and undermine the whole CSC structure. Congress has
provided, by law, for a procedure for the removal of CSC officers, ie. CSC Act of 1959.

Villaluz v. Zaldivar, 15 SCRA 710 (1965)Ruben Villaluz was appointed as the


Administrator of the Motor Vehicles Office in 1958. In 1960, Congressman Joaquin Roces
alleged that Villaluz was an ineffective leader and had caused losses to the government. He
indorsed the removal of Villaluz. Consequently, Executive Secretary CalixtoZaldivar
suspended Villaluz and ordered a committee to investigate the matter. After investigation,
it was recommended that Villaluz be removed. The president then issued an Administrative
Order removing Villaluz from his post. Villaluz averred that the president has no
jurisdiction to remove him.

ISSUE: Is Petitioner under the jurisdiction of the President to be removed considering that
he is an appointee of the president?

***Yes. The president has jurisdiction and not the Civil Service. THE POWER TO
REMOVE IS INHERENT IN THE POWER TO APPOINT. being a presidential
appointee, petitioner belongs to the non-competitive or unclassified service of the
government and is such he can only be investigated and removed from office after
due hearing the President of the Philippines under the principle that “the power to
remove is inherent in the power to appoint” as can be clearly implied from Section 5
of Republic Act No. 2260. Such is what we ruled in the recent case of Ang-Angco
wherein on this point we said:

There is some point in the argument that the power of control of the
President may extend to the power to investigate, suspend or remove officers
and employees who belong to the executive department if they are
presidential appointees or do not belong to the classified service for such can
be justified under the principle that the power to remove is inherent in the
power to appoint (Lacson v. Romero, supra), but not with regard to those
officers or employees who belong, to the classified service for as to them that
inherent power cannot be exercised. This is in line with the provision of our
Constitution which says that the “Congress may by law vest the appointment
of inferior officers, in the President alone, in the courts, or in the head of
departments” (Article VII, Section 10 [3], Constitution). (Ang-Angco v.
Castillo, et al., L-17169, November 30, 1963).

***PRESIDENTIAL APPOINTEE COMES UNDER THE JURISDICTION OF


THE PRESIDENT.There is, therefore, no error of procedure committed by
respondents insofar as the investigation and disciplinary action taken against
petitioner is concerned, even if he is under the control and supervision of the
Department of Public Works, in view of the reason we have already stated
that he is a presidential appointee who comes exclusively under the
jurisdiction of the President.

RATIO:POWER OF CONTROL GIVEN TO THE PRESIDENT BY THE


CONSTITUTION OVER GENERAL POLICY to all officers and employees in
the executive departments which is now involved by respondent as
justification to override the specific provisions of the Civil Service Act. This
power of control is couched in general terms for it does not set in specific
manner its extent and scope. Yes, this Court in the case of Hebron v. Reyes,
supra, occasion to interpret the extent of such power to mean “the power of
an officer to alter or modify or nullify or set aside what a subordinate officer
had done in the performance of his duties and to substitute the judgment of
the former for that of the latter,” to distinguish it from the power of general
supervision over municipal government, but the decision does not go to the
extent of including the power to remove an officer or employee in the
executive department.

Apparently, the power merely applies to the exercise of control over the acts
of the subordinate and not over the actor or agent himself of the act. It only
means that the President may set aside the judgment or action taken by a
subordinate in the performance of his duties. That meaning is also the
meaning given to the word “control” as used in administrative law. Thus, the
Department Head pursuant to Section 79 (c) is given direct control of all
bureaus and offices under his department by virtue of which he may “repeal
or modify decisions of the chiefs of said bureaus or offices,” and under
Section 74 of the same Code, the President’s control over the executive
department only refers to matters of general policy. The term “policy” means
a settled or definite course or method adopted and followed by a
government, body or individual, and it cannot be said that the removal of an
inferior officer comes within the meaning of control over a specific policy of
government. (Ang-Angco v. Castillo, et al., supra)

3. POWER TO REORGANIZE
Pichay v. Office of Deputy Sec., GR No. 196425, July 24, 2012 hen President Gloria
Macapagal-Arroyo issued Executive Order No. 12 (E.O. 12) creating the Presidential Anti-
Graft Commission (PAGC) and vesting it with the power to investigate or hear
administrative cases or complaints for possible graft and corruption, among others, against
presidential appointees and to submit its report and recommendations to the President.

On November 15, 2010, President Benigno Simeon Aquino III issued Executive Order No.
13 (E.O. 13), abolishing the PAGC and transferring its functions to the Office of the Deputy
Executive Secretary for Legal Affairs (ODESLA), more particularly to its newly-established
Investigative and Adjudicatory Division (IAD)

Petitioner asseverates that the President is not authorized under any existing law to create
the Investigative and Adjudicatory Division, Office of the Deputy Executive Secretary for
Legal Affairs (IAD-ODESLA) and that by creating a new, additional and distinct office tasked
with quasi-judicial functions, the President has not only usurped the powers of congress to
create a public office, appropriate funds and delegate quasi-judicial functions to
administrative agencies but has also encroached upon the powers of the Ombudsman.

*** PRESIDENT HAS CONTINUING AUTHORITY TO REORGANIZE THE


ADMINISTRATIVE STRUCTURE OF THE OFFICE OF THE PRESIDENT IN ORDER
TO ACHIEVE SIMPLICITY, ECONOMY, AND EFFICIENCY. This finds support under
E.O. 292 under Administrative Code of 1987. The Office of the President is the nerve
center of the Executive Branch. To remain effective and efficient, the Office of the
President must be capable of being shaped and reshaped by the President in the
manner he deems fit to carry out his directives and policies. After all, the Office of
the President is the command post of the President (Domingo v. Zamora)

Clearly, the abolition of the PAGC and the transfer of its functions to a division
specially created within the ODESLA is properly within the prerogative of the
President under his continuing "delegated legislative authority to reorganize" his
own office pursuant to E.O. 292.

***PRESIDENT’S POWER TO REORGANIZE OFFICE OF THE PRESIDENT MUST BE


DISTINGUISHED FROM HIS AUTHORITY TO ORGANIZE THE OFFICE OF THE
PRESIDENT PROPER. However, the President's power to reorganize the Office of
the President under Section 31 (2) and (3) of EO 292 should be distinguished from
his power to reorganize the Office of the President Proper.

➢ Under Section 31 (1) of EO 292, the President can reorganize the OFFICE
OF THE PRESIDENT PROPER by abolishing, consolidating or merging
units, or by transferring functions from one unit to another.

➢ In contrast, under Section 31 (2) and (3) of EO 292, the President's power
to reorganize offices OUTSIDE THE OFFICE OF THE PRESIDENT PROPER
but still within the Office of the President is limited to merely
transferring functions or agencies from the Office of the President to
Departments or Agencies, and vice versa.

In this case,PAGC and ODESLA both falls under Office of the President Proper, hence
reorganization by way of abolishing PAGC and transferring its functions to ODESLA
is allowable under Section 31 (1). This Reorganization Did not Entail the Creation of
a New, Separate and Distinct Office.

CONCEPT OF REORGANIZATION. Reorganization takes place when there is an


alteration of the existing structure of government offices or units therein, including
the lines of control, authority and responsibility between them. It involves a
reduction of personnel, consolidation of offices, or abolition thereof by reason of
economy or redundancy of functions. A valid reorganization must not only be
exercised through legitimate authority but must also be pursued in good faith. That
is, reorganizationdone for purposes of economy and efficiency.

The President's authority to issue E.O. 13 and constitute the IAD-ODESLA as his fact-
finding investigator cannot be doubted. After all, as Chief Executive, he is granted
full control over the Executive Department to ensure the enforcement of the laws.
Section 17, Article VII of the Constitution provides: XXX

The obligation to see to it that laws are faithfully executed necessitates the
corresponding power in the President to conduct investigations into the conduct of
officials and employees in the executive department

ODESLA DOES NOT USURP WITH OMBUDSMAN. Since the case filed before the
IAD-ODESLA is an administrative disciplinary case for grave misconduct, petitioner
may not invoke the primary jurisdiction of the Ombudsman to prevent the IAD-
ODESLA from proceeding with its investigation. In any event, the Ombudsman's
authority to investigate both elective and appointive officials in the government,
extensive as it may be, is by no means exclusive. It is shared with other similarly
authorized government agencies. While the Ombudsman's function goes into the
determination of the existence of probable cause and the adjudication of the merits
of a criminal accusation, the investigative authority of the IAD- ODESLA is limited to
that of a fact-finding investigator whose determinations and recommendations
remain so until acted upon by the President. As such, it commits no usurpation of
the Ombudsman's constitutional duties.

Anak-Mindanao v. Executive, 530 SCRA 583 (2007) AMIN charges the Executive
Department with transgression of the principle of separation of powers contending that
since DAR, PCUP, and NCIP were created by statutes, they can only be transformed, merged
or attached by statutes and not by Executive Orders. It thus argues that since the legislature
had seen fit to create these agencies at separate times and with distinct mandates, the
President should respect that legislative disposition.

***THE CONSTITUTION CONFERS, BY EXPRESS PROVISION, THE POWER OF


CONTROL OVER EXECUTIVE DEPARTMENTS, BUREAUS AND OFFICES IN THE
PRESIDENT ALONE. And it lays down a limitation on the legislative power.
xxx
Any power, deemed to be legislative by usage and tradition, is necessarily
possessed by Congress, unless the Constitution has lodged it elsewhere. In
fine, except as limited by the Constitution, either expressly or impliedly,
legislative power embraces all subjects and extends to matters of general
concern or common interest…

… The executive power is vested in the President. It is generally defined as


the power to enforce and administer the laws. It is the power of carrying the
laws into practical operation and enforcing their due observance. He has
control over the executive department, bureaus and offices. This means that
he has the authority to assume directly the functions of the executive
department, bureau and office, or interfere with the discretion of its officials.

Corollary to the power of control, the President also has the duty of
supervising and enforcement of laws for the maintenance of general peace
and public order. Thus, he is granted administrative power over bureaus and
offices under his control to enable him to discharge his duties effectively

In establishing an executive department, bureau or office, the legislature necessarily


ordains an executive agency’s position in the scheme of administrative structure.
Such determination is primary, but subject to the Presidents continuing authority to
reorganize the administrative structure. As far as bureaus, agencies or offices in the
executive department are concerned, the power of control may justify the President
to deactivate the functions of a particular office. Or a law may expressly grant the
President the broad authority to carry out reorganization measures.

x xx The law grants the President this power in recognition of the recurring
need of every President to reorganize his office to achieve simplicity,
economy and efficiency. The Office of the President is the nerve center of the
Executive Branch. To remain effective and efficient, the Office of the
President must be capable of being shaped and reshaped by the President in
the manner he deems fit to carry out his directives and policies. After all, the
Office of the President is the command post of the President. This is the
rationale behind the Presidents continuing authority to reorganize the
administrative structure of the Office of the President.

AGENCIES UNDER THE OFFICE OF THE PRESIDENT. It is not disputed that PCUP
and NCIP were formed as agencies under the Office of the President. The Agencies
under the Office of the President refer to those offices placed under the
chairmanship of the President, those under the supervision and control of the
President, those under the administrative supervision of the Office of the President,
those attached to the Office for policy and program coordination, and those that are
not placed by law or order creating them under any special department.

The President may transfer any agency under the Office of the President to any
other department or agency, subject to the policy in the Executive Office and in
order to achieve simplicity, economy and efficiency. Therefore, the challenged
executive orders may not be said to have been issued with grave abuse of discretion
or in violation of the rule of law.

*** The Administrative Code of 1987 administrative relationships


(1) supervision and control,
(2) administrative supervision, and
(3) attachment for policy and program coordination

AMIN has not shown, however, that by placing the NCIP as an attached agency of the
DAR, the President altered the nature and dynamics of the jurisdiction and
adjudicatory functions of the NCIP concerning all claims and disputes involving
rights of indigenous cultural communities and
indigenous peoples. Nor has it been shown, nay alleged, that the reorganization was
made in bad faith. DISMISSED.

Malaria Employees vs. Executive Secretary 528 SCRA 673 (2007) The President has the
authority to carry out a reorganization of the Department of Health under the Constitution
and statutory laws. This authority is an adjunct of his power of control under Article VII,
Sections 1 and 17 of the 1987 Constitution, viz.:

Section 1. The executive power shall be vested in the President of the Philippines.
Section 17. The President shall have control of all the executive departments,
bureaus and offices. He shall ensure that the laws be faithfully executed.
***While the power to abolish an office is generally lodged to the legislature, the
authority of the President to organize the executive branch, which may include such
abolition is permissible under present laws.

DENR v. DENR Employees ,360 SCRA 718 (2001)On November 15, 1999, Regional
Executive Director of DENR Region XII, Israel C. Gaddi, issued a Memorandum directing the
immediate transfer of the DENR XII Regional Offices from Cotabato City to Koronadal
(formerly Marbel), South Cotabato. The Memorandum was issued pursuant to DENR
Administrative Order No. 99-14, issued by then DENR Secretary Antonio H. Cerilles.
Respondents, employees of the DENR Region XII who are members of the employees
association, “COURAGE”, filed with the RTC in Cotabato, a petition for nullity of orders with
prayer for preliminary injunction.

ISSUES: Whether DAO-99-14 and the Memorandum implementing the same were valid;
and Whether the DENR Secretary has the authority to reorganize the DENR.

***YES & YES. The DAO and Memorandum are valid. The acts of the DENR Secretary
are likewise valid. It is apropos to reiterate the
ELEMENTARY DOCTRINE OF QUALIFIED POLITICAL AGENCY, thus:
Under this doctrine, which recognizes the establishment of a single executive, all
executive and administrative organizations are adjuncts of the Executive
Department, the heads of the various executive departments are assistants and
agents of the Chief Executive, and, except in cases where the Chief Executive is
required by the Constitution or law to act in person or the exigencies of the situation
demand that he act personally, the multifarious executive and administrative
functions of the Chief Executive are performed by and through the executive
departments, and the acts of the Secretaries of such departments, performed and
promulgated in the regular course of business, are, unless disapproved or
reprobated by the Chief Executive, presumptively the acts of the Chief Executive.

This doctrine is corollary to the control power of the President as provided for
under Article VII, Section 17 of the 1987 Constitution, which reads:

Sec. 17. The President shall have control of all the executive departments, bureaus,
and offices. He shall ensure that the laws be faithfully executed.

However, as head of the Executive Department, the President cannot be expected to


exercise his control (and supervisory) powers personally all the time. He may
delegate some of his powers to the Cabinet members except when he is required by
the Constitution to act in person or the exigencies of the situation demand that he
acts personally.

Applying the doctrine of qualified political agency, the power of the President to
reorganize the National Government may validly be delegated to his cabinet
members exercising control over a particular executive department. Thus, in DOTC
Secretary v. Mabalot, we held that the President – through his duly constituted
political agent and alter ego, the DOTC Secretary – may legally and validly decree the
reorganization of the Department, particularly the establishment of DOTC-CAR as
the LTFRB Regional Office at the Cordillera Administrative Region, with the
concomitant transfer and performance of public functions and responsibilities
appurtenant to a regional office of the LTFRB.

Similarly, in the case at bar, the DENR Secretary can validly reorganize the DENR by
ordering the transfer of the DENR XII Regional Offices from Cotabato City to
Koronadal, South Cotabato. The exercise of this authority by the DENR Secretary, as
an alter ego, is presumed to be the acts of the President for the latter had not
expressly repudiated the same.

Buklod v. Zamora, 360 SCRA 718 (2001) During the time of President Corazon Aquino,
she created the Economic Intelligence and Investigation Bureau (EIIB) to primarily
conduct anti-smuggling operations in areas outside the jurisdiction of the Bureau of
Customs. In the year 2000, President Estrada issued an order deactivating the EIIB. He
subsequently ordered the employees of EIIB to be separated from the service. Thereafter,
he created thru EO 196 the Presidential Anti-Smuggling Task Force “Aduana”, which EIIB
employees claim to be essentially the same as EIIB. The employees of EIIB, through the
BuklodngKawaning EIIB, invoked the Supreme Court’s power of judicial review in
questioning the said orders. EIIB employees maintained that the president has no power to
abolish a public office, as that is a power solely lodged in the legislature; and that the
abolition violates their constitutional right to security of tenure.

- "the President, subject to the policy of the Executive Office and in order to achieve
simplicity, economy and efficiency, shall have the continuing authority to reorganize
the administrative structure of the Office of the President." For this purpose, he may
transfer the functions of other Departments or Agencies to the Office of the
President.
- Nothing is better settled in our law than that the abolition of an office within the
competence of a legitimate body if done in good faith suffers from no infirmity.
Valid abolition of offices is neither removal nor separation of the incumbents.
Indeed, there is no such thing as an absolute right to hold office. Except
constitutional offices which provide for special immunity as regards salary and
tenure, no one can be said to have any vested right in an office or its salary.

Biraogo V. Phil. Truth Comm., GR No. 192935, Dec. 7, 2010Biraogo assails Executive
Order No. 1 for being violative of the legislative power of Congress under Section 1, Article
VI of the Constitution as it usurps the constitutional authority of the legislature to create a
public office and to appropriate funds therefor. E.O No. 1 establishing the (PTC) of 2010
was signed by President Aquino. The said PTC is a mere branch formed under the Office of
the President tasked to investigate reports of graft and corruption committed by third-level
public officers and employees, their co-principals, accomplices and accessories during the
previous administration and submit their findings and recommendations to the President,
Congress and the Ombudsman. However, PTC is not a quasi-judicial body, it cannot
adjudicate, arbitrate, resolve, settle or render awards in disputes between parties. Its job is
to investigate, collect and assess evidences gathered and make recommendations. It has
subpoena powers but it has no power to cite people in contempt or even arrest. It cannot
determine for such facts if probable cause exist as to warrant the filing of information in
our courts of law.

ISSUE: Does the delegated authority of the President to structurally reorganize the Office of
the President to achieve economy, simplicity, and efficiency include the power to create an
entirely new office was inexistent like the Truth Commission?
***No. the creation of the PTC is not justified by the Presidents power of control.
Control is essentially the power to alter or modify or nullify or set aside what a
subordinate officer had done in the performance of his duties and to substitute the
judgment of the former with that of the latter. Clearly, the power of control is
entirely different from the power to create public offices. The former is inherent in
the Executive, while the latter finds basis from either a valid delegation from
Congress, or his inherent duty to faithfully execute the laws.

*** One of the recognized powers of the President granted pursuant to this
constitutionally-mandated duty is the power to create ad hoc committees. This
flows from the obvious need to ascertain facts and determine if laws have been
faithfully executed under Article VII, Section 17. Having been constitutionally
granted full control of the Executive Department, to which respondents belong, the
President has the obligation to ensure that all executive officials and employees
faithfully comply with the law. With AO 298 as mandate, the legality of the
investigation is sustained. Such validity is not affected by the fact that the
investigating team and the PCAGC had the same composition, or that the former
used the offices and facilities of the latter in conducting the inquiry.

*** The petitioners assail Executive Order No. 1 because it is violative of this
constitutional safeguard. They contend that it does not apply equally to all members
of the same class such that the intent of singling out the "previous administration"
as its sole object makes the PTC an "adventure in partisan hostility." Thus, in order
to be accorded with validity, the commission must also cover reports of graft and
corruption in virtually all administrations previous to that of former President
Arroyo.

The equal protection clause is aimed at all official state actions, not just those of the
legislature. Its inhibitions cover all the departments of the government including the
political and executive departments, and extend to all actions of a state denying
equal protection of the laws, through whatever agency or whatever guise is taken.

Applying these precepts to this case, Executive Order No. 1 should be struck down
as violative of the equal protection clause. The clear mandate of the envisioned truth
commission is to investigate and find out the truth "concerning the reported cases of
graft and corruption during the previous administration"only. The intent to single
out the previous administration is plain, patent and manifest. Mention of it has been
made in at least three portions of the questioned executive order.

C. COMMANDER-IN-CHIEF POWERS (Sec. 18)


Section 18. The President shall be the Commander-in-Chief of all armed forces of the
Philippines and whenever it becomes necessary, he may call out such armed forces to
prevent or suppress lawless violence, invasion or rebellion.
In case of invasion or rebellion, when the public safety requires it, he may, for a period not
exceeding sixty days, suspend the privilege of the writ of habeas corpus or place the
Philippines or any part thereof under martial law. Within forty-eight hours from the
proclamation of martial law or the suspension of the privilege of the writ of habeas corpus,
the President shall submit a report in person or in writing to the Congress. The Congress,
voting jointly, by a vote of at least a majority of all its Members in regular or special session,
may revoke such proclamation or suspension, which revocation shall not be set aside by
the President. Upon the initiative of the President, the Congress may, in the same manner,
extend such proclamation or suspension for a period to be determined by the Congress, if
the invasion or rebellion shall persist and public safety requires it.
The Congress, if not in session, shall, within twenty-four hours following such proclamation
or suspension, convene in accordance with its rules without need of a call.
The Supreme Court may review, in an appropriate proceeding filed by any citizen, the
sufficiency of the factual basis of the proclamation of martial law or the suspension of the
privilege of the writ or the extension thereof, and must promulgate its decision thereon
within thirty days from its filing.
A state of martial law does not suspend the operation of the Constitution, nor supplant the
functioning of the civil courts or legislative assemblies, nor authorize the conferment of
jurisdiction on military courts and agencies over civilians where civil courts are able to
function, nor automatically suspend the privilege of the writ.
The suspension of the privilege of the writ shall apply only to persons judicially charged for
rebellion or offenses inherent in or directly connected with invasion.
During the suspension of the privilege of the writ, any person thus arrested or detained
shall be judicially chargedwithin three days, otherwise he shall be released.

SANLAKAS V. EXECUTIVE SECRETARY


• On July 27, 2003, some three hundred junior officers and enlisted men of the Armed
Forces of the Philippines stormed into the Oakwood Premiere apartments in Makati
City demanding, among others, the resignation of the President, the Secretary of
Defense, and the Chief of the Philippine National Police.
• In the wake of the Oakwood occupation, the President issued Proclamation No. 427
and General Order No. 4, both declaring “a state of rebellion” and calling out the
Armed Forces to suppress the rebellion.
• Even after the Oakwood occupation ceased after the negotiation conducted, the
President, however, did not immediately lift the declaration of a state of rebellion
and did only on August 1, 2003 through Proclamation No. 435 “DECLARING THAT
THE STATE OF REBELLION HAS CEASED TO EXIST.”

Issue.
• WON Proclamation No. 427 and General Order No. 4 (declaring a state of rebellion)
are constitutional.

Held.
No. Petitions dismissed.
Section 18, Article VII does not expressly prohibit the President from declaring a
state of rebellion. Note that the Constitution vests the President not only with Commander-
in-Chief powers but with Executive powers. The President’s authority to declare a state of
rebellion springs in the main from her powers as chief executive and, at the same time,
draws strength from her Commander-in-Chief powers.
The President has full discretionary power to call out the armed forces and to
determine the necessity for the exercise of such power. While the Court may examine
whether the power was exercised within constitutional limits or in a manner constituting
grave abuse of discretion, none of the petitioners here have, by way of proof, supported
their assertion that the President acted without factual basis.
The argument that the declaration of a state of rebellion amounts to a declaration of
martial law is a leap of logic. There is no indication that the President has exercised judicial
and legislative powers.
The petitions do not cite a specific instance where the President has attempted to or
has exercised powers beyond her powers as Chief Executive or as Commander-in-Chief.
The President, in declaring a state of rebellion and in calling out the armed forces, was
merely exercising a wedding of her Chief Executive and Commander-in-Chief powers.
These are purely executive powers, vested on the President by Sections 1 and 18, Article
VII, as opposed to the delegated legislative powers contemplated by Section 23 (2), Article
VI.

Notes.
• Section 18 grants the President, as Commander-in-Chief, a sequence of graduated
power
a. The calling out power;
- The only criterion is whenever it becomes necessary, the President
may call the armed forces to prevent or suppress lawless violence,
invasion or rebellion.
b. The power to suspend the privilege of the writ of habeas corpus; and
c. The power to declare martial law.
- In the exercise of (B) and (C), the Constitution requires:
1. Actual invasion or rebellion; and
2. That public safety requires the exercise of such power.

IBP v. ZAMORA
President Estrada expressed his desire to improve the peace and order situation in
Metro Manila through a more effective crime prevention program including increased
police patrols. The President further stated that to heighten police visibility in the
metropolis, augmentation from the AFP is necessary. Invoking his powers as Commander-
in-Chief under Section 18, Article VII of the Constitution, the President directed the AFP
Chief of Staff and PNP Chief to coordinate with each other for the proper deployment and
utilization of the Marines to assist the PNP in preventing or suppressing criminal or lawless
violence. The President also declared that the services of the Marines in the anti-crime
campaign are merely temporary in nature and for a reasonable period only, until such time
when the situation shall have improved.

Issue.
• WON the Presidents factual determination of the necessity of calling the armed
forces is subject to judicial review.

Held.
No. The President did not commit grave abuse of discretion in calling out the
Marines.

According to the IBP, no emergency exists that would justify the need for the calling
of the military to assist the police force. It contends that no lawless violence, invasion or
rebellion exist to warrant the calling of the Marines.

As held in Marcos v. Manglapos, the power of the President to keep the peace is not
limited merely to exercising the commander-in-chief powers in times of emergency or to
leading the State against external and internal threats to its existence. The President is not
only clothed with extraordinary powers in times of emergency, but is also tasked with
attending to the day-to-day problems of maintaining peace and order and ensuring
domestic tranquillity in times when no foreign foe appears on the horizon. Wide discretion,
within the bounds of law, in fulfilling presidential duties in times of peace is not in any way
diminished by the relative want of an emergency specified in the commander-in-chief
provision. For in making the President commander-in-chief the enumeration of powers
that follow cannot be said to exclude the Presidents exercising as Commander-in-Chief
powers short of the calling of the armed forces, or suspending the privilege of the writ of
habeas corpus or declaring martial law, in order to keep the peace, and maintain public
order and security.

When the President calls the armed forces to prevent or suppress lawless violence,
invasion or rebellion, he necessarily exercises a discretionary power solely vested in his
wisdom. Court, thus, cannot be called upon to overrule the Presidents wisdom or
substitute its own. However, this does not prevent an examination of whether such power
was exercised within permissible constitutional limits or whether it was exercised in a
manner constituting grave abuse of discretion. It is therefore incumbent upon the
petitioner to show that the President’s decision is totally bereft of factual basis. The present
petition fails to discharge such heavy burden as there is no evidence to support the
assertion that there exists no justification for calling out the armed forces. There is,
likewise, no evidence to support the proposition that grave abuse was committed because
the power to call was exercised in such a manner as to violate the constitutional provision
on civilian supremacy over the military.
The President as Commander-in-Chief has a vast intelligence network to gather
information, some of which may be classified as highly confidential or affecting the security
of the state. In the exercise of the power to call, on-the-spot decisions may be imperatively
necessary in emergency situations to avert great loss of human lives and mass destruction
of property. Indeed, the decision to call out the military to prevent or suppress lawless
violence must be done swiftly and decisively if it were to have any effect at all.

Thus, it is the unclouded intent of the Constitution to vest upon the President, as
Commander-in-Chief of the Armed Forces, full discretion to call forth the military when in
his judgment it is necessary to do so in order to prevent or suppress lawless violence,
invasion or rebellion. Unless the petitioner can show that the exercise of such discretion
was gravely abused, the Presidents exercise of judgment deserves to be accorded respect
from this Court.

LACSON v. PEREZ
On May 1, 2001, President Macapagal-Arroyo, faced by an angry and violent mob
armed with explosives, firearms, bladed weapons, clubs, stones and other deadly weapons
assaulting and attempting to break into Malacanang, issued Proclamation No. 38 declaring
that there was a state of rebellion in the National Capital Region. She likewise issued
General Order No. 1 directing the Armed Forces of the Philippines and the Philippine
National Police to suppress the rebellion in the National Capital Region. Warrantless
arrests of several alleged leaders and promoters of the rebellion were thereafter effected.

Held.
Petitioner Lumbao argues that the declaration of a state of rebellion is violative of
the doctrine of separation of powers, being an encroachment on the domain of the judiciary
which has the constitutional prerogative to determine or interpret what took place on May
1, 2001, and that the declaration of a state of rebellion cannot be an exception to the
general rule on the allocation of the governmental powers.

The Supreme Court disagrees.

To be sure, section 18, Article VII of the Constitution expressly provides that the
President shall be the Commander-in-Chief of all armed forces of the Philippines and
whenever it becomes necessary, he may call out such armed forces to prevent or suppress
lawless violence, invasion or rebellion thus, we held in Integrated Bar of the Philippines v.
Hon. Zamora, (G.R. No. 141284, August 15, 2000):

xxx The factual necessity of calling out the armed forces is not easily quantifiable
and cannot be objectively established since matters considered for satisfying the same is a
combination of several factors which are not always accessible to the courts. Besides the
absence of testual standards that the court may use to judge necessity, information
necessary to arrive at such judgment might also prove unmanageable for the courts.
Certain pertinent information necessary to arrive at such judgment might also prove
unmanageable for the courts. Certain pertinent information might be difficult to verify, or
wholly unavailable to the courts. In many instances, the evidence upon which the President
might decide that there is a need to call out the armed forces may be of a nature not
constituting technical proof.

On the other hand, the President as Commander-in-Chief has a vast intelligence


network to gather information, some of which may be classified as highly confidential or
affecting the security of the state. In the exercise of the power to call, on-the-spot decisions
may be imperatively necessary in emergency situations to avert great loss of human lives
and mass destruction of property. xxx

The Court, in a proper case, may look into the sufficiency of the factual basis of the
exercise of this power. However, this is no longer feasible at this time, Proclamation No. 38
having been lifted.
OLAGUER v. MILITARY
Petitioners were all civilians arrested by military authorities and were charged for
subversion. The Chief of Staff of the Armed Forces of the Philippines created the
respondent Military Commission No 34 to try tile criminal case filed against the petitioners.
Petitioners alleged that military commissions have no jurisdiction to try civilians for
offenses alleged to have been committed during the period of martial law. They also
maintain that the proceedings before the respondent Military Commission No. 34 are in
gross violation of their constitutional right to due process of law. Subsequently, President
Marcos lifted martial law and directed that “the military tribunals created pursuant thereto
are hereby dissolved upon final determination of case's pending therein which may not be
transferred to the civil courts without irreparable prejudice to the state in view of the rules
on double jeopardy, or other circumstances which render prosecution of the cases difficult,
if not impossible.”

Issue.
WON military commissions or tribunals have the jurisdiction to try civilians for
offenses allegedly committed during martial law when civil courts are open and
functioning.

Held.
No.

Military tribunals pertain to the Executive Department of the Government and are
simply instrumentalities of the executive power, provided by the legislature for the
President as Commander-in-Chief to aid him in properly commanding the army and navy
and enforcing discipline therein, and utilized under his orders or those of his authorized
military representatives. Following the principle of separation of powers underlying the
existing constitutional organization of the Government of the Philippines, the power and
the duty of interpreting the laws as when an individual should be considered to have
violated the law) is primarily a function of the judiciary. It is not, and it cannot be the
function of the Executive Department, through the military authorities. And as long as the
civil courts in the land remain open and are regularly functioning, as they do so today and
as they did during the period of martial law in the country, military tribunals cannot try
and exercise jurisdiction over civilians for offenses committed by them and which are
properly cognizable by the civil courts. To have it otherwise would be a violation of the
constitutional right to due process of the civilian concerned.

It must also be noted that Proclamation No. 2045 officially lifting martial law in the
Philippines and abolishing all military tribunals created pursuant to the national
emergency effectively divests the respondent Military Commission No. 34 (and all military
tribunals for that matter) of its supposed authority to try civilians, including the herein
petitioners. By virtue of the proclamation itself, all cases against civilians pending therein
should eventually be transferred to the civil courts for proper disposition.

As long as the civil courts in the land are open and functioning, military tribunals
cannot try and exercise jurisdiction over civilians for offenses committed by them. Whether
or not martial law has been proclaimed throughout the country or over a part thereof is of
no moment. The imprimatur for this observation is found in Section 18, Article VII of the
1987 Constitution, to wit —
A state of martial law, does not suspend the operation of the Constitution, nor supplant the
functioning of the civil courts or legislative assemblies, nor authorize the conferment of
jurisdiction on military courts and agencies over civilians where civil courts are able to
function, nor automatically suspend the privilege of the writ.

DAVID v. MACAPAGAL-ARROYO
During EDSA I celebration, PGMA issued Presidential Proclamation 1017 declaring a
State of National Emergency invoking Sec 18, Art VII of the Constitution and calling out the
AFP to maintain peace and order and suppress lawless violence, invasion or rebellion due
to the alliance of the extreme left (NDF-CPP-NPA) and extreme right (Military
Adventurists) threatening to bring down the President.
The respondents provided the following, among others, as factual bases for the
assailed proclamation
• Authorities discovered documents regarding “Operation Hacle 1”, a plan of
bombings during the PMA Alumni Homecoming which aims to assassinate the
President
• Flash disks containing information about meetings between Magdalo and NPA and
the planned “Magdalo D-day” on Feb 24, 06
• B/Gen. Danilo Lim and Col. Ariel Querubin exposing anti-Arroyomovements in the
military and plotting to break the AFP chain of command for a movement against
the administration.
• The bombing of telecommunication towers and cell sites in Bulacan and Bataan.

Because of this, rally permits issued by local government units were cancelled.
However, rallies from KMU and NAFLU-KMU during Edsa 1 anniversary proceeded but
were violently dispersed by authorities. This also caused the warrantless arrest of
petitioners David and Llamas and raid of the Daily Tribune, Malaya and Abante offices and
confiscation of news stories and various documents. Congressman Beltran was also
arrested by the police showing a 1985 warrant from the Marcos regime and attempts on
the arrest of SaturOcampo, Rafael Mariano, et. al.

Held.
WON the proclamation has sufficient factual bases

The standard laid down is not correctness, but arbitrariness. In Integrated Bar of
the Philippines, this Court further ruled that it is incumbent upon the petitioner to show
that the President’s decision is totally bereft of factual basis and that if he fails, by way of
proof, to support his assertion, then this Court cannot undertake an independent
investigation beyond the pleadings.

Petitioners failed to show that President Arroyo’s exercise of the calling-out power,
by issuing PP 1017, is totally bereft of factual basis. OSG’s memorandum shows a detailed
narration of the events leading to the issuance of PP 1017 such as the escape of the
Magdalo Group, their audacious threat of the MagdaloD-Day, the defections in the military,
particularly in the Philippine Marines, and the reproving statements from the communist
leaders. There was also the Minutes of the Intelligence Report and Security Group of the
Philippine Army showing the growing alliance between the NPA and the military. Absent
any contrary allegations, the Court is convinced that the President was justified in issuing
PP 1017 calling for military aid.

WON the proclamation was valid in relation to the President’s “calling-out” power as
commander-in-chief

President Arroyo’s declaration of a state of rebellion was merely an act declaring a


status or condition of public moment or interest, a declaration allowed under Section 4
cited above. Such declaration, in the words of Sanlakas, is harmless, without legal
significance, and deemed not written. In these cases, PP 1017 is more than that. In
declaring a state of national emergency, President Arroyo did not only rely on Section 18,
Article VII of the Constitution, a provision calling on the AFP to prevent or suppress lawless
violence, invasion or rebellion. She also relied on Section 17, Article XII, a provision on the
State’s extraordinary power to take over privately-owned public utility and business
affected with public interest. Indeed, PP 1017 calls for the exercise of an awesome power.
Obviously, such Proclamation cannot be deemed harmless, without legal significance, or
not written, as in the case of Sanlakas.

Contrary to the petitioner’s contention that PP 1017 is a declaration of martial law,


it is no more than a call by the President to the armed forces to prevent or suppress lawless
violence. As such, it cannot be used to justify acts that only under a valid declaration of
Martial Law can be done. Specifically, (a) arrests and seizures without judicial warrants;
(b) ban on public assemblies; (c) take-over of news media and agencies and press
censorship; and (d) issuance of Presidential Decrees, are powers which can be exercised by
the President as Commander-in-Chief only where there is a valid declaration of Martial Law
or suspension of the writ of habeas corpus.
It is clear that PP 1017 is not a declaration of Martial Law. It is merely an exercise of
President Arroyo’s calling-out power for the armed forces to assist her in preventing or
suppressing lawless violence.

GUDANI v. SENGA
The Senate invited Gen. Gudani and Lt. Col. Balutan to clarify allegations of 2004
election fraud and the surfacing of the “Hello Garci” tapes. PGMA issued EO 464 enjoining
officials of the executive department including the military establishment from appearing
in any legislative inquiry without her consent. AFP Chief of Staff Gen. Senga issued a
Memorandum, prohibiting Gen. Gudani, Col. Balutan et al from appearing before the Senate
Committee without Presidential approval. However, the two appeared before the Senate in
spite the fact that a directive has been given to them. As a result, the two were relieved of
their assignments for allegedly violating the Articles of War and the time honoured
principle of the “Chain of Command.” Gen. Senga ordered them to be subjected before the
General Court Martial proceedings for wilfully violating an order of a superior officer.

Issue.
WON the President has the authority to issue an order to the members of the AFP
preventing them from testifying before a legislative inquiry.

Held.
Yes.

The commander-in-chief provision in the Constitution is denominated as Section 18,


Article VII, which begins with the simple declaration that the President shall be the
Commander-in-Chief of all armed forces of the Philippines x xx Outside explicit
constitutional limitations, such as those found in Section 5, Article XVI, the commander-in-
chief clause vests on the President, as commander-in-chief, absolute authority over the
persons and actions of the members of the armed forces. Such authority includes the ability
of the President to restrict the travel, movement and speech of military officers, activities
which may otherwise be sanctioned under civilian law. To a certain degree, individual
rights may be curtailed, because the effectiveness of the military in fulfilling its duties
under the law depends to a large extent on the maintenance of discipline within its ranks.
Hence, lawful orders must be followed without question and rules must be faithfully
complied with, irrespective of a soldier's personal views on the matter.

Critical to military discipline is obeisance to the military chain of command. Wilful


disobedience of a superior officer is punishable by court-martial under Article 65 of the
Articles of War. If they were permitted to act upon their own opinion of their rights, they
can throw off the authority of the commander whenever they supposed it to be unlawfully
exercised.

Further traditional restrictions on members of the armed forces are those imposed
on free speech and mobility. Kapunan is ample precedent in justifying that a soldier may be
restrained by a superior officer from speaking out on certain matters. As a general rule, the
discretion of a military officer to restrain the speech of a soldier under his/her command
will be accorded deference, with minimal regard if at all to the reason for such restraint. It
is integral to military discipline that the soldier’s speech be with the consent and approval
of the military commander.

The Constitution itself recognizes as one of the legislatures functions is the conduct
of inquiries in aid of legislation. Inasmuch as it is ill-advised for Congress to interfere with
the Presidents power as commander-in-chief, it is similarly detrimental for the President to
unduly interfere with Congress’ right to conduct legislative inquiries. The refusal of the
President to allow members of the military to appear before Congress is still subject to
judicial relief. The duty falls on the shoulders of the President, as commander-in-chief, to
authorize the appearance of the military officers before Congress. Even if the President has
earlier disagreed with the notion of officers appearing before the legislature to testify, the
Chief Executive is nonetheless obliged to comply with the final orders of the courts.

AMPATUAN v. PUNO
1. November 24, 2009- the day after the gruesome massacre of 57 men and women,
PGMA issued Proclamation 1946, placing “the Provinces of Maguindanao and Sultan
Kudarat and the City of Cotabato under a state of emergency.”
2. She directed the AFP and the PNP “to undertake such measures as may be allowed
by the Constitution and by law to prevent and suppress all incidents of lawless
violence” in the named places.
3. 3 days after - Under AO 273-A, she also delegated to the DILG the supervision of the
ARMM.
4. The petitioners claimed that the President’s issuances encroached the ARMM’s
autonomy, that it constitutes an invalid exercise of emergency powers, and that the
President had no factual basis for declaring a state of emergency, especially in the
Province of Sultan Kudaratand the City of Cotabato, where no critical violent
incidents occurred. They want Proc. 1946 and AO 273-A be declared
unconstitutional.

Issues/Held.
1. WON PGMA validly exercised emergency powers when she called out the AFP and
the PNP to prevent and suppress lawless violence in Maguindanao, Sultan Kudarat,
and Cotabato City
• Yes.
• Such deployment is not by itself an exercise of emergency powers as
understood under Section 23 (2), Article VI of the Constitution. The
President did not proclaim a national emergency, only a state of emergency
in the three places mentioned. And she did not act pursuant to any law
enacted by Congress that authorized her to exercise extraordinary powers.
The calling out of the armed forces to prevent or suppress lawless violence in
such places is a power that the Constitution directly vests in the President.
She did not need a congressional authority to exercise the same.
2. WON the President had factual bases for her actions.
• The President’s call on the armed forces to prevent or suppress lawless
violence springs from the power vested in her under Section 18, Article VII of
the Constitution, which provides.
SECTION 18. The President shall be the Commander-in-Chief of all
armed forces of the Philippines and whenever it becomes necessary,
he may call out such armed forces to prevent or suppress lawless
violence, invasion or rebellion. x xx
• As the Court acknowledged in Integrated Bar of the Philippines v. Hon.
Zamora, it is clearly to the President that the Constitution entrusts the
determination of the need for calling out the armed forces to prevent and
suppress lawless violence. Unless it is shown that such determination was
attended by grave abuse of discretion, the Court will accord respect to the
President’s judgment.
• Petitioners failed to show that the declaration of a state of emergency in the
Provinces of Maguindanao, Sultan Kudarat and Cotabato City, as well as the
President’s exercise of the "calling out" power had no factual basis. They
simply alleged that, since not all areas under the ARMM were placed under a
state of emergency, it follows that the take over of the entire ARMM by the
DILG Secretary had no basis too. OSG also explained the factual bases for the
President’s decision to call out the armed forces.
• The imminence of violence and anarchy at the time the President issued
Proclamation 1946 was too grave to ignore and she had to act to prevent
further bloodshed and hostilities in the places mentioned. Thus, to pacify the
people’s fears and stabilize the situation, the President had to take
preventive action and called out the armed forces to control the proliferation
of loose firearms and dismantle the armed groups that continuously
threatened the peace and security in the affected places.

D. POWER OF EXECUTIVE CLEMENCY (Sec. 19)

Section 19. Except in cases of impeachment, or as otherwise provided in this Constitution,


the President may grant reprieves, commutations, and pardons, and remit fines and
forfeitures, after conviction by final judgment.

He shall also have the power to grant amnesty with the concurrence of a majority of all the
Members of the Congress

ECHEGARAY v. SECRETARY OF JUSTICE


Despite the decision on the case being final and executor, the Supreme Court issued a
temporary restraining order to the execution of Leo Echegaray on the same day. The
petitioners contended the Court entered into the exclusive ambit of authority of the
executive authority. The issuance of the TRO may be construed as trenching on that sphere
of executive authority;

Issue.
• WON the Supreme Court’s assailed resolution granting the TRO in effect a grant of
reprieve which is an executive function.

Held.
• No.
• Section 19, Article VII of the Constitution:
Except in cases of impeachment, or as otherwise provided in this Constitution,
the President may grant reprieves, commutations, and pardons, and remit fines
and forfeitures after conviction by final judgment. He shall also have the power
to grant amnesty with the concurrence of a majority of all the members of the
Congress.
• The provision is simply the source of power of the President to grant reprieves,
commutations, and pardons and remit fines and forfeitures after conviction by final
judgment. It also provides the authority for the President to grant amnesty with the
concurrence of a majority of all the members of the Congress. The provision,
however, cannot be interpreted as denying the power of courts to control the
enforcement of their decisions after their finality. In truth, an accused who has been
convicted by final judgment still possesses collateral rights and these rights can be
claimed in the appropriate courts.
• The powers of the Executive, the Legislative and the Judiciary to save the life of a
death convict do not exclude each other for the simple reason that there is no higher
right than the right to life. For the public respondents therefore to contend that only
the Executive can protect the right to life of an accused after his final conviction is to
violate the principle of co-equal and coordinate powers of the three branches of our
government.

MONSANTO v. FACTORAN
Monsanto was charged with complex crime of estafa thru falsification of public
documents and sentenced them to imprisonment of four (4) years, two (2) months and one
(1) day of prisioncorreccional as minimum, to ten (10) years and one (1) day of prision
mayor as maximum, and to pay a fine of P3,500. Pending appeal of her conviction, Pres.
Marcos extended an absolute pardon which she duly accepted. By reason of said pardon,
petitioner wrote the Calbayog City treasurer requesting that she be restored to her former
post as assistant city treasurer since the same was still vacant.
Issue.
• WON a public officer, who has been granted an absolute pardon by the Chief
Executive, is entitled to reinstatement to her former position without need of a new
appointment.

Held.
• No.
• As the penalty was prision mayor which carries with it accessory of temporary
absolute disqualification, this bars the convict from public office or employment,
such disqualification to last during the term of the sentence. Even if the offender be
pardoned, as to the principal penalty, the accessory penalties remain unless the
same have been expressly remitted by the pardon. The penalty of
prisioncorreccional carries, as one of its accessory penalties, suspension from public
office.
• Pardon granted after conviction frees the individual from all the penalties and legal
disabilities and restores him to all his civil rights. But unless expressly grounded on
the person's innocence (which is rare), it cannot bring back lost reputation for
honesty, integrity and fair dealing.
• Pardon does not ipso facto restore a convicted felon to public office necessarily
relinquished or forfeited by reason of the conviction although such pardon
undoubtedly restores his eligibility for appointment to that office.
• In the case at bar, the absolute disqualification or ineligibility from public office
forms part of the punishment prescribed by the Revised Penal Code for estafa thru
falsification of public documents. It is clear from the authorities referred to that
when her guilt and punishment were expunged by her pardon, this particular
disability was likewise removed. Henceforth, petitioner may apply for
reappointment to the office which was forfeited by reason of her conviction. And in
considering her qualifications and suitability for the public post, the facts
constituting her offense must be and should be evaluated and taken into account to
determine ultimately whether she can once again be entrusted with public funds.
Stated differently, the pardon granted to petitioner has resulted in removing her
disqualification from holding public employment but it cannot go beyond that. To
regain her former post as assistant city treasurer, she must re-apply and undergo
the usual procedure required for a new appointment.

Note:
• Pardon is defined as "an act of grace, proceeding from the power entrusted with the
execution of the laws, which exempts the individual, on whom it is bestowed, from
the punishment the law inflicts for a crime he has committed. It is the private,
though official act of the executive magistrate, delivered to the individual for whose
benefit it is intended, and not communicated officially to the Court. ... A pardon is a
deed, to the validity of which delivery is essential, and delivery is not complete
without acceptance."
• A pardon looks to the future. It is not retrospective. It makes no amends for the
past. It affords no relief for what has been suffered by the offender. It does not
impose upon the government any obligation to make reparation for what has been
suffered. "

GARCIA v. COA
1. Petitioner was a Supervising Lineman in the Region IV Station of the Bureau of
Telecommunications (Lucena City).
2. 1 April 1975: Petitioner was dismissed from service on the ground of dishonesty in
accordance with the decisions of then Ministry of Public Works (for loss of several
telegraph poles which were located at Sariaya-Lucena and Mauban-Sampaloc). He
did not appeal from the decision.
3. A criminal case for qualified theft was also filed against the petitioner with the CFI.
On 23 January 1980, the same acquitted him of the offense.
4. Petitioner’s request for reinstatement with the Bureau of Telecommunications was
denied. Hence, he pleased to the President for executive clemency.
5. Pursuant to endorsements of the Ministry of Communications and the Civil Service
Commission, Deputy Presidential Executive Assistant, issued Resolution OP 1800,
granting executive clemency to the same.
6. COA denied Garcia’s petition for payment of back salaries effective from the date of
his dismissal. This was denied by COA on the ground that executive clemency did
not provide for payment of back salaries and that he was not reinstated.
7. Upon reinstatement, petitioner filed the same petition w/ COA. This was again,
denied by COA. They claimed that the clemency was silent on payment of back
wages.
8. Petitioner appealed the decisions to the Office of the President, but this was denied
because SC was the proper forum for the appeal.

Issue.
• WON Petitioner is entitled to the payment of back wages after having been
reinstated pursuant to the grant of executive clemency.

Held.
• Yes. Petition is granted.
• If the pardon is based on the innocence of the individual, it affirms this innocence
and makes him a new man and as innocent; as if he had not been found guilty of the
offense charged. When a person is given pardon because he did not truly commit
the offense, the pardon relieves the party from all punitive consequences of his
criminal act, thereby restoring to him his clean name, good reputation and
unstained character prior to the finding of guilt.
• The acquittal of petitioner by the trial court was founded not on lack of proof
beyond reasonable doubt but on the fact that petitioner did not commit the offense
imputed to him. Verily, petitioner's innocence is the primary reason behind the
grant of executive clemency to him. The bestowal of executive clemency on
petitioner in effect completely obliterated the adverse effects of the administrative
decision which found him guilty of dishonesty and ordered his separation from the
service. This can be inferred from the executive clemency itself exculpating
petitioner from the administrative charge and thereby directing his reinstatement,
which is rendered automatic by the grant of the pardon. This signifies that
petitioner need no longer apply to be reinstated to his former employment; he is
restored to his office ipso facto upon the issuance of the clemency.
• Petitioner's automatic reinstatement to the government service entitles him to back
wages. This is meant to afford relief to petitioner who is innocent from the start and
to make reparation for what he has suffered as a result of his unjust dismissal from
the service. The right to back wages is afforded to those with have been illegally
dismissed and were thus ordered reinstated or to those otherwise acquitted of the
charges against them.

TORRES v. GONZALES
Torres was convicted of estafa and was sentenced to an aggregate prison term of 11
years, 10 months, and 22 days to 38 years, 9 months, and 1 day and to pay indemnity.
Conditional pardon was granted to Torres on condition that he would not again violate any
of the penal laws of the Philippines. Board of Pardons and Parole resolved to recommend
to the President the cancellation of the conditional pardon due to his multiple violations of
the law (being charged with 20 counts of estafa, convicted of sedition, etc.) President
cancelled Torres’ conditional pardon. Minister of Justice issued by authority of the
President an Order of Arrest and Recommitment against Torres
Issue.
• WON conviction of a crime by final judgment of a court is necessary before the
petitioner can be validly rearrested and recommitted for violation of the terms of
his conditional pardon and accordingly to serve the balance of his original sentence

Held.
• NO.
• The grant of pardon and the determination of the terms and conditions of a
conditional pardon are purely executive acts which are not subject to judicial
scrutiny.
• The determination of the occurrence of a breach of a condition of a pardon, and the
proper consequences of such breach, may be either a purely executive act, not
subject to judicial scrutiny under Section 64 (i) of the Revised Administrative Code;
or it may be a judicial act consisting of trial for and conviction of violation of a
conditional pardon under Article 159 of the Revised Penal Code. Where the
President opts to proceed under Section 64 (i) of the Revised Administrative Code,
no judicial pronouncement of guilt of a subsequent crime is necessary, much less
conviction therefor by final judgment of a court, in order that a convict may be
recommended for the violation of his conditional pardon.
• Because due process is not semper et unique judicial process, and because the
conditionally pardoned convict had already been accorded judicial due process in
his trial and conviction for the offense for which he was conditionally pardoned,
Section 64 (i) of the Revised Administrative Code is not afflicted with a
constitutional vice.
• What is involved in the instant case is not the prosecution of the parolee for a
subsequent offense in the regular course of administration of the criminal law. What
is involved is rather the ascertainment of whether the convict has breached his
undertaking that he would "not again violate any of the penal laws of the
Philippines" for purposes of reimposition upon him of the remitted portion of his
original sentence. The consequences that we here deal with are the consequences of
an ascertained breach of the conditions of a pardon. A convict granted conditional
pardon who is recommitted must of course be convicted by final judgment of a court
of the subsequent crime or crimes with which he was charged before the criminal
penalty for such subsequent offense(s) can be imposed upon him.
• In proceeding against a convict who has been conditionally pardoned and who is
alleged to have breached the conditions of his pardon, the Executive Department
has two options: (i) to proceed against him under Section 64 (i) of the Revised
Administrative Code; or (ii) to proceed against him under Article 159 of the Revised
Penal Code which imposes the penalty of prisioncorreccional, minimum period,
upon a convict who "having been granted conditional pardon by the Chief Executive,
shall violate any of the conditions of such pardon." Here, the President has chosen to
proceed against the petitioner under Section 64 (i) of the Revised Administrative
Code. That choice is an exercise of the President's executive prerogative and is not
subject to judicial scrutiny.

BARRIOQUINTO v. FERNANDEZ
Petitioners were charged with murder. CIF of Zamboanga then sentenced the petitioner
to life imprisonment. Before the period for perfecting an appeal has expired, petitioner
decided to admit their case to Guerilla Amnesty Commission. After the preliminary
hearing, the Amnesty Commission returned the case to the lower court, without deciding
whether or not they are entitled of the benefits of the said amnesty proclamation, on the
ground that inasmuch as neither Barrioquinto nor Jimenez have admitted having
committed the offense, because Barrioquinto alleged that it was Tolentino who shot and
killed the victim, they cannot invoke the benefits of amnesty.

Held.
• Respondents are hereby ordered to hear and decide the application of the
petitioners for amnesty.
• In order to entitle a person to the benefits of the Amnesty Proclamation, it is not
necessary that he should, as a condition sine qua non, admit having committed the
criminal act or offense with which he is charged and allege the amnesty as a defense.
It is sufficient that the evidence either of the complainant or the accused shows that
the offense committed comes within the terms of said Amnesty Proclamation.
• The right to the benefits of amnesty, once established by the evidence presented
either by the complainant or prosecution, or by the defense, cannot be waived,
because it is of public interest that a person who is regarded by the Amnesty
Proclamation which has the force of a law, not only as innocent, for he stands in the
eyes of the law as if he had never committed any punishable offense because of the
amnesty, but as a patriot or hero, cannot be punishment as a criminal.
• There is no necessity for an accused to admit his responsibility for the commission
of a criminal act before a court of Amnesty Commission may investigate and extend
or not to him the benefits of amnesty. The fact that he pleads not guilty or that he
has not committed the act with which he is charged, does not necessarily prove that
he is not guilty thereof. Notwithstanding his denial, the evidence for the prosecution
or complainant may show the contrary, as it is generally the case in criminal
proceedings, and what should in such a case be determined is whether or not the
offense committed is of political character.

PEOPLE v. CASIDO
1. In a Resolution dated 30 July 1996, the Court ruled that the conditional
pardons granted to accused-appellants William Casido and Franklin Alcorin are
void for having been extended on 19 January 1996 during the pendency of their
instant appeal.
2. In a Comment for the members of the Presidential Committee for the Grant of Bail,
Release or Pardon, two members of the Committee's Secretariat, submitted that:
a. In line with the confidence building measures of government, the President
on August 11, 1992 constituted the Presidential Committee for the Grant of
Bail, Release or Pardon with the directive to establish guidelines for the grant
of bail, release or pardon of persons detained or convicted of crimes against
national security and public order and violations of the Articles of War.
b. On 9 December 1992, the President issued an amendment to the
guidelines incorporating therein a provision which reads: "Those charged,
detained or convicted of common crimes but who can establish by
sufficient evidence that they have actually committed any of the
crimes/offenses enumerated above may apply for possible grant of bail,
release or pardon under these guidelines."
c. Corollary to the constitution of the Committee, a Secretariat was also
constituted which was tasked to process and evaluate the applications of
those desiring to be granted pardon or recommended for release or bail
under the aforementioned guidelines and which will recommend to the
Committee those who qualify under the guidelines.
d. The applications for conditional pardon of the aforenamed prisoners were
recommended by the Committee to the President for the grant of
Conditional Pardon (after the Secretariat had evaluated that the former
committed the crimes for which they had been charged in pursuit of their
political belief) per Memorandum dated May 25, 1995 and approved by the
President on December 29, 1995. The Conditional Pardon paper was signed
by the President on January 19, 1996 and the subject prisoners
(accused-appellants) were released by the Bureau of Corrections on January
25, 1996.
e. Prior to their release, subject prisoners filed an "Urgent Motion to
Withdraw Appeal" which was received by the Supreme Court on January
11, 1996. Unfortunately, the Committee failed to verify first whether the
counsel of the accused had also withdrawn their appeal or that the NGO
lawyers had filed in their behalf a motion to Withdraw their Appeal. It was
upon the honest belief of the Secretariat that the NGO lawyers would
perform their agreed undertaking, that the Secretariat indorsed the
applications for conditional pardon of subject prisoners for favorable
action by the Committee, and thereafter by the President.
f. There was no intention on the part of the Secretariat and the Presidential
Committee to violate Section 19, Article VII, of the Constitution, but that what
happened was a clear misappreciation of fact.
3. Earlier, the Court received a letter informing the Court that the applications for
amnesty of Alcorin and Casido were "favorably acted" upon by the National
Amnesty Commission.
4. OSG alleged that the accused-appellants in this case, "in an effort to seek their
release at the soonest possible time, applied for pardon before the Presidential
Committee on the Grant of Bail, Release or Pardon (PCGBRP), as well as for amnesty
before the National Amnesty Commission (NAC)"; then contended that since
amnesty, unlike pardon, may be granted before or after the institution of the
criminal prosecution and sometimes even after conviction, as held in Barrioquinto v.
Fernandez, the amnesty then granted accused-appellants Casido and Alcorin
"rendered moot and academic the question of the premature pardon granted to
them."

Issue.
• Whether or not the pardon granted in this case is void for having been
extended during the pendency of the appeal or before conviction by final judgment
and, therefore, in violation of the first paragraph of Section 19, Article VII of the
Constitution.

Held.
• While the pardon in this case was void for having been extended during the
pendency of the appeal or before conviction by final judgment and, therefore, in
violation of the first paragraph of Section 19, Article VII of the Constitution, the
grant of the amnesty, for which accused-appellants William Casido and Franklin
Alcorin voluntarily applied under Proclamation No. 347, 3 was valid. This
Proclamation was concurred in by both Houses of Congress in Concurrent
Resolution No.12 adopted on 2 June 1994.
• The release then of accused-appellants William Casido and Franklin Alcorin can only
be justified by the amnesty, but not by the "pardon."
• As to the "pardon," in no uncertain terms the necessity of a final judgment before
parole or pardon could be extended. Members of the Secretariat were aware that a
pardon could only be granted after conviction by final judgment; hence, they
required from the lawyers of the applicants the filing with this Court of "motions for
the withdrawal of the applicants' appeals." Thus, they cannot plead ignorance of this
condition sine qua non to the grant of pardon. They should have demanded from the
applicants the submission of proof of their compliance of the requirement before
submitting to the President a favorable recommendation. That alone, at the very
least, could have been the basis of a finding of good faith. In failing to observe due
care in the performance of their duties, the Members of the Committee caused the
President serious embarrassment and thus deserve an admonition.

PEOPLE v. PATRIARCA
• Accused-appellant applied for amnesty under Proclamation No. 724 amending
Proclamation No. 347, dated March 25, 1994, entitled "Granting Amnesty to Rebels,
Insurgents, and All Other Persons Who Have or May Have Committed Crimes
Against Public Order, Other Crimes Committed in Furtherance of Political Ends, and
Violations of the Article of War, and Creating a National Amnesty Commission." His
application was favorably granted by the National Amnesty Board. Attached to
appellant's brief is the Notice of Resolution of the National Amnesty Commission
(NAC).
Amnesty commonly denotes a general pardon to rebels for their treason or other high
political offenses, or the forgiveness which one sovereign grants to the subjects of another,
who have offended, by some breach, the law of nations. Amnesty looks backward, and
abolishes and puts into oblivion, the offense itself; it so overlooks and obliterates the
offense with which he is charged, that the person released by amnesty stands before the
law precisely as though he had committed no offense. Paragraph 3 of Article 89 of the
Revised Penal Code provides that criminal liability is totally extinguished by amnesty,
which completely extinguishes the penalty and all its effects.

The Court took judicial notice of the grant of amnesty upon accused-appellant Jose N.
Patriarca, Jr. Once granted, it is binding and effective. It serves to put an end to the appeal

E. BORROWING POWER (Sec. 20)

Section 20. The President may contract or guarantee foreign loans on behalf of the
Republic of the Philippines with the prior concurrence of the Monetary Board, and subject
to such limitations as may be provided by law. The Monetary Board shall, within thirty days
from the end of every quarter of the calendar year, submit to the Congress a complete
report of its decision on applications for loans to be contracted or guaranteed by the
Government or government-owned and controlled corporations which would have the
effect of increasing the foreign debt, and containing other matters as may be provided by
law.
CONSTANTINO V. CUISIA
Facts:
• Buy-back programs and bond-conversion programs were initiated by the Corazon
Aquino administration to reduce the country’s external debt.
• The petitioners object to the debt-relief contracts entered into pursuant to the
Financing Program as beyond the powers granted to the President under Sec. 20,
Art. VII of the Constitution.

Issue: Whether or not the programs were unconstitutional.

Ruling: The programs were constitutional.


• Sec. 20, Art. VII of the Constitution provides, viz:
The President may contract or guarantee foreign loans in behalf of the
Republic of the Philippines with the prior concurrence of the
Monetary Board and subject to such limitations as may be provided
under law. The Monetary Board shall, within thirty days from the end
of every quarter of the calendar year, submit to the Congress a
complete report of its decisions on applications for loans to be
contracted or guaranteed by the government or government-owned
and controlled corporations which would have the effect of increasing
the foreign debt, and containing other matters as may be provided by
law.

• On Bond-Conversion:
The word bond means contract, agreement, or guarantee. All of these terms
are applicable to the securities known as bonds. An investor who purchases a
bond is lending money to the issuer, and the bond represents the issuers
contractual promise to pay interest and repay principal according to specific
terms. A short-term bond is often called a note.

The language of the Constitution is simple and clear as it is broad. It allows


the President to contract and guarantee foreign loans. It makes no
prohibition on the issuance of certain kinds of loans or distinctions as to
which kinds of debt instruments are more onerous than others. This Court
may not ascribe to the Constitution meanings and restrictions that would
unduly burden the powers of the President. The plain, clear and
unambiguous language of the Constitution should be construed in a sense
that will allow the full exercise of the power provided therein. It would be the
worst kind of judicial legislation if the courts were to misconstrue and
change the meaning of the organic act.

• On the Buyback Scheme:


Buyback is a necessary power which springs from the grant of the foreign
borrowing power. Every statute is understood, by implication, to contain all
such provisions as may be necessary to effectuate its object and purpose, or
to make effective rights, powers, privileges or jurisdiction which it grants,
including all such collateral and subsidiary consequences as may be fairly
and logically inferred from its terms. The President is not empowered to
borrow money from foreign banks and governments on the credit of the
Republic only to be left bereft of authority to implement the payment despite
appropriations therefor.

Even petitioners concede that [t]he Constitution, as a rule, does not


enumerate let alone enumerate all the acts which the President (or any other
public officer) may not do,and [t]he fact that the Constitution does not
explicitly bar the President from exercising a power does not mean that he or
she does not have that power.

It is inescapable from the standpoint of reason and necessity that the


authority to contract foreign loans and guarantees without restrictions on
payment or manner thereof coupled with the availability of the
corresponding appropriations, must include the power to effect payments or
to make payments unavailing by either restructuring the loans or even
refusing to make any payment altogether.

F. DIPLOMATIC POWERS (Sec. 21)


Section 21. No treaty or international agreement shall be valid and effective unless
concurred in by at least two-thirds of all the Members of the Senate.
1. FOREIGN AFFAIRS POWER

VINUYA V. EXEC. SEC. ROMULO


Facts:
• Petitioners are all members of the MALAYA LOLAS, a non-stock, non-profit
organization registered with the Securities and Exchange Commission, established
for the purpose of providing aid to the victims of rape by Japanese military forces in
the Philippines during the Second World War.
• Petitioners narrate that during the Second World War, the Japanese army attacked
villages and systematically raped the women as part of the destruction of the
village. Their communities were bombed, houses were looted and burned, and
civilians were publicly tortured, mutilated, and slaughtered. Japanese soldiers
forcibly seized the women and held them in houses or cells, where they were
repeatedly raped, beaten, and abused by Japanese soldiers. As a result of the actions
of their Japanese tormentors, the petitioners have spent their lives in misery, having
endured physical injuries, pain and disability, and mental and emotional suffering.
• Petitioners claim that since 1998, they have approached the Executive Department
through the DOJ, DFA, and OSG, requesting assistance in filing a claim against the
Japanese officials and military officers who ordered the establishment of the
"comfort women" stations in the Philippines.
• However, officials of the Executive Department declined to assist the petitioners,
and took the position that the individual claims of the comfort women for
compensation had already been fully satisfied by Japan’s compliance with the Peace
Treaty between the Philippines and Japan.
• Petitioners: The general waiver of claims made by the Philippine government in the
Treaty of Peace with Japan is void. They claim that the comfort women system
established by Japan, and the brutal rape and enslavement of petitioners
constituted a crime against humanity,sexual slavery, and torture.They allege that
the prohibition against these international crimes is jus cogens norms from which
no derogation is possible; as such, in waiving the claims of Filipina comfort women
and failing to espouse their complaints against Japan, the Philippine government is
in breach of its legal obligation not to afford impunity for crimes against humanity.
Finally, petitioners assert that the Philippine government’s acceptance of the
"apologies" made by Japan as well as funds from the Asian Women’s Fund (AWF)
were contrary to international law.
• Respondents: Respondents maintain that all claims of the Philippines and its
nationals relative to the war were dealt with in the San Francisco Peace Treaty of
1951 and the bilateral Reparations Agreement of 1956.

Issue: Whether the petitioners can compel the respondents (basically the Executive) to
espouse their claims for official apology and other forms of reparations against Japan
before the International Court of Justice (ICJ) and other international tribunals.

Ruling: The petition is dismissed.


• It is well-established that "[t]he conduct of the foreign relations of our government
is committed by the Constitution to the executive and legislative--'the political'--
departments of the government, and the propriety of what may be done in the
exercise of this political power is not subject to judicial inquiry or decision."
• To be sure, not all cases implicating foreign relations present political questions, and
courts certainly possess the authority to construe or invalidate treaties and
executive agreements.However, the question whether the Philippine government
should espouse claims of its nationals against a foreign government is a foreign
relations matter, the authority for which is demonstrably committed by our
Constitution not to the courts but to the political branches.
• In this case, the Executive Department has already decided that it is to the best
interest of the country to waive all claims of its nationals for reparations against
Japan in the Treaty of Peace of 1951. The wisdom of such decision is not for the
courts to question. Neither could petitioners herein assail the said determination by
the Executive Department via the instant petition for certiorari.
• Of course, we greatly sympathize with the cause of petitioners, and we cannot begin
to comprehend the unimaginable horror they underwent at the hands of the
Japanese soldiers. We are also deeply concerned that, in apparent contravention of
fundamental principles of law, the petitioners appear to be without a remedy to
challenge those that have offended them before appropriate fora. Needless to say,
our government should take the lead in protecting its citizens against violation of
their fundamental human rights. Regrettably, it is not within our power to order the
Executive Department to take up the petitioners’ cause. Ours is only the power
to urge and exhort the Executive Department to take up petitioners’ cause.

2. TREATY AND EXECUTIVE AGREEMENTS

USSAFE V. TREASURER
Facts:
• The central issue in this litigation concerns the validity of the Romulo-Snyder
Agreement (1950) whereby the Philippine Government undertook to return to the
United States Government in ten annual installments, a total of about 35-million
dollars advanced by the United States to, but unexpanded by, the National Defense
Forces of the Philippines.
• However, there was lack of ratification of the Agreement by the Senate of the
Philippines.
Issue: Whether or not the agreement was constitutional.

Ruling: The Agreement was constitutional.


• The agreement is not a "treaty" as that term is used in the Constitution, is conceded.
The agreement was never submitted to the Senate for concurrence (Art. VII, Sec. 10
(7). However, it must be noted that treaty is not the only form that an international
agreement may assume. For the grant of the treaty-making power to the Executive
and the Senate does not exhaust the power of the government over international
relations. Consequently, executive agreements may be entered with other states and
are effective even without the concurrence of the Senate.
• It is observed in this connection that from the point of view of the international law,
there is no difference between treaties and executive agreements in their binding
effect upon states concerned as long as the negotiating functionaries have remained
within their powers.
• Executive Agreements fall into two classes: (1) agreements made purely as
executive acts affecting external relations and independent of or without legislative
authorization, which may be termed as presidential agreements and (2) agreements
entered into in pursuant of acts of Congress, which have been designated as
Congressional-Executive Agreements.
• The Romulo-Snyder Agreement may fall under any of these two classes, for
precisely on September 18, 1946, Congress of the Philippines specifically authorized
the President of the Philippines to obtain such loans or incur such indebtedness
with the Government of the United States, its agencies or instrumentalities
(Republic Act No. 16, September 18, 1946, amended by Republic Act No. 213, June 1,
1948)
• Even granting, arguendo, that there was no legislative authorization, it is hereby
maintained that the Romulo-Snyder Agreement was legally and validly entered into
to conform to the second category, namely, "agreements entered into purely as
executive acts without legislative authorization." This second category usually
includes money agreements relating to the settlement of pecuniary claims of
citizens.
• Indeed, the Agreement was not submitted to the U.S. Senate either; but we do not
stop to check the authorities above listed nor test the conclusions derived therefrom
in order to render a definite pronouncement, for the reason that our Senate
Resolution No. 153 practically admits the validity and binding force of such
Agreement. Furthermore, the acts of Congress Appropriating funds for the yearly
installments necessary to comply with such Agreements constitute a ratification
thereof, which places the question the validity out of the Court's reach, no
constitutional principle having been invoked to restrict Congress' plenary power to
appropriate funds-loan or no loan.

COMMISSIONER OF INTERNAL REVENUE V. GOTAMCO


Facts:
• A Host Agreement entered into between the Republic of the Philippines and the
World Health Organization (WHO) on July 22, 1951. Section 11 of that Agreement
provides, inter alia, that "the Organization, its assets, income and other properties
shall be: (a) exempt from all direct and indirect taxes. It is understood, however,
that the Organization will not claim exemption from taxes which are, in fact, no
more than charges for public utility services; . . .”
• The WHO decided to construct a building to house its own offices, as well as the
other United Nations offices stationed in Manila. The construction contract was
awarded to respondent John Gotamco& Sons, Inc. (Gotamco).
• WHO received an opinion from the Commissioner of the Bureau of Internal Revenue
stating that "as the 3% contractor's tax is an indirect tax on the assets and income of
the Organization, the gross receipts derived by contractors from their contracts with
the WHO for the construction of its new building, are exempt from tax in accordance
with . . . the Host Agreement." Subsequently, however, on June 3, 1958, the
Commissioner of Internal Revenue reversed his opinion and stated that "as the 3%
contractor's tax is not a direct nor an indirect tax on the WHO, but a tax that is
primarily due from the contractor, the same is not covered by . . . the Host
Agreement."
• Respondent Gotamco appealed the Commissioner's decision to the Court of Tax
Appeals, which after trial rendered a decision, in favor of Gotamco and reversed the
Commissioner's decision. The Court of Tax Appeal's decision is now before us for
review on certiorari.
• Commissioner questions the entitlement of the WHO to tax exemption, contending
that the Host Agreement is null and void, not having been ratified by the Philippine
Senate as required by the Constitution.
• Commissioner maintains that even assuming that the Host Agreement granting tax
exemption to the WHO is valid and enforceable, the 3% contractor's tax assessed on
Gotamco is not an "indirect tax" within its purview. Petitioner's position is that the
contractor's tax "is in the nature of an excise tax which is a charge imposed upon the
performance of an act, the enjoyment of a privilege or the engaging in an occupation.
. . It is a tax due primarily and directly on the contractor, not on the owner of the
building. Since this tax has no bearing upon the WHO, it cannot be deemed an
indirect taxation upon it."

Issue: Whether the WHO is entitled to tax exemption by virtue of the Host Agreement not
ratified by the Philippine Senate.

Ruling: WHO is entitled to tax exemption.


• While treaties are required to be ratified by the Senate under the Constitution, less
formal types of international agreements may be entered into by the Chief Executive
and become binding without the concurrence of the legislative body. The Host
Agreement comes within the latter category; it is a valid and binding international
agreement even without the concurrence of the Philippine Senate.
• The Host Agreement, in specifically exempting the WHO from "indirect taxes,"
contemplates taxes which, although not imposed upon or paid by the Organization
directly, form part of the price paid or to be paid by it.
• (TANGENT ONLY) As to the payment of the 3% contractor’s tax, the Supreme Court
upheld the decision of the Court of Tax Appeals:
In context, direct taxes are those that are demanded from the very
person who, it is intended or desired, should pay them; while indirect
taxes are those that are demanded in the first instance from one
person in the expectation and intention that he can shift the burden to
someone else. (Pollock vs. Farmers, L & T Co., 1957 US 429, 15 S. Ct.
673, 39 Law.Ed. 759.) The contractor's tax is of course payable by the
contractor but in the last analysis it is the owner of the building that
shoulders the burden of the tax because the same is shifted by the
contractor to the owner as a matter of self-preservation. Thus, it is an
indirect tax. And it is an indirect tax on the WHO because, although it
is payable by the petitioner, the latter can shift its burden on the
WHO. In the last analysis it is the WHO that will pay the tax indirectly
through the contractor and it certainly cannot be said that 'this tax has
no bearing upon the World Health Organization’.

BAYAN V. EXECUTIVE SECRETARY


• Confronting the Court for resolution in the instant consolidated petitions for
certiorari and prohibition are issues relating to, and borne by, an agreement forged
in the turn of the last century between the Republic of the Philippines and the
United States of America - the Visiting Forces Agreement.
• On October 5, 1998, President Joseph E. Estrada, through respondent Secretary of
Foreign Affairs, ratified the VFA.
• On May 27, 1999, Proposed Senate Resolution No. 443 was approved by the Senate,
by a two-thirds (2/3) vote of its members. Senate Resolution No. 443 was then re-
numbered as Senate Resolution No. 18.

Issues:
A. Whether or not the VFA is constitutional;
B. Is the VFA governed by the provisions of Section 21, Article VII or of Section 25,
Article XVIII of the Constitution?

Ruling: A. VFA is constitutional;


• By constitutional fiat and by the intrinsic nature of his office, the President, as head
of State, is the sole organ and authority in the external affairs of the country. In
many ways, the President is the chief architect of the nations foreign policy; his
dominance in the field of foreign relations is (then) conceded. Wielding vast powers
an influence, his conduct in the external affairs of the nation, as Jefferson describes,
is executive altogether."
• As regards the power to enter into treaties or international agreements, the
Constitution vests the same in the President, subject only to the concurrence of at
least two-thirds vote of all the members of the Senate. In this light, the negotiation of
the VFA and the subsequent ratification of the agreement are exclusive acts which
pertain solely to the President, in the lawful exercise of his vast executive and
diplomatic powers granted him no less than by the fundamental law itself. Into the
field of negotiation the Senate cannot intrude, and Congress itself is powerless to
invade it.[53] Consequently, the acts or judgment calls of the President involving the
VFA-specifically the acts of ratification and entering into a treaty and those
necessary or incidental to the exercise of such principal acts - squarely fall within
the sphere of his constitutional powers and thus, may not be validly struck down,
much less calibrated by this Court, in the absence of clear showing of grave abuse of
power or discretion.
• It is the Courts considered view that the President, in ratifying the VFA and in
submitting the same to the Senate for concurrence, acted within the confines and
limits of the powers vested in him by the Constitution. It is of no moment that the
President, in the exercise of his wide latitude of discretion and in the honest belief
that the VFA falls within the ambit of Section 21, Article VII of the Constitution,
referred the VFA to the Senate for concurrence under the aforementioned provision.
Certainly, no abuse of discretion, much less a grave, patent and whimsical abuse of
judgment, may be imputed to the President in his act of ratifying the VFA and
referring the same to the Senate for the purpose of complying with the concurrence
requirement embodied in the fundamental law. In doing so, the President merely
performed a constitutional task and exercised a prerogative that chiefly pertains to
the functions of his office. Even if he erred in submitting the VFA to the Senate for
concurrence under the provisions of Section 21 of Article VII, instead of Section 25
of Article XVIII of the Constitution, still, the President may not be faulted or scarred,
much less be adjudged guilty of committing an abuse of discretion in some patent,
gross, and capricious manner.
B. (TANGENT) The VFA is governed by both aforecited provisions.
• Section 21, Article VII v. Section 25, Article XVIII
Section 21, Article VII deals with treatise or international agreements in
general, in which case, the concurrence of at least two-thirds (2/3) of all the
Members of the Senate is required to make the subject treaty, or
international agreement, valid and binding on the part of the
Philippines. This provision lays down the general rule on treatise or
international agreements and applies to any form of treaty with a wide
variety of subject matter, such as, but not limited to, extradition or tax
treatise or those economic in nature. All treaties or international agreements
entered into by the Philippines, regardless of subject matter, coverage, or
particular designation or appellation, requires the concurrence of the Senate
to be valid and effective.
In contrast, Section 25, Article XVIII is a special provision that applies to
treaties which involve the presence of foreign military bases, troops or
facilities in the Philippines. Under this provision, the concurrence of the
Senate is only one of the requisites to render compliance with the
constitutional requirements and to consider the agreement binding on the
Philippines. Section 25, Article XVIII further requires that foreign military
bases, troops, or facilities may be allowed in the Philippines only by virtue of
a treaty duly concurred in by the Senate, ratified by a majority of the votes
cast in a national referendum held for that purpose if so required by
Congress, and recognized as such by the other contracting state.
• Section 25, Article XVIII disallows foreign military bases, troops, or facilities in the
country, unless the following conditions are sufficiently met, viz: (a) it must be
under a treaty; (b) the treaty must be duly concurred in by the Senate and, when so
required by congress, ratified by a majority of the votes cast by the people in a
national referendum; and (c) recognized as a treaty by the other contracting state.
• There is no dispute as to the presence of the first two requisites in the case of the
VFA. The concurrence handed by the Senate through Resolution No. 18 is in
accordance with the provisions of the Constitution . . . the provision in [in §25,
Article XVIII] requiring ratification by a majority of the votes cast in a national
referendum being unnecessary since Congress has not required it.
• This Court is of the firm view that the phrase “recognized as a treaty” means that
the other contracting party accepts or acknowledges the agreement as a treaty. To
require the other contracting state, the United States of America in this case, to
submit the VFA to the United States Senate for concurrence pursuant to its
Constitution, is to accord strict meaning to the phrase.
• Moreover, it is inconsequential whether the United States treats the VFA only as an
executive agreement because, under international law, an executive agreement is as
binding as a treaty. To be sure, as long as the VFA possesses the elements of an
agreement under international law, the said agreement is to be taken equally as a
treaty.

BAYAN MUNA V. ROMULO


Facts:
• Petitioner seeks to nullify the Non-Surrender Agreement concluded by and between
the Republic of the Philippines (RP) and the United States of America (USA).
• In esse, the Agreement aims to protect what it refers to and defines as persons of the
RP and US from frivolous and harassment suits that might be brought against them
in international tribunals.It is reflective of the increasing pace of the strategic
security and defense partnership between the two countries.
• Petitioner imputes grave abuse of discretion to respondents in concluding and
ratifying the Agreement and prays that it be struck down as unconstitutional, or at
least declared as without force and effect.
• Respondent countered that the Agreement, being in the nature of an executive
agreement, does not require Senate concurrence for its efficacy. And for reasons
detailed in their comment, respondents assert the constitutionality of the
Agreement.

Issue: Whether or not the Agreement is valid, binding and effective without the
concurrence by at least two-thirds (2/3) of all the members of the senate.

Ruling:The Agreement is valid; Senate concurrence not needed.


• Article 2 of the Vienna Convention on the Law of Treaties defines a treaty as an
international agreement concluded between states in written form and governed by
international law, whether embodied in a single instrument or in two or more
related instruments and whatever its particular designation.International
agreements may be in the form of (1) treaties that require legislative concurrence
after executive ratification; or (2) executive agreements that are similar to treaties,
except that they do not require legislative concurrence and are usually less formal
and deal with a narrower range of subject matters than treaties.
• Under international law, there is no difference between treaties and executive
agreements in terms of their binding effects on the contracting states concerned, as
long as the negotiating functionaries have remained within their powers. Neither, on
the domestic sphere, can one be held valid if it violates the Constitution.
• Authorities are, however, agreed that one is distinct from another for accepted
reasons apart from the concurrence-requirement aspect. As has been observed by
US constitutional scholars, a treaty has greater dignity than an executive agreement,
because its constitutional efficacy is beyond doubt, a treaty having behind it the
authority of the President, the Senate, and the people; a ratified treaty, unlike an
executive agreement, takes precedence over any prior statutory enactment.
• The Court has, in Eastern Sea Trading,as reiterated in Bayan,given recognition to the
obligatory effect of executive agreements without the concurrence of the Senate:
x xx [T]he right of the Executive to enter into binding agreements
without the necessity of subsequent Congressional approval has been
confirmed by long usage. From the earliest days of our history, we
have entered executive agreements covering such subjects as
commercial and consular relations, most favored-nation rights, patent
rights, trademark and copyright protection, postal and navigation
arrangements and the settlement of claims. The validity of these has
never been seriously questioned by our courts.

3. DEPORTATION POWERS

QUA CHEE GAN V. DEPORTATION


Facts:
• Special Prosecutor Emilio L. Galang charged the Qua CheeGan, James Uy, Daniel
Dy alias Dee Pac, Chan Tiong Yu, Chua Chu Tian, Chua Lim Pao alias Jose Chua, and
Basilio King before the Deportation Board, with having purchased U.S. dollars in the
total sum of $130,000.00, without the necessary license from the Central Bank of the
Philippines, and of having clandestinely remitted the same to Hongkong and
petitioners, Qua CheeGan, Chua Lim Pao alias Jose Chua, and Basilio King, with
having attempted to bribe officers of the Philippine and United States Governments
(Antonio Laforteza, Chief of the Intelligence Division of the Central Bank, and Capt.
A. P. Charak of the OSI, U.S. Air Force) in order to evade prosecution for said
unauthorized purchase of U.S. dollars.
• Awarrant for the arrest of said aliens was issued by the presiding member of the
Deportation Board. Upon their filing surety bond petitioners-appellants were
provisionally set at liberty.
• Petitioners-appellants filed a joint motion to dismiss the charges presented against
them in the Deportation Board for the reason, among others, that the same do not
constitute legal ground for deportation of aliens from this country, and that said
Board has no jurisdiction to entertain such charges.
• Respondent Board filed its answer to the original petition, maintaining among
others, that the Deportation Board, as an agent of the President, has jurisdiction
over the charges filed against petitioners and the authority to order their arrest.
• Petitioners-appellants contest the power of the President to deport aliens and,
consequently, the delegation to the Deportation Board of the ancillary power to
investigate, on the ground that such power is vested in the Legislature. In other
words, it is claimed, for the power to deport to be exercised, there must be a
legislation authorizing the same.

Issue: Whether the warrant of arrest was valid.

Held:The warrant of arrest was void.


• Under the present and existing laws, deportation of an undesirable alien may be
effected in two ways: by order of the President, after due investigation, pursuant to
Section 69 of the Revised Administrative Code, and by the Commissioner of
Immigration, upon recommendation by the Board of Commissioners, under Section
37 of Commonwealth Act No. 613.
• There seems to be no doubt that the President's power of investigation may be
delegated. This is clear from a reading of Section 69 of the Revised Administrative
Code which provides for a "prior investigation, conducted by said Executive (the
President) or his authorized agent."
• By virtue of Executive Order No. 33 dated May 29, 1936, President Quezon created
the Deportation Board primarily to receive complaints against aliens charged to be
undesirable, to conduct investigation pursuant to Section 69 of the Revised
Administrative Code and the rules and regulations therein provided, and make the
corresponding recommendation. Since then, the Deportation Board has been
conducting the investigation as the authorized agent of the President.
• This gives rise to the question regarding the extent of the power of the President to
conduct investigation, i.e., whether such authority carries with it the power to order
the arrest of the alien complained of, since the Administrative Code is silent on the
matter, and if it does, whether the same may be delegated to the respondent
Deportation Board.
• Let it be noted that Section 69 of the Revised Administrative Code, unlike
Commonwealth Act No. 613 wherein the Commissioner of Immigration was
specifically granted authority, among others, to make arrests, fails to provide the
President with like specific power to be exercised in connection with such
investigation.
• Note that the executive order only required the filing of a bond to secure appearance
of the alien under investigation. It did not authorize the arrest of the respondent. It
must be remembered that the right of an individual to be secure in his person is
guaranteed by the Constitution in the following language:.
3. The right of the People to be secure in their persons, houses, papers
and effects against unreasonable searches and seizures shall not be
violated, and no warrants shall issue but upon probable cause, to be
determined by the judge after examination under oath or affirmation
of the complainant and the witnesses he may produce, and
particularly describing the place to be searched, and the persons or
things to be seized." (Sec 1, Art. III, Bill of Rights, Philippine
Constitution).
• Unquestionably, the exercise of the power to order the arrest of an individual
demands the exercise of discretion by the one issuing the same, to determine
whether under specific circumstances, the curtailment of the liberty of such
person is warranted. The fact that the Constitution itself, as well as the statute
relied upon, prescribe the manner by which the warrant may be issued, conveys the
intent to make the issuance of such warrant dependent upon conditions the
determination of the existence of which requires the use of discretion by the person
issuing the same. In other words, the discretion of whether a warrant of arrest shall
issue or not is personal to the one upon whom the authority devolves. And
authorities are to the effect that while ministerial duties may be delegated, official
functions requiring the exercise of discretion and judgment, may not be so
delegated. Indeed, an implied grant of power, considering that no express authority
was granted by the law on the matter under discussion, that would serve the
curtailment or limitation on the fundamental right of a person, such as his security
to life and liberty, must be viewed with caution, if we are to give meaning to the
guarantee contained in the Constitution. If this is so, then guarantee a delegation of
that implied power, nebulous as it is, must be rejected as inimical to the liberty of
the people. The guarantees of human rights and freedom can not be made to rest
precariously on such a shaky foundation.
• Executive Order No. 398, series of 1951, insofar as it empowers the Deportation
Board to issue warrant of arrest upon the filing of formal charges against an alien or
aliens and to fix bond and prescribe the conditions for the temporary release of said
aliens, is declared illegal. As a consequence, the order of arrest issued by the
respondent Deportation Board is declared null and void and the bonds filed
pursuant to such order of arrest, decreed cancelled.
• In other words, the provision of Executive Order No. 398, of 1951, the
Deportation Board to issue a warrant of arrest upon the filing of formal
charges against an alien, is "illegal" or unconstitutional because it is contrary
to the provision in Section 1(3), Article III of the 1935 Constitution that
warrants shall be issued by the judge after examination under oath or
affirmation of the complainant and the witnesses he may produce, and
particularly describing the place to be searched, and the persons or things to
be seized.

GO TEK V. DEPORTATION
Facts:
• The chief prosecutor of the Deportation Board filed a complaint against Go Tek a
Chinaman residing at Ilagan, Isabela and 1208-B, Misericordia Street, Sta. Cruz
Manila.
• It was alleged in the complaint that in December, 1963 certain agents of the National
Bureau of Investigation (NBI) searched an office located at 1439 O'Donnel Street,
Sta. Cruz, Manila believed to be the headquarters of a guerilla unit of the
"Emergency Intelligence Section, Army of the United States" and that among those
arrested thereat was Go Tek an alleged sector commander and intelligence and
record officer of that guerilla unit.
• It was further alleged that fake dollar checks were found in Go Tek's possession and
that, therefore, he had violated article 168 of the Revised Penal Code and rendered
himself an undesirable alien.
• The prosecutor prayed that after trial the Board should recommend to the President
of the Philippines the immediate deportation of Go Tek as an undesirable alien, "his
presence in this country having been, and will always be and a menace to the peace ,
welfare, and security of the community".
• Go Tek filed a motion to dismiss on the ground that the complaint was premature
bemuse them was a pending case against him in the city fiscal's office of Manila for
violation of article 168 (IS 64-7267).
• The Board denied Go Teks motion on the ground that a conviction is not a
prerequisite before the State my exercise its rights to deport an undesirable alien
and that the Board is only a fact finding body whose function is to make a report and
recommendation to the President in whom is lodged the exclusive power to deport
an alien or a deportation proceeding.

Issue: Whether or not the Board has acquired jurisdiction in spite of the fact that he has
not yet been convicted of illegal possession thereof under article 168 of the Revised Penal
Code and notwithstanding that act is not the grounds for the deportation of undesirable
aliens as enumerated in section 37 of the Immigration Law.

Ruling:The Board has acquired jurisdiction.


• A thorough comprehension of the President's power to deport aliens may show the
baselessness of the instant prohibition action of Go Tek The President's power to
deport aliens and the investigation of aliens subject to deportation are provided for
in the following provisions of the Revised Administrative Code:
SEC. 69.Deportation of subject of foreign power. — A subject of a
foreign power residing in the Philippine Islands shall not be deported
expelled, or excluded from said Islands or repatriated to his own
country by the Governor-General except upon prior investigator,
conducted by said Executive or his authorized agent, of the ground
upon which such action is contemplated. In such case the person
concerned shall he informed of the charge or charges against him and
he shall be allowed not less than three days for the preparation of his
defense. He shall also have the right to be heard by himself or counsel,
to produce witnesses in his own behalf, and to cross-examine the
opposing witnesses.
• On the other hand, Section 37 of the Immigration Law Provides that certain aliens
may be arrested upon the warrant of the Commissioner of Immigration or of any
other officer designated by him for the purpose and deported upon the
Commissioner's warrant - "after a determination by the Board of Commissioners of
the existence of the ground for deportation as charged against the alien."
• The State has the inherent power to deport undesirable aliens. That power may be
exercised by the Chief Executive "when he deems such action necessary for the
peace and domestic tranquility of the nation.”
• Section 69 and Executive Order No. 398, the Deportation Board, do not specify the
grounds for deportation. Paragraph l(a) of Executive Order No. 398 merely provides
that "the Deportation Board, motuproprio or upon complaint of any person is
authorized to conduct investigations in the manner prescribed in section 69 of the
Revised Administrative Code to determine whether a subject of a foreign power in
the Philippines is an undesirable alien or not, and thereafter to recommend to the
President of the Philippines the deportation of such alien."

• "It is fundamental that an executive order for deportation is not dependent on a


prior judicial conviction in a case" Thus, it was held that the fact that an alien has
been acquitted in the charge does not prevent the deportation of such alien based on
the same charge. Such acquittal does not constitute res judicata in the deportation
proceedings. Conviction of a crime is not a requisite to warrant deportation.

DOMINGO V. SCHEER
Facts:
• Respondent Herbert Markus Emil Scheer, a native of Ochsenfurt, Germany, was a
frequent visitor of the Philippines. On July 18, 1986, his application for permanent
resident status was granted.The Bureau of Immigration and Deportation (BID)
issued in favor of the respondent Alien Certificate of Registration No. B-396907
dated September 16, 1987and Immigration Certificate of Residence No. 256789
dated February 24, 1988.
• The Department of Foreign Affairs received from the German Embassy in Manila
Note Verbale No. 369/95 dated July 26, 1995, informing it that the respondent was
wanted by the German Federal Police; that a warrant of arrest had been issued
against him; and that the respondent will be served with an official document
requesting him to turn over his German passport to the Embassy which was
invalidated on July 2, 1995. The Embassy requested the Department of Foreign
Affairs to inform the competent Philippine authorities of the matter.
• The (Board of Commissioners) BOC thereafter issued a Summary Deportation Order
dated September 27, 1997.
• When the respondent was apprised of the deportation order, he forthwith aired his
side to then BID Commissioner Leandro T. Verceles. The Commissioner allowed the
respondent to remain in the Philippines, giving the latter time to secure a clearance
and a new passport from the German Embassy.
• The respondent, through counsel, filed on December 5, 1995 an Urgent Motion for
Reconsideration of the Summary Deportation Order of the BOC.
• On February 15, 1996, the District Court of Straubing rendered a Decision
dismissing the criminal case against the respondent for physical injuries.The
German Embassy in Manila, thereafter, issued a temporary passport to the
respondent.
• When petitioner Immigration Commissioner Andrea T. Domingo assumed office, she
wrote the German Embassy and inquired if the respondent was wanted by the
German police. On April 12, 2002, the German Embassy replied that the respondent
was not so wanted.
• At about midnight on June 6, 2002, Marine operatives and BID agents apprehended
the respondent in his residence on orders of the petitioner. He was whisked to the
BID Manila Office and there held in custody while awaiting his deportation. Despite
entreaties from the respondent’s wifeand his employees, the petitioner refused to
release the respondent.
• The respondent (petitioner therein) alleged, inter alia, that his arrest and detention
were premature, unjust, wrongful, illegal and unconstitutional, effected without
sufficient cause and without jurisdiction or with grave abuse of discretion. He
asserted that there was no speedy remedy open to him in the ordinary course of
lawand that his Urgent Motion for Reconsideration of the Summary Deportation
Order of the BOC had not yet been resolved despite the lapse of more than six years.
The respondent averred that he was a fully documented alien, a permanent resident
and a law-abiding citizen.

Issue: Whether the arrest of Scheer was valid.

Ruling.The arrest was invalid.


• The BOC committed a graveabuse of discretion amountingto lack or excess of
jurisdictionin issuing its summary deportationorder and omnibus resolution;
thepetitioner committed a grave abuseof her discretion amounting tolack or excess
of jurisdiction incausing the arrest and detentionof the private respondent.
• The settled rule is that the entry or stay of aliens in the Philippines is merely a
privilege and a matter of grace; such privilege is not absolute nor permanent and
may be revoked. However, aliens may be expelled or deported from the Philippines
only on grounds and in the manner provided for by the Constitution, the
Immigration Act of 1940, as amended, and administrative issuances pursuant
thereto.
• In this case, the BOC ordered the private respondents deportation on September 27,
1995 without even conducting summary deportation proceedings. The BOC merely
relied on the June 29, 1995 Letter of the German Vice Consul and of the German
Embassy.
• The BOC issued its Summary Deportation Order without affording the respondent
the right to be heard on his motion and adduce evidence thereon. It merely
concluded that the respondent was involved in illegal activities in Palawan. What
made matters worse was that the BOC indulged in sheer speculation, that the
German Embassy is unlikely to issue a new passport to the respondent.
• The deportation of aliens should not be based on mere speculation or a mere
product of procrastinations as in this case. As it turned out, the German Embassy re-
issued the respondents passport; he was issued a temporary passport, and,
thereafter, a regular passport, yet to expire on March 12, 2006. The petitioner
cannot feign ignorance of this matter because the respondent himself, six years
before he was arrested, informed then Immigration Commissioner Verceles in a
Letter dated March 1, 1996. The respondents letter forms part of the records of the
BOC. There is no evidence on record that the respondent committed any illegal
activities in Palawan. He was even designated as special agent of the NBI, and was,
in fact, issued clearances by the PNP and the NBI no less. Despite all the foregoing,
the petitioner ordered and caused the arrest and detention of the respondent.
• The deportation of an alien is not intended as a punishment or penalty. But in a real
sense, it is. In Bridges v. Wixon,Mr. Justice Murphy declared that the impact of
deportation upon the life of an alien is often as great if not greater than the
imposition of a criminal sentence. In dealing with deportation, there is no justifiable
reason for disregarding the democratic and human tenets of our legal system and
descending to the practices of despotism.
• As Justice Brewer opined in Fong Yue Ting v. United States,deportation is a
punishment because it requires first, an arrest, a deprivation of liberty and second, a
removal from home, from family, from business, from property. To be forcibly taken
away from home, family, business and property and sent across the ocean to a
distant land is punishment; and that oftentimes is most severe and cruel. It would be
putting salt on the respondents woes occasioned by the BOCs
ineptitude. Considering the peculiar backdrop and the equities in this case, the
respondents deportation and the cancellation of his permanent resident visa as a
precondition to his re-entry into this country is severe and cruel; it is a form of
punishment.
SECRETARY OF JUSTICE V. KORUGA
Facts:
• Then Bureau of Immigration (BI) Commissioner Andrea Domingo received an
anonymous letterrequesting the deportation of respondent as an undesirable alien
for having been found guilty of Violation of the Uniform Controlled Substances Act
in the State of Washington, United States of America (USA) for attempted possession
of cocaine sometime in 1983.
• On the basis of a Summary of Information,the Commissioner issued Mission Order
No. ADD-01-162on September 13, 2001 directing Police Superintendent, Chief of the
Intelligence Mission and any available BI Special Operations Team Member to
conduct verification/ validation of the admission status and activities of respondent
and effect his immediate arrest if he is found to have violated the Philippine
Immigration Act of 1940, as amended.
• On September 17, 2001, respondent was arrested and charged before the Board of
Special Inquiry (BSI) for violation of Section 37(a)(4) of the Philippine Immigration
Act of 1940, as amended
• Subsequently, respondent was convicted and/or sentenced for Uniform Controlled
Substance Act in connection with his being Drug Trafficker and/or Courier of
prohibited drugs in the State of Washington, United States of America, thus, making
him an undesirable alien and/or a public burden in violation of Sec. 37(a)(4) of the
Philippine Immigration Act of 1940, as amended.
• The CA rendered a Decision20 setting aside the Resolution dated April 1, 2003 and
held, inter alia, that even if respondent was convicted and sentenced for the alleged
offense, his deportation under Section 37(a)(4) is improper, since the prohibited
drugs law referred to therein refers not to a foreign drugs law but to the Philippine
drugs law, then Republic Act No. 6425 or the "Dangerous Drugs Act of 1972".
• Petitioners contend that there is a valid basis to declare respondent's undesirability
and effect his deportation since respondent has admitted guilt of his involvement in
a drug-related case.
• Respondent submits that even assuming that Section 37(a)(4) of the Philippine
Immigration Act of 1940 does not apply, there is no reason, whether compelling or
slight, to deport respondent.

Issues: Whether or not there is a valid and legal ground for the deportation of respondent.

Ruling:There is a valid and legal ground for the deportation of respondent.


• Respondent was charged with violation of Section 37(a)(4) of the Philippine
Immigration Act of 1940, as amended, which provides:
Sec. 37. (a) The following aliens shall be arrested upon the warrant of
the Commissioner of Immigration or of any other officer designated
by him for the purpose and deported upon the warrant of the
Commissioner of Immigration after a determination by the Board of
Commissioners of the existence of the ground for deportation as
charged against the alien.
x xxx
(4) Any alien who is convicted and sentenced for a violation of the law
governing prohibited drugs;
x xxx (Emphasis supplied)
• Were the Court to follow the letter of Section 37(a)(4) and make it applicable only to
convictions under the Philippine prohibited drugs law, the Court will in effect be
paving the way to an absurd situation whereby aliens convicted of foreign
prohibited drugs laws may be allowed to enter the country to the detriment of the
public health and safety of its citizens. It suggests a double standard of treatment
where only aliens convicted of Philippine prohibited drugs law would be deported,
while aliens convicted of foreign prohibited drugs laws would be allowed entry in
the country.
• Moreover, since Section 37(a)(4) makes no distinction between a foreign prohibited
drugs law and the Philippine prohibited drugs law, neither should this Court.
• There is no dispute that respondent was convicted of Violation of the Uniform
Controlled Substances Act in the State of Washington, USA for attempted possession
of cocaine, as shown by the Order Deferring Imposition of Sentence
(Probation).While he may have pleaded guilty to a lesser offense, and was not
imprisoned but applied for and underwent a one-year probation, still, there is no
escaping the fact that he was convicted under a prohibited drugs law, even though it
may simply be called a "misdemeanor drug offense.” The BOC did not commit grave
abuse of discretion in ordering the deportation of respondent.
• Every sovereign power has the inherent power to exclude aliens from its territory
upon such grounds as it may deem proper for its self-preservation or public
interest.The power to deport aliens is an act of State, an act done by or under the
authority of the sovereign power.It is a police measure against undesirable aliens
whose continued presence in the country is found to be injurious to the public good
and the domestic tranquility of the people.

***CRUZ Notes
Powers of the President
Ordinance Powers

1) Executive Orders: acts of the Presdient providing for rules of a general or permanent
character in implementation or execution of constitutional or statutory powers
2) Administrative Orders- acts of the President which relate to particular aspects of
governmental operations in pursuance of his duties as administrative head
3) Proclamations- status or condition of public moment or interest, upon the existence of
which the operation of a specific law or regulation is made to depend
ART. VIII
JUDICIAL DEPARTMENT
FISCAL AUTONOMY
SECTION 3
The Judiciary shall enjoy fiscal autonomy. Appropriations for the Judiciary may not
be reduced by the legislature below the amount appropriated for the previous year
and, after approval, shall be automatically and regularly released

CSC vs. DBM


2005
Facts:
CSC files a petition for Mandamus to compel the DBM to release the rest of their allotted
budget for the fiscal year of 2002. Petitioner alleges that DBM is withholding the balance of
Php 5,807,392.30 because of their “no report, no release” policy. CSC claims that the
application of the policy violates the principle of fiscal autonomy and therefore,
unconstitutional.
Respondent denies having enforced the policy strictly upon offices that are vested with
fiscal autonomy. They said that the withholding was because of shortage of revenue.
Issue: W/N the application of the “no report, no release” policy on the offices vested with
fiscal autonomy is unconstitutional
Ruling:
Respondent’s act of withholding the subject funds from petitioner due to revenue
shortfall is UNCONSTITUTIONAL.
1. “No report, no release policy” cannot be enforced against offices possessing fiscal
autonomy
- It violates Art. IX (A), Sec. 5 of the Constitution: “The Commission shall enjoy fiscal
autonomy. Their approved appropriations shall be automatically and regularly
released.
- “Automatic release” (explained in Province of Batangas vs. Romulo) connotes
something mechanical, spontaneous and perfunctory. As such LGU’s are not required
to perform any act to receive the “just share” accruing to them from the national
coffers.
- statutory construction: no condition to fund releases to it may be imposed
- A.M. No 92-9-029-SC, Constitutional Mandate on Judiciary’s Fiscal Autonomy -
prohibited the “no report, no release policy against the Judiciary which has also been
granted fiscal autonomy by the Constitution
2. Shortfall in revenues was not substantiated, even if there was, it does not justify non-
compliance with the mandate with Art. IX (A), Sec. 5, of the Constitution. An adoption of
the respondent’s defense:
(1) would make the automatic release mandate by the constitution insignificant
(2) place petitioner and other entities entitled to fixcal autonomy (Judiciary and the
ConComs) on equal footing with those who are not granted fiscal autonomy.
➡ The agencies which the Constitution has vested with fiscal autonomy should thus be
given priority in the release of their approved appropriations over all other agencies
not similarly vested when there is a revenue shortfall
3. Such policy violates Sec. 64 of the GAA of Year 2002 which excepted the ConComs and the
Judiciary from the reduction and retention of appropriations because of budget deficits.
- even if there was a budget deficit, the DBM would not be allowed to withhold the
allotted budget from the offices who have fiscal autonomy
4. A.M. No 92-9-029-SC states “after approval by Congress, the appropriations for the
Judiciary shall be automatically…. subject to availability of funds”
- does not contradict the fiscal autonomy
- situation contemplated is where total revenue collections are so low that they are not
sufficient to cover the total appropriations for all entities vested with fiscal autonomy.
<- only in this situation can there be a relaxation of constituional mandate to release
automatically and regularly the appropriations
- only a small portion of the total annual budget is allotted to the ConComs and the
judiciary (around 2%)
5. Claim of petitioner that its budget may not be reduced by Congress lower than that of
the previous fiscal year, as is the case of the Judiciary must be rejected
(1) Sec. 3, Art. VIII - The judiciary shall enjoy fiscal autonomy. Appropriations for the
Judiciary may not be reduced by the legislature below the amount appropriated for
the previous year and, after approval, shall be automatically and regularly released
(2) The explicit prohibition of the reduction of the budget from the previous year is
absent in Art. IX (A), Sec. 5 of the Constitution - “The Commission shall enjoy fiscal
autonomy. Their approved annual appropriations shall be automatically and
regularly released.
➡ omission implies that Congress is not prohibited from reducing the appropriations of
Constitutional Commissions below the amount appropriated for them in the
previous year.

Re: Clarifying and Strengthening the Organizational Structure and Administrative


Set-Up of the Philippine Judicial Academy
January 31, 2006
Facts:
Philippine Judicial Academy pursuant to the resolution promulgated on Feb. 24, 2004,
it created the ff positions with their salary grades for divisions of PhilJA:
(1) Chief Judiciary Staff Officer : salary grade 25
(2) Supervising Judicial Staff Officer: salary grade 23
DBM downgraded said positions and the salary grades to 24 and 22, respectively. The
Court issued a Resolution on July 5, 2005 retaining the original salary grades. Chancellor
Melencio-Herrera sent a memorandum addressed to CJ Hilario Davide requesting for an
upgrade of the positions of CJ Staff Head, and others. In the memorandum, she said that to
allow the DBM to disregard such resolution would “undermine the independence of the
Judiciary and impinge on the Supreme Court’s exercise of its fiscal autonomy expressly
granted by the Constitution. CJ Davide denied the request, held the retention of the original
salary grades. Atty. Diño from the Office of Chief Attorney, recommended that the SC
reiterate their July 5 Resolution.
Issue: W/N DBM violated the Constitutional mandate of fiscal autonomy by downgrading
the salary grades of the said positions.
Ruling:
The DBM cannot modify or amend a Resolution of the SC. It is a violation of the
principle of separation of powers.
What is the DBM’s mandate? It is authorized to evaluate and determine whether a
proposed reclassification and upgrading scheme is consistent with applicable laws and
regulations; simply to review the compensation and benefits plan of the government
agency or entity concerned and determine if it complies with the prescribed policies and
guidelines issued in this regard.

The authority of the DBM is supervisorial in nature, its main duty being to ascertain that
the proposed compensation, benefits and other incentives to be given to government
officials and employees adhere to the policies and guidelines issues in accordance with
applicable laws.

The authority of DBM to review Supreme issuances relative to court personnel on matters
of compensation is even more limited, circumscribed as it is by the provisions of the
Constitution, Art. VIII, Sec. 3 and 6.

What is Fiscal Autonomy? It means freedom from outside control.


Bengzon vs. Drilon: Fiscal autonomy…
(1) contemplates a guarantee of full flexibility to allocate and utilize their resources with
the wisdom and dispatch that their needs require
(2) recognizes the power and authority to levy, assess and collect fees, fix rates of
compensation not exceeding the highest rates authorized by law for compensation and
pay plans of the government and allocate and disburse such sums as may be provided by
law or prescribed by them in the course of the discharge of their functions
(3) means freedom from outside control = if Judiciary needs 100 typewriters, but DBM
rules that we need only 10, and sends its recommendations to Congress without even
informing the SC, the autonomy given by the Constitution becomes an empty and
illusory platitude.

DBM overstepped its authority and encroached upon the Court’s fiscal autonomy and
supervision of the court personnel as enshrined in the Constitution, in violation of
the Constitution.
Report of the Office of the Chief Attorney:
(1) the Special Provision of the Judiciary under GAA 2003 was deemed reenacted when the
GAA 2003 was deemed reenacted in 2004
- vests in the CJ the authority to formulate and implement the organizational structure
of the Judiciary, to fix and determine the salaries, allowances and other benefits of
their personnel, and whenever public interest so requires, make adjustments in the
personal services itemization, including but not limited to the transfer of item or
creation of new positions in the Judiciary
(2) DBM violated the separation of powers as it tried to alter a Resolution of the Court
- As the Constitution binds the Judiciary, as its ultimate interpreter, to observe it and
the law, while the DBM is duty bound not only to accord respect for the issuances of
the SC but also the implementation
- DBM cannot presume that the Court will violate budgetary laws or go beyond the
ambit of its authority or issue administrative resolutions in derogation of law
(3) DBM can only “call the attention of the Court” on what it may perceive as erroneous
application of budgetary laws and rules on position classification
- only after calling its attention may it (Court) amend or modify its Resolution, as its
judgement and discretion may dictate under the law
➡ the change of two position titles was made apparently to conform to position titles
indicated in the personnel services itemization for all government positions, clearly
oblivious of the fact that positions in the Judiciary are peculiar only to that branch of
government. It appearing that the salary grades of 25 and 23 are proper positions
equivalent to those of SC Chief Judicial Staff Officer and Supervising Judicial Staff Officer,
respectively, under the Salary Standardization Law, and that the Court prescribed those
position titles only after consideration of the nature of work and functions that the
holders of those positions must perform, there is no reason to amend the Resolutions of
24 February 2004, and of 5 July 2005, so as to reflect the position titles and salary grades
stated in the NOSCA for the same positions.”

GSIS vs. Heirs of Caballero


October 4, 2012
Facts:
Respondent Fernando, represented by his daughter and attorney in fact Jocelyn
Caballero, filed a petition for certiorari, praying for the annulment of the title issued to
CMTC by GSIS. Fernando Caballero secured a mortgage (Ph 20,000) on the subject property
with GSIS. It was foreclosed and after years of negotiating rental fees, the property was
eventually sold on public auction. CMTC was the highest bidder. Respondent also wants to
annul the decision of the GSIS Board, awarding the bid to CMTC instead of letting them buy
it back. RTC rendered a decision against the respondents and thus ordered them to pay
rent to GSIS amounting to Php240k. Upon the appeal of Fernando, CA deleted the rental fee.
Issue: W/N the bid award should be nullified
Ruling:
(2) Since, GSIS was not exempted from paying the docket fees, the RTC did not acquire
jurisdiction over the case thus, its decision is null and void
- in permissive counterclaims, the counterclaimant is bound to pay the prescribed
docket fees which it did not do
- GSIS asserted that its claim for the collection of rent was a compulsory counterclaim
(3) GSIS cannot be exempted from the payment legal fees
- In Re: Petition for Recognition of the Exemption of GSIS from Payment of Legal Fees:
Section 39 of RA 8291 which exempts GSIS from “all taxes, assessments, fees, charges
and duties of all kinds,” cannot operate to exempt it from the payment of legal fees
- Why? 1987 Constitution removed the power of the Congress to repeal, alter or
supplement the rules of the SC concerning pleading, practice and procedure (unlike
1935 and 1973 Constitutions)
(4) SC has sole authority to promulgate rules concerning pleading, practice and procedure in
all courts
- In Re: Petition for Recognition of the Exemption of GSIS from Payment of Legal Fees:
(a) separation of powers: it is an impregnable wall that keeps the power to promulgate
rules of pleading, practice and procedure within the sole province of this Court
➡viewed from this perspective, the claim of a legislative grant of exemption from the
payment of legal fees under Section 39 of RA 8291 necessarily fails
(b) violates fiscal autonomy: Fiscal autonomy recognizes the power and authority of the
Court to levy, assess and collect fees, including legal fees
➡legal fees under Rule 141 have two basic components, the Judiciary Development
Fund (JDF) and the Special Allowance for the Judiciary Fund (SAJF) whose purpose is
to guarantee the independence of the Judiciary as mandated by the Constitution and
public policy
➡Any exemption from the payment of legal fees granted by Congress to government-
owned or controlled corporations and local government units will necessarily reduce
the JDF and the SAJF. Undoubtedly, such situation is constitutionally infirm for it
impairs the Court’s guaranteed fiscal autonomy and erodes its independence.

COMPOSITION
SECTION 4
(1) The Supreme Court shall be composed of a Chief Justice and fourteen Associate
Justices. It may sit en banc or in its discretion, in division of three, five, or seven
Members. Any vacancy shall be filled within ninety days from the occurrence
thereof.
(2) All cases involving the constitutionality of a treaty, international or executive
agreement, or law, which shall be heard by the Supreme Court en banc, and all
other cases which under the Rules of Court are required to be heard en banc,
including those involving the constitutionality, application, or operation of
presidential decrees, proclamations, orders, instructions, ordinances, and other
regulations, shall be decided with the concurrence of a majority of the Members
who actually took part in the deliberations on the issues in the case and voted
thereon.
(3) Cases or matters heard by a division shall be decided or resolved with the
concurrence of a majority of the Members who actually took part in the
deliberations on the issues in the case and voted thereon, and in no case without
the concurrence of at least three of such Members. When the required number is
not obtained, the case shall be decided en banc: Provided, that no doctrine or
principle of law laid down by the court in a decision rendered en banc or in
division may be modified or reversed except by the court sitting en banc.

I. APPOINTMENT
De Castro vs. JBC 2010
March 17, 2010
Facts:
CJ Puno to retire just days before the election. Although the seat is not yet vacant, the
JBC had already started making the short list for nominations.
Issue: W/N the seat for Chief Justice should be filled within 90 days from the
occurence or must wait for the next President
Ruling:
The CJ position should be filled up.
(1) Intent of the framers of the mandate for the President to fill up the vacancy within 90
days
- had the framers intended to extend the prohibition contained in Section 15, Art. VII to
the appointment of the Members of the SC, they could have explicitly done so.
- Valenzuela wherein the Court held that the prohibition covered judicial appointments
is repealed because it did not firmly rest on the deliberations of the Constitutional
Commission
(2) Sec. 15, Art. VII does not apply as well to the other appointments in the Judiciary
- Aytona vs. Castillo: Sec. 15. Art. VII was adopted to avoid midnight appointments
- Valenzuela: Sec. 15, Art. VII is directed against 2 types of appointments: 1) those
made for buying votes and 2) those made for partisan considerations (midnight
appointments)
- only applies to appointments made in the executive department
- framers did not need to extent the prohibition to judiciary appointments because of
the establishment of the JBC and subjecting the the nomination and screening of
candidates for judicial positions to the unhurried and deliberate prior process of the
JBC ensured that there would no longer be midnight appointments to the Judiciary
- creation of JBC eliminates the danger that appointments to the Judiciary can be made
for the purpose of buying votes
(3) the non-applicability of Sec.15, Art. VII to appointments in the Judiciary was confirmed
by the Senior Associate Justice Regalado
- Justice Regalado confirmed that Constitutional Commission records show that the
election ban had no application to appointments to the Court of Appeals
- this confirmation was accepted by the JBC, which then submitted to the President for
consideration the nominees for the 8 vacancies in the CA
(4) Every statute must be interpreted with reference to the context, i.e. that every part must
be considered together with the other parts, and kept subservient to the general intent
of the whole enactment
- Section 14, 15 and 16 of Art. VII are of the same character which all concern the
appointing power of the President
- Section 14 and 16 refer only to appointment in the Executive Department thus
Section 15 also applies only to the same
- absurd to to assume that the framers situated 15 between 14 and 16, if they intended
Section 15 to cover all kinds of Presidential appointments
- if that was their intention, they could have easily inserted a similar prohibition in Art.
VIII within Sec. 4(1)
(5) To hold like the Court did in Valenzuela that Section 15 extends to appointments to the
Judiciary further undermines the intent of the Constitution of ensuring the
independence of the Judicial Department from the Executive and Legislative
Departments.
(6) The period of prohibition is longer than the full mandatory 90-day period to fill the
vacancy in the SC, thus the argument that “the incumbent President does not need to
appoint a CJ during the prohibition period because there will still be about 45 days of
the 90 days mandated in Section 4(1), Art. VIII remaining” cannot hold
- absurd situation
- no deliberations on the mandatory period for the appointment of CJ under Section
4(1) of Art. VIII, the framers neither discussed, nor mentioned, nor referred to the
ban against midnight appointments under Section 15, Art. VII, or its effects = because
they didn’t need to; Section 15 does not apply to appointments in the SC and the
lower courts
(7) in an extreme case, JBC nominee list for the SC vacancy not necessary
- Sec. 9, Art. VIII, refers to appointments coming into the SC from outside

Sections 4(1) imposes on the President the imperative duty to make an appointment of a
Member of the Supreme Court within 90 days from the occurrence of the vacancy. The
failure by the President to do so will be a clear disobedience to the Constitution. The 90-day
limitation fixed in Section 4(1), Article VIII for the President to fill the vacancy in the
Supreme Court was undoubtedly a special provision to establish a definite mandate for the
President as the appointing power, and cannot be defeated by mere judicial interpretation
in Valenzuela to the effect that Section 15, Article VII prevailed because it was “couched in
stronger negative language.” Such interpretation even turned out to be conjectural, in light
of the records of the Constitutional Commission’s deliberations on Section 4 (1), Article
VIII.

on the issue that there is no urgency for the Pres. to appoint a CJ


- Can they appoint an Acting CJ?
➡No. Sections 4(1) and 9 of Article VIII shows that the Supreme Court is composed of a
Chief Justice and 14 Associate Justices, who all shall be appointed by the President from
a list of at least three nominees prepared by the JBC for every vacancy, which
appointments require no confirmation by the Commission on Appointments.
➡The express reference to a Chief Justice abhors the idea that the framers contemplated
an Acting Chief Justice to head the membership of the Supreme Court. Otherwise, they
would have simply written so in the Constitution.
➡CJ position = permanent

May the JBC be compelled to submit the list of nominees to the President?
➡No. It is premature because it is still clear that the JBC still has until May 17, 2010
at the latest to submit the list.
➡for mandamus to lie, the following requisites must be complied with: (a) the plaintiff
has a clear legal right to the act demanded; (b) it must be the duty of the defendant to
perform the act, because it is mandated by law; (c) the defendant unlawfully neglects
the performance of the duty enjoined by law; (d) the act to be performed is ministerial,
not discretionary; and (e) there is no appeal or any other plain, speedy and adequate
remedy in the ordinary course of law.
➡JBC should start the process of selecting the candidates to fill the vacancy in the
Supreme Court before the occurrence of the vacancy = no discretion to submit the list
after the vacancy because it shortens the 90 day period which will illegally deprive the
President ample time granted under the Constitution for the President to make the
appointment
➡ministertial duty of the JBC: to submit a list of nominees before the start of the
President’s mandatory 90-day period to appoint is
➡selection of the candidates whose names will be in the list to be submitted to the
President lies within the discretion of the JBC.
➡The object of the petitions for mandamus herein should only refer to the duty to submit
to the President the list of nominees for every vacancy in the Judiciary, because in
order to constitute unlawful neglect of duty, there must be an unjustified delay in
performing that duty.88 For mandamus to lie against the JBC, therefore, there should be
an unexplained delay on its part in recommending nominees to the Judiciary, that is, in
submitting the list to the President.

Re: Seniority among the four recent appointments


September 28, 2010
Facts:
1. Myra Feb. 16, 55466
Fernandez 2010
2. Eduardo Feb. 16, 55467
Peralta 2010
3. Ramon Feb. 16, 55468
Hernando 2010
4. Nina Feb. 24, 55465
Antonio- 2012
Valenzuela
Office of the President transmitted to the SC the appointments of CA Associate Justices:

All of them took oath on the same day. Fernandez (55466) is the senior associate
justice. There is a conflict between (1) Section 1 Rule 1 of 2009 Internal Rules of the CA
which states that the date and sequence of the appointment of the Justices determine their
seniority courtwide; and (2) Rule II Section 1 of 2009 IRCA, that states that the Associate
Justices shall have precedence according to the order of their appointments as officially
transmitted to the Supreme Court. CA Committee on Rules opined that Section 1, Rule 1
prevails based on the basic rule of statutory construction that gives premium to a specific
provision over a general one. There is clear legislative intent that the seniority will be
based on the dates of their respective appointments. Applying Rule I and Section I, Rule II
of the IRCA vis-a-vis Section 1 of Republic Act No. 8246, the order of precedence/seniority
among Justices Fernandez, Peralta, Jr. and Hernando should be determined according to the
chronological order indicated in the March 10, 2010 letter of transmittal from Hon.
Executive secretary Leandro R. Mendoza and the bar codes accompanying their respective
appointment papers. On the other hand, having been appointed on February 16, 2010, it
logically follows that said Justices collectively have precedence/seniority over Justice
Valenzuela who, despite the placement of her name in said transmittal letter before the
names of the other three new justices of the Court of Appeals and the lower bar code
number accompanying her appointed, was appointed only on February 24, 2010. However,
Valenzuela (55465) opposed saying that seniority should be determined based on the
order of their appointment as transmitted to the SC thus, Rule II should prevail.
Issue: Who is the senior associate? Which rule prevails?
Ruling:
Court disagrees with Valenzuela.
An appointment to a public office is the unequivocal act, of one who has the authority, of
designating or selecting an individual to discharge and perform the duties and functions of
an office or trust. Where the power of appointment is absolute and the appointee has been
determined upon, no further consent or approval is necessary and the formal evidence of
the appointment, the commission, may issue at once. The appointment is deemed complete
once the last act required of the appointing authority has been complied with. SC adopted
the opinion of the CA Committee on Rules .
In Valencia v. Peralta, 8 SCRA 692 (1963), the Court ruled that a written memorial that
can render title to public office indubitable is required. This written memorial is known as
the commission. For purposes of completion of the appointment process, the
appointment is complete when the commission is signed by the executive, and sealed
if necessary, and is ready to be delivered or transmitted to the appointee. Thus,
transmittal of the commission is an act which is done after the appointment has
already been completed. It is not required to complete the appointment but only to
facilitate the effectivity of the appointment by the appointee’s receipt and acceptance
thereof.
The date the commission has been signed by the President (which is the date appearing
on the face of such document) is the date of the appointment. Such date will determine the
seniority of the members of the Court of Appeals in connection with Section 3, Chapter I of
BP 129, as amended by RA 8246. In other words, the earlier the date of the commission of
an appointee, the more senior he/she is over the other subsequent appointees.

Concurring opinion of J.Carpio:


appointments do not bear the same date => seniority is based on the date of the
respective appointments

when the appointments bear the same date => seniority is based on the order in which
the appointments were issued by the President

The law (BP 129 which was amended by Section 1, Rule II of the 2009 of Internal Rules of
CA) clearly specifies that for purposes of determining precedence in seniority in cases
where the appointments do not bear the same date, it is the date of appointment that is the
reckoning point.

When is the appointment completed? Upon acceptance.

II. EN BANC/DIVISION CASES

Public Interest vs. Elma


Facts:
Respondent Elma was appointed as PCGG Chairman. During his tenure, he was
appointed as Chief Presidential Legal Counsel. He accepted the 2nd appointment but
waived any remuneration he may accept as CPLC. Petitioners seek for the appointment to
be held unconstitutional for being incompatible offices pursuant to Art. IX-B, Sec. 7, par. 2
of the Constitution..
The Court held that:
(1) the concurrent appointments of Elma were unconstitutional. The acts of Elma as CPLC
is subject to review of the PCGG Chairman
(2) Sec. 13, Art. VII of the Constitution does not apply because neither the PCGG Chairman
nor the CPLC is a secretary, undersecretary. The section allows the concurrent holding
of positions only when the 2nd post is required by the primary functions of the 1st
appointment and is exercised in ex officio capacity.
Respondent’s request for clarification:
Are both appointments void?
No. Following the common law rule on incompatibility of offices, Elma had in effect
vacated his first seat as PCGG Chairman when he accepted his position as CPLC.
Can the case be heard en banc?
No. What is in question in the present case is the constitutionality of respondent Elma’s
concurrent appointments, and not the constitutionality of any treaty, law or agreement.
The mere application of constitutional provisions does not require the case to be heard and
decided en banc. Contrary to the allegations of the respondent, the decision of the Court in
this case does not modify the ruling in Civil Liberties Union v. Executive Secretary. It should
also be noted that Section 3 of Supreme Court Circular No. 2–89, dated 7 February 1989
clearly provides that the Court en banc is not an Appellate Court to which decisions or
resolutions of a Division may be appealed.

Fortich vs. Corona


August 19, 1999
Facts:
This is a 2nd Motion for Reconsideration regarding the decision made by the SC division
which set aside the CA decision nullifying RTC’s decision allowing the Province of Cam Sur
to take possession of private respondent’s property, orders the RTC to suspend the
expropriation proceedings and requires the Province of Cam Sur to obtain the approval of
the DAR to convert or reclassify private respondent’s property to agricultural to non-
agricultural use. The assailed decision had the unanimous vote of the 5 members of the 2nd
Division.
Respondents and Intervenors file motion for reconsideration and motion to refer this
case en banc. Court’s resolution dated Nov. 17, 1998 did not effectively resolve the motions
for reconsiderations as well as whether the case should be heard en banc because of the tie
vote of the Court (2-2.) Respondents and intervenors contend that the case should be heard
en banc pursuant to Art. VIII, Section 4(3).

Issue: W/N the case should be heard en banc


Ruling:
Difference between ‘matter’ and ‘case’
A careful reading of the above constitutional provision, however, reveals the intention
of the framers to draw a distinction between cases, on the one hand, and matters, on the
other hand:
Cases - to be decided; “decision” refer to cases
Matters - to be resolved; “resolved” refer to matters which include motions
– applying the rule of reddendo singula singulis. This is true not only in the interpretation of
the above-quoted Article VIII, Section 4(3), but also of the other provisions of the
Constitution where these words appear.

Thus, only cases are to be referred en banc for decision whenever the required vote is
not obtained. The rule does not apply where, as in this case, the required 3 votes is
not obtained in the resolution of a motion for reconsideration.

➡ Second sentence of Section 4(3), Art. VIII:


(1) refers only to cases and not matters
(2) only apply in cases decided by a division
If there is a tie in voting, there is no decision. The only way to dispose of the case is to
refer it en banc.
If a case has already been decided by the division, and the losing party files a motion for
reconsideration, the failure of the division to resolve the motion because of a tie in voting
does not leave the case undecided. In Motions for Reconsiderations, if the voting results in
a tie, the motion is lost. The assailed decision is deemed affirmed.

➡2nd Motions for Reconsiderations are prohibited (1997 rules of civil procedure)
- must be filed with express leave of court

Firestone vs. CA
June 28, 2000
Facts:
This is a Resolution of the SC issued pertaining to the Motions to Refer to the Court En
Banc the consolidated cases which the 3rd division decided on Sept. 2, 1999. The Motion
for Recon are filed by RP and Firestone Ceramics. The issue at hand involves a 99-hectare
land presumptively belonging to RP and the validity of the disposition of the same.
March 8, 2000, 3rd division voted 4-1 to deny the motion to transfer the cases en banc.
March 14, 2000, the Court deliberated on the consulta and thereafter voted 9-5 to accept
the cases en banc.

Justification of the SC for their action of accepting the case en banc.


It is a valid and legitimate exercise of its residual powers within the cotemplation of
paragraph 9 of the Resolution En Banc of November 18, 1993, which reads: “All other cases
as the court en banc by a majority of its actual membership may deem of sufficient
importance to merit its attention.”
The nature of the cases calls for en banc attention and consideration. Taking into
account the importance of these cases and the issues raised, let alone the enormous value
of the area in litigation, which is claimed as government property, there is merit in the
prayer of petitioners that their pending motions for reconsideration should be resolved by
the Court En Banc.

League of Cities vs. COMELEC


December 21, 2009
Facts:
These are petitions to declare Cityhood laws unconstitutional. Court en banc on
November 18, 2008, by a 6-5 vote, granted the petitions and nullified the 16 Cityhood laws
for being violative of Section 10, Art. X of the Constitution.
1. local governments assailed it in MR
- SC denied MR in 1st resolution dated March 31, 2009
- 6-6 vote
2. 2nd Resolution dated April 28, 2009
- 6-6 vote
- denied MR for being a prohibited pleading
3. May 14, 2009: respondent LGUs filed a Motion to Amend the Resolution of April 28,
2009 by Declaring Instead that Respondents’ Motion for Reconsideration of the
Rsolution of March 31, 2009 and Motion for Leave to File and to Admit Attached ‘Second
Motion for Reconsideration of the Decision Dated November 18, 2008 Remain
Unresolved and to Conduct Further Proceedings Thereon.
- tie vote is inadequate to declare a law unconstitutional
- June 2, 2009 Resolution: May 14, 2009 motion reverted to expunged.
4. LGU’s filed a MR again of the Resolution of June 2, 2009
- Pursuant to Sec. 4(2), Art. VIII of the Constitution, all cases involving the
constitutionality of a law shall be heard en banc and decided with the concurrence of a
majority of the Members who actually took part in the deliberations on the issues in
the case and voted thereon
Issue: W/N the required vote set forth in Sec. 4(2), Art. VIII is limited only to
(1) the initial vote on the petition or also to the subsequent voting on the MR where
the Court is called upon and actually votes on the constitutionality of the law or
like issuances; OR
(2) would a minute resolution dismissing, on a tie vote, a MR on the ground that the
issues have already been passed suffice the voting requirement required for a
declaration of the unconstitutionality of the cityhood laws in question

Ruling:
Sec. 4(2), Art. VIII, 1987 Constitution
“All cases involving the constitutionality of a treaty, international or executive
agreement, or law shall be heard by the Supreme Court en banc, x x x shall be decided
with the concurrence of a majority of the Members who actually took part in the
deliberations on the issues in the case and voted thereon.”
A.M. No. 99-1-09-SC
x x x A motion for reconsideration of a decision or resolution of the Court En Banc or of a
Division may be granted upon a vote of a majority of the En Banc or of a Division, as the
case may be, who actually took part in the deliberation of the motion.
If the voting results in a tie, the motion for reconsideration is deemed denied.”

The instant cases all under Sec. 4(2), Art. VIII. The last vote on the issue of the
constitutionality of the cityhood bills is the April 28, 2009 Resolution which is 6-6.
1. the finality of the Nov. 18, 2008 decision has yet to set in
2. deadlock on the 2nd MR did not settle the constitutionality of the cityhood laws
➡Court has powers to suspend its own rules when the ends of justice will be served
thereby.

The Court, by a vote of 6-4, grants the respondent LGUs’ motion for reconsideration of the
Resolution of June 2, 2009, as well as their May 14, 2009 motion to consider the second
motion for reconsideration of the November 18, 2008 Decision unresolved, and also grants
said second motion for reconsideration.

The Court finds that the LGC-amending RA 9009, no less, intended the LGUs covered by the
cityhood laws to be exempt from the PhP 100 million income criterion. In other words, the
cityhood laws, which merely carried out the intent of RA 9009, adhered, in the final
analysis, to the “criteria established in the Local Government Code,” pursuant to Sec. 10,
Art. X of the 1987 Constitution.

Lu vs. Lu Ym,Sr.
February 11, 2011
Facts:
This is an amended 2nd Motion for Reconsideration and Motion to refer Resolution to
the Court En Banc.
Ruling:

The Internal Rules of the Supreme Court (IRSC) states that the Court en banc shall act on
the following matters and cases:
(1) cases in which the constitutionality or validity of any treaty, international or executive
agreement, law, executive order, presidential decree, proclamation, order, instruction,
ordinance, or regulation is in question;
(2) criminal cases in which the appealed decision imposes the death penalty or reclusion
perpetua;
(3) cases raising novel questions of law;
(4) cases affecting ambassadors, other public ministers, and consuls;
(5) cases involving decisions, resolutions, and orders of the Civil Service Commission, the
Commission on Elections, and the Commission on Audit;
(6) cases where the penalty recommended or imposed is the dismissal of a judge, the
disbarment of a lawyer, the suspension of any of them for a period of more than one
year, or a fine exceeding forty thousand pesos;
(7) cases covered by the preceding paragraph and involving the reinstatement in the
judiciary of a dismissed judge, the reinstatement of a lawyer in the roll of attorneys, or
the lifting of a judge’s suspension or a lawyer’s suspension from the practice of law;
(8) cases involving the discipline of a Member of the Court, or a Presiding Justice, or any
Associate Justice of the collegial appellate court;
(9) cases where a doctrine or principle laid down by the Court en banc or by a Division my
be modified or reversed;
(10) cases involving conflicting decisions of two or more divisions;
(11) cases where three votes in a Division cannot be obtained;
(12) Division cases where the subject matter has a huge financial impact on businesses or
affects the welfare of a community;
(13) Subject to Section 11 (b) of this rule, other division cases that, in the opinion of at least
three Members of the Division who are voting and present, are appropriate for transfer
to the Court en banc;
(14) cases that the Court en banc deems of sufficient importance to merit its attention; and
(15) all matters involving policy decisions in the administrative supervision of all courts and
their personnel.

reiterated decision in Firestone


In Firestone Ceramics v. Court of Appeals, 334 SCRA 465 (2000), the Court treated the
consolidated cases as En Banc cases and set the therein petitioners’ motion for oral
argument, after finding that the cases were of sufficient importance to merit the Court En
Banc’s attention. It ruled that the Court’s action is a legitimate and valid exercise of its
residual power.
In Limketkai Sons Milling, Inc. v. Court of Appeals, the Court conceded that it is not
infallible. Should any error of judgment be perceived, it does not blindly adhere to such
error, and the parties adversely affected thereby are not precluded from seeking relief
therefrom, by way of a motion for reconsideration. In this jurisdiction, rectification of an
error, more than anything else, is of paramount importance. x x x x
It bears stressing that where, as in the present case, the Court En Banc entertains a case
for its resolution and disposition, it does so without implying that the Division of origin is
incapable of rendering objective and fair justice. The action of the Court simply means that
the nature of the cases calls for en banc attention and consideration. Neither can it be
concluded that the Court has taken undue advantage of sheer voting strength. It was
merely guided by the well-studied finding and sustainable opinion of the majority of its
actual membership—that, indeed, subject cases are of sufficient importance meriting the
action and decision of the whole Court. It is, of course, beyond cavil that all the members of
this highest Court of the land are always embued with the noblest of intentions in
interpreting and applying the germane provisions of law, jurisprudence, rules and
Resolutions of the Court—to the end that public interest be duly safeguarded and rule of
law be observed.

Doctrine of Immutability of Decisions


The doctrine of immutability of decisions applies only to final and executory decisions.
Since the present cases may involve a modification or reversal of a Court-ordained doctrine
or principle, the judgment rendered by the Special Third Division may be considered
unconstitutional, hence, it can never become final. It finds mooring in the deliberations of
the framers of the Constitution: On proposed Section 3(4), Commissioner Natividad asked
what the effect would be of a decision that violates the proviso that “no doctrine or
principle of law laid down by the court in a decision rendered en banc or in division may be
modified or reversed except by the court en banc.” The answer given was that such a
decision would be invalid. Following up, Father Bernas asked whether the decision, if not
challenged, could become final and binding at least on the parties. Romulo answered that,
since such a decision would be in excess of jurisdiction, the decision on the case could be
reopened anytime. (emphasis and underscoring supplied) A decision rendered by a
Division of this Court in violation of this constitutional provision would be in excess of
jurisdiction and, therefore, invalid. Any entry of judgment may thus be said to be
“inefficacious” since the decision is void for being unconstitutional.

Justice Minerva Gonzaga-Reyes opined in Firestone


En Banc exercises no appellate jurisdictions over its Divisions, the only constraint is that
any doctrine or principle of law laid down by the Court, either rendered en banc or in
division, may be overturned or reversed only by the Court sitting en banc.

EN BANC CAN REVERSE DOCTRINES OR PRINCIPLES. The law allows a determination at


first impression that a doctrine or principle laid down by the court en banc or in division
may be modified or reversed in a case which would warrant a referral to the Court En Banc.
The use of the word “may” instead of “shall” connotes probability, not certainty, of
modification or reversal of a doctrine, as may be deemed by the Court. Ultimately, it is the
entire Court which shall decide on the acceptance of the referral and, if so, “to reconcile any
seeming conflict, to reverse or modify an earlier decision, and to declare the Court’s
doctrine.”

POWER TO CORRECT WELL-ENTRENCHED DOCTRINES. The Court has the power and
prerogative to suspend its own rules and to exempt a case from their operation if and when
justice requires it, as in the present circumstance where movant filed a motion for leave
after the prompt submission of a second motion for reconsideration but, nonetheless, still
within 15 days from receipt of the last assailed resolution. Well-entrenched doctrines or
principles of law that went astray need to be steered back to their proper course.
Specifically, as David Lu correctly points out, it is necessary to reconcile and declare the
legal doctrines regarding actions that are incapable of pecuniary estimation, application of
estoppel by laches in raising an objection of lack of jurisdiction, and whether bad faith can
be deduced from the erroneous annotation of lis pendens.

JUDICIAL POWER
SECTION 5
The Supreme Court shall have the following powers:
(1) Exercise original jurisdiction over cases affecting ambassadors, other public
ministers and consuls, and over petitions for certiorari, prohibition, mandamus,
quo warranto, and habeas corpus.
(2) Review, revise, reverse, modify, or affirm on appeal or certiorari, as the law or the
Rules of Court may provide, final judgments and orders of lower courts in:
(a) All cases in which the constitutionality or validity of any treaty, international
or executive agreement, law, presidential decree, proclamation, order,
instruction, ordinance, or regulation is in question.
(b) All cases involving the legality of any tax, impost, assessment, or toll, or any
penalty imposed in relation thereto.
(c) All cases in which the jurisdiction of any lower court is in issue.
(d) All criminal cases in which the penalty imposed is reclusion perpetua or
higher.
(e) All cases in which only an error or question of law is involved.
(3) Assign temporarily judges of lower courts to other stations as public interest may
require. Such temporary assignment shall not exceed six months without the
consent of the judge concerned.
(4) Order a change of venue or place of trial to avoid a miscarriage of justice.
(5) Promulgate rules concerning the protection and enforcement of constitutional
rights, pleading, practice, and procedure in all courts, the admission to the
practice of law, the integrated bar, and legal assistance to the underprivileged.
Such rules shall provide a simplified and inexpensive procedure for the speedy
disposition of cases, shall be uniform for all courts of the same grade, and shall
not diminish, increase, or modify substantive rights. Rules of procedure of special
courts and quasi-judicial bodies shall remain effective unless disapproved by the
Supreme Court.
(6) Appoint all officials and employees of the Judiciary in accordance with the Civil
Service Law.

JURISDICTION
Planters vs. Fertphil
March 14, 2008
Facts:
This is a petition for Review filed by Planters assailing the decision of the CA affirming
with modification that of the RTC of Makati finding petitioner Planters Products, Inc. liable
to private respondent Fertphil Corporation for the levies it paid under LOI 1465.
Pres. Marcos levied taxes by virtue or LOI 1465 thereby increasing the price of
fertilizers. The revenue of such will go to Planters. After the EDSA Revolution, Fertilizer and
Pesticide Authority voluntarily stopped the imposition of the P10 levy. With the return of
the democracy, Fertiphil demanded from PPI a refund of the amounts it paid under LOI
1465.
RTC rendered LOI 1465 unconstitutional for the taxes were not used for public purpose
but for the private corporation, Planters.
CA affirmed the decision of the RTC only deleting attorney’s fees. The Constitutionality
of LOI 1465 is the very lis mota of the case at bar.
- General Rule: where the controversy can be settled on other grounds the courts will not
resolve the constitutionality of a law (Lim vs. Pacquing)
- courts can rule on the constitutionality of a law when the following are present:
(1) actual case
(2) ripe for adjudication
(3) locus standi
(4) question of constitutionality must have been raised at the earliest opportunity
(5) issue of constitutionality must be the very lis mota of the case

Issue: W/N the RTC had jurisdiction over the case

Ruling:
RTC has jurisdiction to resolve the constitutionality of LOI 1465 pursuant to Sec. 5,
Art. VIII of the 1987 Constitution.

Mirasol vs. CA - Regional Trial Courts have the authority and jurisdiction to consider the
constitutionality of a statute, presidential decree, or executive order. The Constitution vests
the power of judicial review or the power to declare a law, treaty, international or
executive agreement, presidential decree, order, instruction, ordinance, or regulation not
only in this Court, but in all Regional Trial Courts.
Equi-Asia Placement, Inc vs. DFA - the Constitution vests the power of judicial review or the
power to declare a law, treaty, international or executive agreement, presidential decree,
order, instruction, ordinance, or regulation in the courts, including the regional trial
courts.”

Judicial review of official acts on the ground of unconstitutionality may be sought or


availed of through any of the actions cognizable by courts of justice, not necessarily in a suit
for declaratory relief. The constitutional issue, however, (a) must be properly raised and
presented in the case, and (b) its resolution is necessary to a determination of the case, i.e.,
the issue of constitutionality must be the very lis mota presented.

British American vs. Camacho (Lucky Strike)


August 20, 2008
Facts:
Petitioners assail the constitutionality of section 145 of the National Internal Revenue
Code as recodified by (1) RA 8424, (2) RA 9334, (3) Revenue Regulations Nos. 1-97, 9-2003
and 22-2003 and (4) Revenue Memorandum Order No. 6-2003. Petitioners claim that it
violates the equal protection and the uniformity clauses of the Constitution.
Paragraph (c) of Sec. 145 provides 4 tiers of tax rates based on the net retail price per
pack of cigarettes. New brands of cigarettes shall be taxed according to their current net
retail price while existing or “old” brands shall be taxed based on their net retail price as of
October 1, 1996.
RTC dismissed the petition filed by petitioners for a TRO of Revenue Memorandum
Order 6-2003. It said that the courts have no authority to restrain the collection of taxes.
Upon the MR of the petitioners, the RTC upheld the constitutionality of Section 145 of NIRC.
Pending the petition before the SC, RA 9334 increased excise tax as provided in
paragraph (c) of Sec. 145. Thus making British American liable for P 22,775,000.00 in
taxes. It filed a Motion to Admit Attached Supplement and a Supplement to the petition for
review, assailing the constitutionality of RA 9334 and prayed for a downward clasification
of Lucky Strike products which are currently taxed at P26 per pack as Marlboro and Philip
Morris are taxed at P10.88 per pack.
Issue: W/N RTC or regular courts have authority to pass upon constitutionality of a
statute.
Ruling:
While the above statute confers on the CTA jurisdiction to resolve tax disputes in
general, this does not include cases where the constitutionality of a law or rule is
challenged. Where what is assailed is the validity or constitutionality of a law, or a rule or
regulation issued by the administrative agency in the performance of its quasi-legislative
function, the regular courts have jurisdiction to pass upon the same.
The determination of whether a specific rule or set of rules issued by an administrative
agency contravenes the law or the constitution is within the jurisdiction of the regular
courts. Indeed, the Constitution vests the power of judicial review or the power to declare a
law, treaty, international or executive agreement, presidential decree, order, instruction,
ordinance, or regulation in the courts, including the regional trial courts. This is within the
scope of judicial power, which includes the authority of the courts to determine in an
appropriate action the validity of the acts of the political departments. Judicial power
includes the duty of the courts of justice to settle actual controversies involving rights
which are legally demandable and enforceable, and to determine whether or not there has
been a grave abuse of discretion amounting to lack or excess of jurisdiction on the part of
any branch or instrumentality of the Government.

Gerochi vs. Department of Energy


July 17, 2007
Facts:
Petitioners seek to render RA 9136 otherwise known as Electric Power Industry
Reform Act of 2001 (EPIRA) imposing universal charge and Rule 18 of the Rules and
Regulations which seek to implement the imposition, be declared unconstitutional and that
refunds be made, TRO issued to stop the collection of the said charge. They said that the
universal charge provided for under Sec. 34 of the EPIRA, and sought to be implemented
under Sec. 2, rule 18 of the IRR of the said law is a tax which is to be collected from all
electric end-users and self-generating entities. The power to tax is strictly a legislative
function and as such, the delegation of said power to any executive or administrative
agency like the ERC is unconstitutional.
The petitioners directly filed this petition to the SC violating the doctrine of hierarchy of
courts. The complaint is bereft of any allegation of grave abuse of discretion on the part of
the ERC or any of the public respondents, in order for the Court to consider it as a petition
for certiorari or prohibition.
Issue: W/N the SC has jurisdiction over the case
Ruling:
Court’s jurisdiction to issue writs of certiorari, prohibition, mandamus, quo warranto,
and habeas corpus, as provided for in Art. VIII, Sec. 5(1) and (2) of the 1987 Constitution
while concurrent with that of the regional trial courts and the Court of Appeals, does not
give litigants unrestrained freedom of choice of forum from which to seek such relief.
It has long been established that this Court will not entertain direct resort to it unless
the redress desired cannot be obtained in the appropriate courts, or where exceptional and
compelling circumstances justify availment of a remedy within and call for the exercise of
our primary jurisdiction.29 This circumstance alone warrants the outright dismissal of the
present action.
This procedural infirmity notwithstanding, we opt to resolve the constitutional issue
raised herein. We are aware that if the constitutionality of Sec. 34 of the EPIRA is not
resolved now, the issue will certainly resurface in the near future, resulting in a repeat of
this litigation, and probably involving the same parties. In the public interest and to avoid
unnecessary delay, this Court renders its ruling now.
The instant complaint is bereft of merit.

Kida vs. Senate


February 28, 2012
Facts:
This case concerns the constitutionality of RA 9054 and 10153 which allows the
holdover of the term of the officers in the ARMM, extending their terms beyond the 3-year
limit for the synchronization of the Elections. The petitioners in G.R. No. 197280, in their
Manifestation and Motion dated December 21, 2011, question the propriety of the
appointment by the President of Mujiv Hataman as acting Governor and Bainon Karon as
acting Vice Governor of the ARMM. They argue that since our previous decision was based
on a close vote of 8-7, and given the numerous motions for reconsideration filed by the
parties, the President, in recognition of the principle of judicial courtesy, should have
refrained from implementing our decision until we have ruled with finality on this case.

Issue: W/N Principle of Judicial Courtesy applies


Ruling:
No. The principle of “judicial courtesy” applies where there is a strong probability that
the issues before the higher court would be rendered moot and moribund as a result of the
continuation of the proceedings in the lower court or court of origin. Consequently, this
principle cannot be applied to the President, who represents a co-equal branch of
government. To suggest otherwise would be to disregard the principle of separation of
powers, on which our whole system of government is founded upon.
Secondly, the fact that our previous decision was based on a slim vote of 8-7 does not,
and cannot, have the effect of making our ruling any less effective or binding. Regardless of
how close the voting is, so long as there is concurrence of the majority of the members of
the en banc who actually took part in the deliberations of the case, a decision garnering
only 8 votes out of 15 members is still a decision of the Supreme Court en banc and must be
respected as such. The petitioners are, therefore, not in any position to speculate that,
based on the voting, “the probability exists that their motion for reconsideration may be
granted.”

EFFECT OF UNCONSTITUTIONALITY
Planters vs. Fertphil
Ruling:
Planters’s argument that Fertphil cannot ask for a refund is untenable even if LOI 1465
is declared unconstitutional. Doctrine of operative fact is inapplicable in this case.
General Rule: An unconstitutional law is void thus produces no rights, imposes no duties
and affords no protection. It has no legal effect. It is, in legal contemplation, inoperative as if
it has not been passed.

Exception: Doctrine of operative fact, is the exception to the general rule, only applies as
a matter of equity and fair play. It nullifies the effects of an unconstitutional law by
recognizing that the existence of a statute prior to a determination if unconstitutionality is
an operative fact and may have consequences which cannot always be ignored. The past
cannot always be erased by a new judicial declaration. The doctrine is applicable when a
declaration of unconstitutionality will impose an undue burden on those who have relied
on the invalid law.

➡Thus Fertiphil is not required to pay the levy and should be refunded in accordance with
the general civil code principle against unjust enrichment
Art. 7. Laws are repealed only by subsequent ones, and their violation or non-observance
shall not be excused by disuse or custom or practice to the contrary.
When the courts declare a law to be inconsistent with the Constitution, the former shall
be void and the latter shall govern.”

We cannot allow PPI to profit from an unconstitutional law. Justice and equity dictate
that PPI must refund the amounts paid by Fertiphil.

De Agbayani vs. PNB


1971
Facts:
De Agbayani was able to get a favorable judgement permanently enjoining PNB and
Provincial Sheriff of Pangasinan from proceeding with an extra-judicial foreclosure sale of
land belonging to plaintiff which she mortgaged. The prescription had already lapsed which
had effectively rendered the the right of foreclosure unenforceable. The issue is whether
the moratorium under the Executive Order and later an Act subsequently found
unconstitutional were to be counted in the computation of the prescription period.
RTC ruled that the unconstitutional act is not a law, creating no rights and imposing no
duties, and thus inoperative as if it had never been.
Issue: W/N an unconstitutional Act still has effects
Ruling:
Yes. It is now accepted as a doctrine that prior to its being nullified, its existence as a
fact must be reckoned with. This is merely to reflect awareness that precisely because the
judiciary is the governmental organ which has the final say on whether or not a legislative
or executive measure is valid, a period of time may have elapsed before it can exercise the
power of judicial review that may lead to a declaration of nullity. It would be to deprive the
law of its quality of fairness and justice then, if there be no recognition of what had
transpired prior to such adjudication.
The existence of of a statute or EO prior to its being adjudged void is an operative fact to
which legal consequences are attached.
Although the Moratorium Law was unconstitutional due to the impairment of the
obligations of contracts, precisely though because of the judicial recognition that
moratorium was a valid governmental response to the plight of the debtors who were war
sufferers, this Court has made clear its view in a series of cases impressive in their number
and unanimity that during the eight-year period that Executive Order No. 32 and Republic
Act No. 342 were in force, prescription did not run. Thus, the lower court erred in
sustaining plaintiff’s suit. When extra-judicial foreclosure proceedings were started by the
PNB, the time consumed was 6 days short of 15 years hence, within the prescription
period.

Flores vs. Drilon


June 22, 1993
Facts:
This case involves the constitutionality of Sec. 13, par. (d) of RA 7227 or the Bases
Conversion and Development Act of 1992. Pursuant to the said law, Mayor Richard Gordon
of Olongapo City was appointed Chairman and CEO of the Subic Bay Metropolitan
Authority.

Issue:
whether the proviso in Sec. 13, par. (d), of RA 7227 which states, “Provided, however, That
for the first year of its operations from the effectivity of this Act, the mayor of the City of
Olongapo shall be appointed as the chairman and chief executive officer of the Subic
Authority,” violates the constitutional proscription against appointment or designation of
elective officials to other government posts.
Ruling:
It is unconstitutional.
It violates Sec. 7, par. 1, Art. IX-B of the Constitution.
It limits the appointing powers of the President.

Congress did not contemplate making the subject SBMA posts as ex officio or
automatically attached to the Office of the Mayor of Olongapo City without need of
appointment. The phrase “shall be appointed” unquestionably shows the intent to make the
SBMA posts appointive and not merely adjunct to the post of Mayor of Olongapo City. Had it
been the legislative intent to make the subject positions ex officio, Congress would have, at
least, avoided the word “appointed” and, instead, “ex officio” would have been used.

Did he vacate his position as Mayor?


Consequently, as long as he is an incumbent, an elective official remains ineligible for
appointment to another public office. Hence he does not automatically forfeit his elective
office nor remove his ineligibility imposed by the Constitution. (different than Sec. 13, Art.
VI where no senator or member of HoR) Other incumbent elective officials must first resign
their posts before they can be appointed, thus running the risk of losing the elective post as
well as not being appointed to the other post. It is therefore clear that ineligibility is not
directly related with forfeiture of office.
Are his acts without legal effect? What kind of officer was he?
Mayor Gordon’s acts as SBMA official are not necessarily null and void; he may be
considered a de facto officer, “one whose acts, though not those of a lawful officer, the law,
upon principles of policy and justice, will hold valid so far as they involve the interest of the
public and third persons, where the duties of the office were exercised x x x x under color of
a known election or appointment, void because the officer was not eligible, or because
there was a want of power in the electing or appointing body, or by reason of some defect
or irregularity in its exercise, such ineligibility, want of power or defect being unknown to
the public x x x x [or] under color of an election, or appointment, by or pursuant to a public
unconstitutional law, before the same is adjudged to be such (State vs. Carroll, 38 Conn.,
499; Wilcox vs. Smith, 5 Wendell [N.Y.], 231; 21 Am. Dec. 213; Sheehan’s Case, 122 Mass,
445, 23 Am. Rep., 323).”
Conformably with our ruling in Civil Liberties Union, any and all per diems, allowances and
other emoluments which may have been received by respondent Gordon pursuant to his
appointment may be retained by him.

Tan vs. Barrios


October 18, 1990
Facts:
This case involves the General Orders issued by the President pursuant to his
Proclamation fixing the jurisdiction of military tribunals during martial law. Included
crimes that were under the jurisdiction of civil courts. Thus after martial law ended, Court
promulgated in their decision in Olaguer vs. Military Commission No. 34, et. al. that military
commissions and tribunals have no jurisdiction, even during the period of martial law, over
civilians charged with criminal offenses properly cognizable by civil courts, as long as those
courts are open and functioning as they did during the period of martial law.
In Cruz vs. Enrile the Court nullified the proceedings made by the military tribunals and
tried the case of therein defendants anew.
Tan is one of the defendants in another criminal case of murder with possession of
unlicensed firearm which was tried under the military tribunals. He was already acquitted
even before the issuance of Oliguer and Cruz vs. Enrile. However, the Court, in line with
their decision in Cruz vs. Enrile, ordered a re-investigation by Barrios, the fiscal of Cagayan
de Oro. William Tan, Joaquin Tan Leh and Vicente Tan filed this petition for certiorari and
prohibition praying that the informations in the criminal cases be annulled, and to stop the
public respondents from trying them anew for crimes they have already been acquitted of.
First Division of SC dismissed the petition for being premature.
2nd Motion for Recon was filed informing the Court that warrants for their arrest had
already been issued. SC released TRO enjoining the petitioners’ arrest.
Issue: W/N the unconstitutionality of the General Orders fixing the military tribunals
and the jurisdiction thereof consequently nullified the proceedings already made.
Ruling:
No. Court held that the Olaguer and Cruz vs. Enrile rulings must be prospectively applied.
There should be no retroactive nullification of final judgements. Final judgements should
not be disturbed unless there is a serious denial of the accused’s Constitutional rights.

Doctrine of operative facts applies to the proceedings against the petitioners and
their co-accused before Military Commission No. 1. The principle of absolute invalidity of
the jurisdiction of the military courts over civilians should not be allowed to obliterate the
"operative facts" that in the particular case of the petitioners, the proceedings were fair,
that there were no serious violations of their constitutional right to due process, and that
the jurisdiction of the military commission that heard and decided the charges against
them during the period of martial law, had been affirmed by this Court (Aquino vs. Military
Commission No. 2, 63 SCRA 546) years before the Olaguer case arose and came before us.
Because of these established operative facts, the refiling of the information against the
petitioners would place them in double jeopardy, in hard fact if not in constitutional logic.

Depriving the petitioners of the protection of the judgment of acquittal rendered by the
military commission in their particular case by retroactively divesting the military
commission of the jurisdiction it had exercised over them would amount to an ex post facto
law or ruling.

What is an ex post facto law?


(1)makes criminal an act done before the passage of the law and which was innocent when
done, and punishes such an act;
(2)aggravates a crime, or makes it greater than it was, when committed;
(3)changes the punishment and inflicts a greater punishment than the law annexed to the
crime when committed
(4)alters the legal rules of evidence, and authorizes conviction upon less or different
testimony than the law required at the time of the commission of the offense;
(5)assuming to regulate civil rights and remedies only, in effect imposes penalty or
deprivation of a right for something which when done was lawful; and,
(6)deprives a person accused of a crime of some lawful protection to which he has become
entitled, such as the protection of a former conviction or acquittal, or a proclamation of
amnesty." (In re: Kay Villegas Kami, Inc., 35 SCRA 428, 431)

Article IV, Section 22, of the 1987 Constitution prohibits the enactment of an ex post
facto law or bill of attainder.

Tatad vs. Secretary


November 5, 1997
Facts:
Involves the constitutionality of RA 8180 which deregulates the downstream oil
industry which sets a different tariff rate to newcomers.
Held:
1. W/N RA 8180 is unconstitutional = YES
2. W/N the offending provisions can be individually struck down without
invalidating the entire RA 8180.
Entire law has to be struck down.
General Rule: Where part of a statute is void as repugnant to the Constitution, while
another part is valid, the valid portion, if separable from the invalid, may stand and be
enforced.
- the valid portion must be so far independent of the invalid portion that it is fair to
presume that the legislature would have enacted it by itself if it had supposed that it
could not constitutionaly enact the other.
- enough must remain to make a complete, intelligible and valid statute, which carries
out the legislative intent
Exception: when the parts of a statute are so mutually dependent and connected, as
conditions, considerations, inducements, or compensations for each other, as to warrant a
belief that the legislature intended them as a whole, the nullity of one part will vitiate the
rest. In making the parts of the statute dependent, conditional, or connected with one
another, the legislature intended the statute to be carried out as a whole and would not
have enacted it if one part is void, in which case if some parts are unconstitutional, all the
other provisions thus dependent, conditional, or connected must fall with them.”
R.A. No. 8180 contains a separability clause. Section 23 provides that “if for any reason,
any section or provision of this Act is declared unconstitutional or invalid, such parts not
affected thereby shall remain in full force and effect.”
This separability clause notwithstanding, we hold that the offending provisions of R.A.
No. 8180 so permeate its essence that the entire law has to be struck down. The
provisions on tariff differential, inventory and predatory pricing are among the principal
props of R.A. No. 8180. Congress could not have deregulated the downstream oil industry
without these provisions. Unfortunately, contrary to their intent, these provisions on tariff
differential, inventory and predatory pricing inhibit fair competition, encourage
monopolistic power and interfere with the free interaction of market forces. R.A. No. 8180
needs provisions to vouchsafe free and fair competition. The need for these vouchsafing
provisions cannot be overstated. Before deregulation, PETRON, SHELL and CALTEX had no
real competitors but did not have a free run of the market because government controls
both the pricing and non-pricing aspects of the oil industry. After deregulation, PETRON,
SHELL and CALTEX remain unthreatened by real competition yet are no longer subject to
control by government with respect to their pricing and non-pricing decisions. The
aftermath of R.A. No. 8180 is a deregulated market where competition can be corrupted
and where market forces can be manipulated by oligopolies.

Central Bank vs. Bangko


December 15, 2004
Facts:
RA 7653 abolished the old Central Bank of the Philippines, and created a new BSP.
R.A. No. 7653, petitioner Central Bank (now BSP) Employees Association, Inc., filed a
petition for prohibition against BSP and the Executive Secretary of the Office of the
President, to restrain respondents from further implementing the last proviso in Section
15(c), Article II of R.A. No. 7653, on the ground that it is unconstitutional.
Salary Standardization Law only covered the rank-and-file employees and not the BSP
officers. The petitioners claim that this is a classic case of class legislation and thus violates
the equal protection of the law.
Issue: W/N last paragraph of Section 15(c), Article II, of RA 7653 violates the equal
protection clause of the Constitution
Ruling:
We hold that with the passage of the subsequent laws amending the charter of seven
(7) other governmental financial institutions (GFIs), the continued operation of the last
proviso of Section 15(c), Article II of Republic Act (R.A.) No. 7653, constitutes invidious
discrimination on the 2,994 rank-and-file employees of the Bangko Sentral ng Pilipinas
(BSP).

Concept of Relative Constitutionality


The constitutionality of a statute cannot, in every instance, be determined by just
comparing its provisions with applicable provisions of the Constitution, since the statute
may be constitutionally valid as applied to one set of facts and invalid in its application to
another.
A statute valid at one time may become void at another time because of altered
circumstances. Thus, if a statute in its practical operation becomes arbitrary or confiscatory,
its validity, eventhough affirmed by a former adjudication, is open to inquiry and
investigation in the light of changed conditions.
Yes, said portion of the law is violative of the equal protection clause
The disparity of treatment between BSP rank-and-file and the rank-and-file of the other
seven GFIs definitely bears the unmistakable badge of invidious discrimination—no one
can, with candor and fairness, deny the discriminatory character of the subsequent blanket
and total exemption of the seven other GFIs from the SSL when such was withheld from the
BSP. Alikes are being treated as unalikes without any rational basis.

League of Cities vs. COMELEC


August 24, 2010
Facts:
SC renders the Cityhood laws to be violative of the equal protection clause thus,
unconstitutional.
Issue: W/N the operative fact doctrine operates to constitutionalize the
unconstitutional Cityhood Laws
Ruling:
No.
Under the operative fact doctrine, the law is recognized as unconstitutional but the
effects of the unconstitutional law, prior to its declaration of nullity, may be left
undisturbed as a matter of equity and fair play. In fact, the invocation of the operative fact
doctrine is an admission that the law is unconstitutional.
The operative fact doctrine is a rule of equity. As such, it must be applied as an
exception to the general rule that an unconstitutional law produces no effects. It can never
be invoked to validate as constitutional an unconstitutional act.
The operative fact doctrine never validates or constitutionalizes an unconstitutional
law. Under the operative fact doctrine, the unconstitutional law remains unconstitutional,
but the effects of the unconstitutional law, prior to its judicial declaration of nullity, may be
left undisturbed as a matter of equity and fair play. In short, the operative fact doctrine
affects or modifies only the effects of the unconstitutional law, not the unconstitutional law
itself.
Thus in this case, Cityhood Laws remain unconstitutional but payments of salaries and
supplies by the “new cities” or their issuances of licenses or execution of contracts, may be
recognized as valid and effective.
Only the effects of the implementation of these unconstitutional laws are left
undisturbed as a matter of equity and fair play to innocent people who may have relied on
the presumed validity of the Cityhood Laws prior to the Court’s declaration of their
unconstitutionality.

Yap vs. Thenaman’s Ship


May 30, 2011
Facts:
Claudio Yap is a seaman whose ship was sold and scrapped. He filed a Complaint for
Illegal Dismissal with Damages and Atty’s fees before the LA. He said that he is entitled to
the salaries corresponding to the unexpired portion of his contract. He only received one
month basic wage.
LA decided in favor of Yap which awarded him P100,000 plus P50,000 in moral and
exemplary damages. NLRC affirmed the LA’s findings that petitioner was illegally dismissed
and modified it holding that petitioner is only entitled to 3 months as provided for under
Sec 10 of RA 8042 which provides that in case of termination of overseas employment
without just, valid or authorized cause as defined by law or contract, the workers shall be
entitled to the full reimbursement of his placement fee with interest of twelve percent (12%)
per annum, plus his salaries for the unexpired portion of his employment contract or for
three (3) months for every year of the unexpired term, whichever is less.
Without getting a favorable decision from the NLRC, the respondent’s went to the CA
and then the SC.
Pending the petition, 5th paragraph of Section 10 (above mentioned) was struck down
as unconstitutional in Serrano vs. Gallant Maritime.
Issue: W/N doctrine of operative fact should apply
Ruling:
Court reiterated Planters <see ruling>
Doctrine of operative fact should not be applied. It was not the fault of petitioner that he
lost his job due to an act of illegal dismissal committed by respondents. To rule otherwise
would be iniquitous to petitioner and other OFWs, and would, in effect, send a wrong signal
that principals/employers and recruitment/manning agencies may violate an OFW’s
security of tenure which an employment contract embodies and actually profit from such
violation based on an unconstitutional provision of law.
SC awards the salaries of Yap for his unexpired term at a rate of $ 1430 / month

Hacienda Luisita vs. Presidential Agrarian Reform Council


November 22, 2011
Facts:
On July 5, 2011, the Supreme Court en banc voted unanimously (11-0) to
DISMISS/DENY the petition filed by HLI and AFFIRM with MODIFICATIONS the resolutions
of the PARC revoking HLI’s Stock Distribution Plan (SDP) and placing the subject lands in
Hacienda Luisita under compulsory coverage of the Comprehensive Agrarian Reform
Program (CARP) of the government.
The Court however did not order outright land distribution. Voting 6-5, the Court noted
that there are operative facts that occurred in the interim and which the Court cannot
validly ignore. Thus, the Court declared that the revocation of the SDP must, by application
of the operative fact principle, give way to the right of the original 6,296 qualified
farmworkers-beneficiaries (FWBs) to choose whether they want to remain as HLI
stockholders or [choose actual land distribution]. It thus ordered the Department of
Agrarian Reform (DAR) to “immediately schedule meetings with the said 6,296 FWBs and
explain to them the effects, consequences and legal or practical implications of their choice,
after which the FWBs will be asked to manifest, in secret voting, their choices in the ballot,
signing their signatures or placing their thumbmarks, as the case may be, over their printed
names.”
In their motion for partial reconsideration, DAR and PARC argue that the doctrine of
operative fact does not apply to the instant case since: (1) there is no law or rule which has
been invalidated on the ground of unconstitutionality; (2) the doctrine of operative fact is a
rule of equity which may be applied only in the absence of a law, and in this case, they
maintain that there is a positive law which mandates the distribution of the land as a result
of the revocation of the stock distribution plan (SDP).
AMBALA submits that the operative fact doctrine should only be made to apply in the
extreme case in which equity demands it, which allegedly is not in the instant case.
FARM posits that the option given to the FWBs is equivalent to an option for HLI to
retain land in direct violation of RA 6657.
Respondents argue that the operative fact doctrine should not be applied since there is
a positive law, particularly, Sec. 31 of RA 6657, which directs the distribution of the land as
a result of the revocation of the SDP.

Issue: W/N doctrine of operative fact applies in this case where PARC Resolution No.
89-12-2, an executive act, was previously declared invalid
Ruling:
1. Doctrine of operative fact applies to:
a) unconstitutional laws; and
b) executive acts subsequently declared invalid
- De Agbayani vs. CA: legislative or executive act, prior to its being declared
unconstitutional by the courts is valid and must be complied with
- City of Makati vs. CSC: a void act though in law a mere scrap of paper nonetheless
confers legitimacy upon past acts or omissions done in reliance thereof
- Tanada vs. Tuvera : the implementation/enforcement of presidential decrees
prior to their publication in the Official Gazette is ‘an operative fact which may
have consequences which cannot be justly ignored. The past cannot always be
erased by a new judicial declaration that an all-inclusive statement of a principle
of absolute retroactive invalidity cannot be justified
2. Executive acts should be understood in general terms and thus should be considered to
which doctrine of operative fact can apply to. The operative fact doctrine is not confined
to statutes and rules and regulations issued by the executive department that are
accorded the same status as that of a statute or those which are quasi-legislative in
nature.
- Public Interest Center vs. Elma : Prior to the declaration of unconstitutionality of the
said executive act, certain acts or transactions were made in good faith and in reliance
of the appointment of Elma which cannot just be set aside or invalidated by its
subsequent invalidation.
- Tan vs. Barrios : despite the invalidity of the jurisdiction of the military courts over
civilians, certain operative facts must be acknowledged to have existed so as not to
trample upon the rights of the accused therein.
- This doctrine, in the interest of justice and equity, can be applied liberally and in a
broad sense to encompass said decisions of the executive branch. In keeping with the
demands of equity, the Court can apply the operative fact doctrine to acts and
consequences that resulted from the reliance not only on a law or executive act which
is quasi-legislative in nature but also on decisions or orders of the executive branch
which were later nullified.
➡A decision made by the President or the administrative agencies has to be complied
with because it has the force and effect of law, springing from the powers of the
President under the Constitution and existing laws. Prior to the nullification or recall
of said decision, it may have produced acts and consequences in conformity to and in
reliance of said decision, which must be respected. It is on this score that the operative
fact doctrine should be applied to acts and consequences that resulted from the
implementation of the PARC Resolution approving the SDP of HLI.
3. Doctrine of operative fact as Recourse in Equity
- by applying the operative fact doctrine, this Court merely gave the qualified FWBs the
option to remain as stockholders of HLI and ruled that they will retain the homelots
and other benefits which they received from HLI by virtue of the SDP.
- this is favorable to the FWBs

RULE MAKING POWER

Tañada vs. Angara


May 2, 1997
Facts:
Philippines joined the World Trade Organization as its founding member with the goal,
as stated by PFVR, of improving Philippine access to foreign markets, especially its major
trading partners, through the reduction of tariffs on its exports, particularly agricultural
and industrial products. The Agreement was ratified by the Senate. Petitioner seeks that
the treaty be nullified on Constitutional grounds.
The petitioner argues that
(1) WTO places nationals and products of member-countries on the same footing as
Filipinos and local products
(2) WTO “intrudes, limits and/or impairs” the constitutional powers of both Congress an d
th e Su preme Court, the instant petition before this Court assails the WTO Agreement
for violating the mandate of the 1987 Constitution to “develop a self-reliant and
independent national economy effectively controlled by Filipinos x x x (to) give
preference to qualified Filipinos (and to) promote the preferential use of Filipino labor,
domestic materials and locally produced goods.

Petitioners aver that paragraph 1, Article 34 of the General Provisions and Basic
Principles of the Agreement on Trade-Related Aspects of Intellectual Property Rights
(TRIPS)49 intrudes on the power of the Supreme Court to promulgate rules concerning
pleading, practice and procedures.

In Art. 34 of TRIPS, a WTO Member is required to provide a rule of disputable


presumption that a product shown to be identical to one produced with the use of a
patented process shall be deemed to have been obtained by the (illegal) use of the said
patented process, (1) where such product obtained by the patented product is new, or (2)
where there is “substantial likelihood” that the identical product was made with the use of
the said patented process but the owner of the patent could not determine the exact
process used in obtaining such identical product.

Issue: W/N the provisions unduly impair or interfere with the exercise of judicial
power by the Court in promulgating rules on evidence
Ruling:
No. The patent owner still has the “burden of proof” since, regardless of the presumption
provided under paragraph 1 of Article 34, such owner still has to introduce evidence of the
existence of the alleged identical product, the fact that it is “identical” to the genuine one
produced by the patented process and the fact of “newness” of the genuine product or the
fact of “substantial likelihood” that the identical product was made by the patented process.

The foregoing should really present no problem in changing the rules of evidence as the
present law on the subject, Republic Act No. 165, as amended, otherwise known as the
Patent Law, provides a similar presumption in cases of infringement of patented design or
utility model, thus:
“Sec. 60. Infringement.—Infringement of a design patent or of a patent for utility model
shall consist in unauthorized copying of the patented design or utility model for the
purpose of trade or industry in the article or product and in the making, using or selling of
the article or product copying the patented design or utility model. Identity or substantial
identity with the patented design or utility model shall constitute evidence of copying.” (italics
supplied)

Article 34 does not contain an unreasonable burden, consistent as it is with due process
and the concept of adversarial dispute settlement inherent in our judicial system.
So too, since the Philippines is a signatory to most international conventions on patents,
trademarks and copyrights, the adjustment in legislation and rules of procedure will not be
substantial.

Webb vs. De Leon


August 23, 1995
Facts:
This involves the Vizconde massacre. Hubert Webb, among others, in their petitions at
bar, contend: (1) respondent Judges de Leon and Tolentino gravely abused their discretion
when they failed to conduct a preliminary examination before issuing warrants of arrest
against them; (2) the DOJ Panel likewise gravely abused its discretion in holding that there
is probable cause to charge them with the crime of rape with homicide; (3) the DOJ Panel
denied them their constitutional right to due process during their preliminary
investigation; and (4) the DOJ Panel unlawfully intruded into judicial prerogative when it
failed to charge Jessica Alfaro in the Information as an accused.
Petitioners fault the DOJ Panel for not including Alfaro in the Information considering
her alleged conspiratorial participation in the crime of rape with homicide. The non-
inclusion of Alfaro is anchored on Republic Act No. 6981, entitled “An Act Providing For A
Witness Protection, Security And Benefit Program And For Other Purposes” enacted on April
24, 1991. Alfaro qualified under its Section 10. Upon qualification of Alfaro to the program,
Section 12 of the said law mandates her non-inclusion in the criminal Complaint or
Information.
The validity of these provisions is challenged by petitioner Webb. It is urged that they
constitute “x x x an intrusion into judicial prerogative for it is only the court which has the
power under the Rules on Criminal Procedure to discharge an accused as a state witness.”
The argument is based on Section 9, Rule 11938 which gives the court the prerogative to
approve the discharge of an accused to be a state witness.
Issue:W/N Section 10 and 12 intrudes into the power of the court under the Rules on
Criminal Procedure to discharge an accused as a state witness.
Ruling:
Petitioner’s argument lacks appeal for it lies on the faulty assumption that the decision
whom to prosecute is a judicial function, the sole prerogative of courts and beyond
executive and legislative interference. In truth, the prosecution of crimes appertains to the
executive department of government whose principal power and responsibility is to see
that our laws are faithfully executed. A necessary component of this power to execute our
laws is the right to prosecute their violators. The right to prosecute vests the prosecutor
with a wide range of discretion—the discretion of whether, what and whom to charge, the
exercise of which depends on a smorgasbord of factors which are best appreciated by
prosecutors.
We thus hold that it is not constitutionally impermissible for Congress to enact R.A. No.
6981 vesting in the Department of Justice the power to determine who can qualify as a
witness in the program and who shall be granted immunity from prosecution.39 Section 9 of
Rule 119 does not support the proposition that the power to choose who shall be a state
witness is an inherent judicial prerogative. Under this provision, the court is given the
power to discharge a state witness only because it has already acquired jurisdiction over
the crime and the accused. The discharge of an accused is part of the exercise of jurisdiction
but is not a recognition of an inherent judicial function. Moreover, the Rules of Court have
never been interpreted to be beyond change by legislation designed to improve the
administration of our justice system. R.A. No. 6981 is one of the much sought penal reform
laws to help government in its uphill fight against crime, one certain cause of which is the
reticence of witnesses to testify. The rationale for the law is well put by the Department of
Justice, viz: “Witnesses, for fear of reprisal and economic dislocation, usually refuse to
appear and testify in the investigation/prosecution of criminal complaints/cases. Because
of such refusal, criminal complaints/ cases have been dismissed for insufficiency and/or
lack of evidence. For a more effective administration of criminal justice, there was a
necessity to pass a law protecting witnesses and granting them certain rights and benefits
to ensure their appearance in investigative bodies/courts.”40 Petitioner Webb’s challenge
to the validity of R.A. No. 6981 cannot therefore succeed.

Nicolas vs. Romulo


February 11, 2009
Facts:
Respondent Lance Corporal Daniel Smith is a member of the USAF. He was charged with
rape committed against a Filipina, Suzette Nicolas. Pending the court proceedings against
respondent, the custody of respondent was granted to the US pursuant to the VFA. which
the Philippines entered with the US. While waiting for orders, the RTC of Makati detained
him in the Makati Jail.
Smith was taken out of the Makati Jail and brought to the facilities of the US for
detainment. This is because of the new agreements between the US and the Philippines =
Romulo-Kenney Agreement of December 19, 2006 which granted the US custody of Smith.
Petitioners contend that the transfer of the custody of Smith violate Sec 5(5) of Art. VIII
which provides for the exclusive power of the Court to adopt rules of procedure for all
courts of the Philippines. The transfer of custody is tantamount to providing a different
procedure for that accused which also violates the equal protection clause of the
Constitution.
Held:
W/N the presence of USAF in Philippine territory pursuant to the VFA is allowed
under a treaty duly concurred in by the Senate and recognized as a treaty by the
other contracting State.
Yes. Why?
1. In Bayan v. Zamora, the VFA was duly concurred in by the Philippine Senate and has
been recognized as a treaty by the United States as attested and certified by the duly
authorized representative of the United States government.
2. The VFA is the implementing agreement to the RP-US Mutual Defense Treaty of August
30, 1951 which was signed and duly ratified with the concurrence with both the
Philippine Senate and the US Senate.
- as an implementing agreement, it was not necessary to submit the VFA to the US
Senate for advice and consent but merely to the US Congress under the Case-Zablocki
Act within 60 days of ratification
- this substantially complies with the requirements of Art. XVIII, Sec. 25 of the
Constitution
W/N the violates Art. VIII, Sec. 5(5) of the 1987 Constitution
The situation involved is not one in which the power of this Court to adopt rules of
procedure is curtailed or violated, but rather one in which, as is normally encountered
around the world, the laws (including rules of procedure) of one State do not extend or
apply—except to the extent agreed upon—to subjects of another State due to the
recognition of extraterritorial immunity given to such bodies as visiting foreign armed
forces.
Nothing in the Constitution prohibits such agreements recognizing immunity from
jurisdiction or some aspects of jurisdiction (such as custody), in relation to long-recognized
subjects of such immunity like Heads of State, diplomats and members of the armed forces
contingents of a foreign State allowed to enter another State’s territory. On the contrary,
the Constitution states that the Philippines adopts the generally accepted principles of
international law as part of the law of the land. (Art. II, Sec. 2).

Echegaray vs. Secretary


January 19, 1999
Facts:
Public Respondents filed an Urgent Motion for Reconsideration of the Resolution of the
Court dated January 4, 1999 temporarily restraining the execution of petitioner and
Supplemental to Urgent Motion for Reconsideration.
Issues:
(1) The Decision in this case having become final and executory, its execution enters the
exclusive ambit of authority of the executive authority. The issuance of the TRO may be
construed as trenching on that sphere of executive authority;

(2) The issuance of the temporary restraining order x x x creates dangerous precedent as
there will never be an end to litigation because there is always a possibility that
Congress may repeal a law;
(3) Congress had earlier deliberated extensively on the death penalty bill. To be certain,
whatever question may now be raised on the Death Penalty Law before the present
Congress within the 6-month period given by this Honorable Court had in all probability
been fully debated upon x x x
(4) Under the time honored maxim lex futuro, judex praeterito, the law looks forward while
the judge looks at the past, x x x the Honorable Court in issuing the TRO has transcended
its power of judicial review;
Ruling:
Court granted Motion of the Public Respondents
=> ordered the setting a new date for the execution
(1) The Court did not lose its jurisdiction over the case at bar. The Court did not change the
final Decision. It is appropriate to examine with precision the metes and bounds of the
Decision of this Court that became final.
- Justice Quiason: “the finality of a judgment does not mean that the Court has lost all
its powers nor the case. By the finality of the judgment, what the court loses is its
jurisdiction to amend, modify or alter the same. Even after the judgment has become
final the court retains its jurisdiction to execute and enforce it.3 There is a difference
between the jurisdiction of the court to execute its judgment and its jurisdiction to
amend, modify or alter the same. The former continues even after the judgment has
become final for the purpose of enforcement of judgment; the latter terminates when
the judgment becomes final.4 x x x For after the judgment has become final facts and
circumstances may transpire which can render the execution unjust or impossible.
- To be sure, the most important part of a litigation, whether civil or criminal, is the
process of execution of decisions where supervening events may change the
circumstance of the parties and compel courts to intervene and adjust the rights of
the litigants to prevent unfairness. It is because of these unforseen, supervening
contingencies that courts have been conceded the inherent and necessary power of
control of its processes and orders to make them conformable to law and justice.
(2) The power to control the execution of its decision is an essential aspect of jurisdiction. It
cannot be the subject of substantial subtraction for our Constitution vests the entirety of
judicial power in one Supreme Court and in such lower courts as may be established by
law.
- The most important part of a litigation, whether civil or criminal, is the process of
execution of decisions where supervening events may change the circumstance of the
parties and compel courts to intervene and adjust the rights of the litigants to prevent
unfairness.
- It is because of these unforseen, supervening contingencies that courts have been
conceded the inherent and necessary power of control of its processes and orders to
make them conformable to law and justice.
- For this purpose, Section 6 of Rule 135 provides that “when by law jurisdiction is
conferred on a court or judicial officer, all auxiliary writs, processes and other means
necessary to carry it into effect may be employed by such court or officer and if the
procedure to be followed in the exercise of such jurisdiction is not specifically pointed
out by law or by these rules, any suitable process or mode of proceeding may be
adopted which appears conformable to the spirit of said law or rules.”
- It bears repeating that what the Court restrained temporarily is the execution of its
own Decision to give it reasonable time to check its fairness in light of supervening
events in Congress as alleged by petitioner. The Court, contrary to popular
misimpression, did not restrain the effectivity of a law enacted by Congress.
(3) Court promulgated rules on execution of judgements — rules predicated on the
assumption that courts have the inherent, necessary, and incidental power to control
and supervise the process of execution of their decisions
- Rule 39 - execution
- Rule 120 - judgements in criminal case
(4) It should be stressed that the power to promulgate rules of pleading, practice and
procedure was granted by our Constitutions to this Court to enhance its independence, for
in the words of Justice Isagani Cruz “without independence and integrity, courts will lose
that popular trust so essential to the maintenance of their vigor as champions of justice.”
- 1987 made a stronger and more independent Judiciary when it eliminated the power
of the Congress to repeal, alter, or supplement the rules concerning pleading, practice
of law in the Philippines as in the 1935 and 1973 Constitutions
- Rule -making power of Judiciary was expanded pursuant to Art. VIII, Section 5 (5)
(a) power to promulgate its own rules concerning the protection and enforcement of
constitutional rights
(b) power to disapprove rules of procedure of special courts and quasi-judicial
bodies

Re: Petition for Recognition


February 11, 2010
Issue: W/N GSIS is exempt from paying legal fees
Ruling:
GSIS is not exempt from paying legal fees.
Payment of legal fees, an integral party of the Rule-Making Power. The payment of
legal fees under Rule 141 of the Rules of Court is an integral part of the rules promulgated
by this Court pursuant to its rule-making power under Section 5(5), Article VIII of the
Constitution. In particular, it is part of the rules concerning pleading, practice and
procedure in courts. Indeed, payment of legal (or docket) fees is a jurisdictional
requirement. It is not simply the filing of the complaint or appropriate initiatory pleading
but the payment of the prescribed docket fee that vests a trial court with jurisdiction over
the subject-matter or nature of the action. Appellate docket and other lawful fees are
required to be paid within the same period for taking an appeal. Payment of docket fees in
full within the prescribed period is mandatory for the perfection of an appeal. Without such
payment, the appellate court does not acquire jurisdiction over the subject matter of the
action and the decision sought to be appealed from becomes final and executory.
Court has waived legal fees with regard to indigent litigants. Interesting aspect of
legal fees is that which relates to indigent or pauper litigants. In proper cases, courts may
waive the collection of legal fees. This, the Court has allowed in Section 21, Rule 3 and
Section 19, Rule 141 of the Rules of Court in recognition of the right of access to justice by
the poor under Section 11, Article III of the Constitution. Mindful that the rule with respect
to indigent litigants should not be ironclad as it touches on the right of access to justice by
the poor, the Court acknowledged the exemption from legal fees of indigent clients of the
Public Attorney’s Office under Section 16-D of the Administrative Code of 1987, as
amended by RA 9406. This was not an abdication by the Court of its rule-making power but
simply a recognition of the limits of that power. In particular, it reflected a keen awareness
that, in the exercise of its rule-making power, the Court may not dilute or defeat the right of
access to justice of indigent litigants.
Legal fees, vital source of Court’s financial resources and essential element of the
Court’s fiscal independence. Congress could not have carved out an exemption for the
GSIS from the payment of legal fees without transgressing another equally important
institutional safeguard of the Court’s independence—fiscal autonomy. Fiscal autonomy
recognizes the power and authority of the Court to levy, assess and collect fees, including
legal fees. Moreover, legal fees under Rule 141 have two basic components, the Judiciary
Development Fund (JDF) and the Special Allowance for the Judiciary Fund (SAJF). The laws
which established the JDF and the SAJF expressly declare the identical purpose of these
funds to “guarantee the independence of the Judiciary as mandated by the Constitution and
public policy.” Legal fees therefore do not only constitute a vital source of the Court’s
financial resources but also comprise an essential element of the Court’s fiscal
independence. Any exemption from the payment of legal fees granted by Congress to
government-owned or controlled corporations and local government units will necessarily
reduce the JDF and the SAJF. Undoubtedly, such situation is constitutionally infirm for it
impairs the Court’s guaranteed fiscal autonomy and erodes its independence.

INTEGRATED BAR OF THE PHILIPPINES


Garcia vs. De Vera
December 11, 2003
Facts:
This is a case to disqualify Leonard de Vera from being Elected Governor of Eastern
Mindanao in 16th IBP Regional Governor’s Elections.
De Vera transferred his membership from Paranaque to Agusan del Sur chapter. The
transfer of IBP membership to Agusan del Sur, the petitioners went on, is a brazen abuse
and misuse of the rotation rule, a mockery of the domicile rule and a great insult to lawyers
from Eastern Mindanao for it implies that there is no lawyer from the region qualified and
willing to serve the IBP.
Petitioners claim that de Vera lacks the moral fitness required of a candidate of a
regional director for the ff. reasons:
1. irresponsibly attacking the SC Justices during the deliberations on the constitutionality
of the plunder law
2. de Vera could have been disbarred in the US for misappropriating his client’s funds had
he not surrendered his California license to practice law
3. he actively campaigned for the position of Eastern Mindanao Governor during the IBP
National Convention held on May 22-24, 2003, a prohibited act under the IBP By-Laws.
Respondent on the other hand contends that the Court does not have jurisdiction over
the case because being an issue of whether qualified for IBP Governor, is a purely internal
matter. He assails the petitioners’ legal standing because IBP By Laws does not have a
provision for the disqualfication of IBP Members aspiring for the position of Regional
governors and pursuant to Art. IV, Section 40, only a valid nominee can lodge an election
protest. Thus, only members from the Surigao del Norte and Agusan del Sur IBP Chapters
are qualified to run for Governor for Eastern Mindanao Region. The Petitioners are not
qualified to be nominees.
IBP Board dismissed the petition that was also raised before them on the ground that it
is premature as De Vera has not been nominated for Governor.
Held:
W/N this Court has jurisdiction over the present controversy
Yes. Section 5, Art. VIII of the 1987 Constitution confers on the SC the power to
promulgate rules affecting the IBP. Implicit in this constitutional grant is the power to
supervise all the activities of the IBP, including the elections of the officers. Pursuant to this
power of supervision, the Court initiated the integration of the Philippine Bar by creating
on October 5, 1970 the Commission on Bar Integration, which was tasked to ascertain the
advisability of unifying the Philippine Bar.
Not long after, Republic Act No. 6397 was enacted and it confirmed the power of the
Supreme Court to effect the integration of the Philippine Bar. Finally, on January 1, 1973, in
the per curiam Resolution of this Court captioned “In the Matter of the Integration of the
Bar to the Philippines,” we ordained the Integration of the Philippine Bar in accordance
with Rule 139-A, of the Rules of Court, which we promulgated pursuant to our rule-making
power under the 1935 Constitution.

W/N the present Petition is premature;


assuming that petitioners have a cause of action and that the present petition is not
premature,
Court agrees that it is premature for the petitioners to seek disqualification of
respondent De Vera from being elected IBP Governor for the Eastern Mindanao Region.
Before a candidate can be elected for Governor, he must be nominated. In this case, de Vera
has not been nominated.
W/N respondent De Vera is disqualified to run for Governor of the IBP Eastern
Mindanao Region;
It is clearly stated in the afore-quoted section of the By-Laws that it is not automatic
that a lawyer will become a member of the chapter where his place of residence or work is
located. He has the discretion to choose the particular chapter where he wishes to gain
membership. Only when he does not register his preference that he will become a member
of the Chapter of the place where he resides or maintains his office. The only proscription
in registering one’s preference is that a lawyer cannot be a member of more than one
chapter at the same time.
The only condition in Sec. 29-2 of the IBP By-Laws is that the transfer must be made not
less than 3 months prior to the election of the officers in the chapter to which the lawyer
wishes to transfer. It has been 17 months since de Vera transferred his membership to
Agusan, therefore making him qualified to be a candidate for Governor of the IBP Eastern
Mindanao Region.
W/N De Vera is disqualified because of moral reasons
No. As long as an aspiring member meets the basic requirements provided in the IBP
By-Laws, he cannot be barred. The basic qualifications for one who wishes to be elected
governor for a particular region are: (1) he is a member in good standing of the IBP;37 (2)
he is included in the voter’s list of his chapter or he is not disqualified by the Integration
Rule, by the ByLaws of the Integrated Bar, or by the By-Laws of the Chapter to which he
belongs; (3) he does not belong to a chapter from which a regional governor has already
been elected, unless the election is the start of a new season or cycle;39 and (4) he is not in
the government service.
Moral fitness is not a qualification explicitly provided for in the IBP By-Laws to run for a
position in the IBP. Why? The disqualification of a candidate involving lack of moral fitness
should emanate from his disbarment or suspension from the practice of law by this Court,
or conviction by final judgment of an offense which involves moral turpitude. How about
that case wherein the Court held him in contempt? Court said that the ruling cannot serve
as a basis to consider respondent De Vera immoral. The act for which he was found guilty
of indirect contempt does not involve moral turpitude.
The petitioners failed to substantiate proof that would show De Vera to be morally unfit
to run for Governor.
Petition dismissed.

Re: Undated Letter of Louis Biraogo


February 24, 2009
Facts:
This is the Report of the Investigating Committee created under the Resolution dated
December 10,2008 to investigate the unauthorized release of the unpromulgated ponencia
of Justice Reuben Reyes in the consolidated cases of Limkaichong vs. COMELEC, Villando vs.
COMELEC, Biraogo vs. Nograles and Limkaichong and Paras vs. Nograles, to determine who
are responsible for the leakage of a confidential internal document of the En Banc.
The Justices en banc, withheld the promulgation of the decision drafted by Justice Reyes
because majority only wanted to concur “in the result” which effectively would make the
ponencia meaningless.
December 9, 2008, Louis Biraogo, petitioner in Biraogo vs. Nograles and Limkaichong
held a press conference and circulated to the media an undated letter signed by him with a
photocopy of the draft decision of Justice Reyes.
Because of the unauthorized release of the unpromulgated ponencia which infringed on
the confidential internal deliberations of the Court and constituted contempt of Court,
Court issued a Resolution which directed the creation of the investigating committee and
for Louis Biraogo to show cause within 10 days from receipt why he shouldn’t be punished
for contempt.
Biraogo submits a letter signed by Justice Reyes.
Finding of the Investigating Committee
1. The Gilbert copy bearing the signatures of 14 Justices was photocopied and that a copy
thereof was intentionally leaked directly or indirectly to Biraogo. As will be discussed
below, the committee FINDS that the leak came from the Office of Justice Reyes.
2. the committee FINDS that based on the circumstantial evidence reflected above,
particularly the evident undue interest of Justice Reyes to circulate a draft ponencia of
the case soonest even before the memoranda of all the parties fell due, and to withhold
the information to Atty. Evangelista and Del Rosario that the promulgation of the
ponencia was put on hold and, instead, allow the immediate promulgation after lunch
despite his admission that the decision to hold the promulgation was arrived at at
lunchtime, it was Justice Reyes himself who leaked a photocopy thereof.
3. Even after the Justices had, at lunchtime of July 15, 2008, unanimously decided that the

promulgation of the Gilbert copy would be put on hold—and this was, it bears repeating,
admitted by Justice Reyes—, Justice Reyes, after partaking lunch at the dining room and
before 1:00 p.m., instead of advising his Chief of Staff Atty. Evangelista and Del Rosario
that the promulgation was put on hold, still instructed them to reprint the second
signature page (page 36) and to have the reprinted page immediately brought to the
Office of Justice Nachura for signature; and before Justice Reyes left for the session hall
for the oral arguments of that case scheduled at 1:30 p.m. that day, Justice Reyes still
followed up the case by asking Manabat if Justice Nachura had already signed the Gilbert
copy.
4. To the members of the committee, the foregoing proven facts and circumstances
constitute more than substantial evidence which reasonably points to Justice Reyes,
despite his protestations of innocence, as THE source of the leak. He must, therefore, be
held liable for GRAVE MISCONDUCT.

What is the effect of Reyes’ retirement?


The subsequent retirement of a judge or any judicial officer from the service does not
preclude the finding of any administrative liability to which he is answerable.
Even if one is already retired, if he is found to be guilty of an administrative charge, he
can be disqualified to hold any government office and his benefits can be forfeited.
The Court may impose its authority upon erring judges who actuations, on their face,
would show gross incompetence, ignorance of the law or misconduct.

Doctrine of res ipsa loquitur. The res ipsa loquitur doctrine does not except or dispense
with the necessity of proving the facts on which the inference of evil intent is based. It
merely expresses the clearly sound and reasonable conclusion that when such facts are
admitted or are already shown by the record, and no credible explanation that would
negative the strong inference of evil intent is forthcoming, no further hearing to establish
them to support a judgment as to the culpability of a respondent is necessary.

Here, the act of Justice Reyes not only violated the New Code of Judicial Conduct for the
Philippine Judiciary, the Code of Judicial Conduct and the Canons of Judicial Ethics, it also
infringed on the internal deliberations of the Court and impeded and degraded the
administration of justice.
That Justice Reyes was an impeachable officer when the investigation started is of no
moment. The rule prohibiting the institution of disbarment proceedings against an
impeachable officer who is required by the Constitution to be a member of the bar as a
qualification in office applies only during his or her tenure and does not create immunity
from liability for possibly criminal acts or for alleged violations of the Code of Judicial
Conduct or other supposed violations.
Once the said impeachable officer is no longer in office because of his removal,
resignation, retirement or permanent disability, the Court may proceed against him or her
and impose the corresponding sanctions for misconduct committed during his tenure,
pursuant to the Court’s power of administrative supervision over members of the bar.
Provided that the requirements of due process are met, the Court may penalize retired
members of the Judiciary for misconduct committed during their incumbency.

Ruling of the Court:


The Court finds the above-quoted report well taken. Pursuant to Section 13, Article VIII of
the Constitution, this per curiam decision was reached after deliberation of the Court En
Banc by a unanimous decision of all the members of the Court except for two (2) Justices
who are on official leave.
WHEREFORE, in view of the foregoing, the Court ADOPTS the findings and APPROVES
WITH MODIFICATION the Recommendations of the Investigating Committee as follows:

(1) Justice Ruben T. Reyes (Ret.) is held liable for GRAVE MISCONDUCT for leaking a
confidential internal document of the Court and he is FINED P500,000.00, to be charged
against his retirement benefits, and disqualified to hold any office or employment in any
branch or instrumentality of the government including government-owned or controlled
corporations; furthermore, Justice Ruben T. Reyes is directed to SHOW CAUSE within ten
(10) days from receipt of a copy of this Decision why he should not be disciplined as a
member of the Bar in light of the aforementioned findings.

(2) Atty. Rosendo B. Evangelista and Armando Del Rosario are held liable for SIMPLE
NEGLECT OF DUTY and are ordered to pay the FINE in the amount of P10,000.00 and
P5,000.00, respectively.

Section 6. The Supreme Court shall have administrative supervision over all courts
and the personnel thereof.
Maceda v. Vasquez (1993)
Facts:
The Office of the Ombudsman held the Presiding Judge of Antique RTC Branch 12, Bonifacio
Maceda criminally liable for falsifying his Certificate of Service in 1989 and 1990.
However, in his petition for certiorari, Maceda contends that he had been granted by this
Court an extension of ninety (90) days to decide the aforementioned cases.
Petitioner also contends that the Ombudsman has no jurisdiction over said case despite
this Court's ruling in Orap vs. Sandiganbayan since the offense charged arose from the
judge's performance of his official duties, which is under the control and supervision of the
Supreme Court. Furthermore, the investigation of the Ombudsman constitutes an
encroachment into the Supreme Court's constitutional duty of supervision over all inferior
courts.
Issue: Whether the Office of the Ombudsman could entertain a criminal complaint for the
alleged falsification of a judge's certification submitted to the Supreme Court?
**assuming that it can, whether a referral should be made first to the Supreme Court?
Ruling:
There is nothing in the decision in Orap that would restrict it only to offenses committed by
a judge unrelated to his official duties.

Liability. A judge who falsifies his certificate of service is administratively liable to the
Supreme Court for serious misconduct and inefficiency under Section 1, Rule 140 of the
Rules of Court, and criminally liable to the State under the Revised Penal Code for his
felonious act. However, in the absence of any administrative action taken against him by
the Supreme Court with regard to his certificates of service, the investigation being
conducted by the Ombudsman encroaches into the Court's power of administrative
supervision over all courts and its personnel, in violation of the doctrine of separation of
powers.
Administrative Supervision. Article VIII, section 6 of the 1987 Constitution exclusively
vests in the Supreme Court administrative supervision over all courts and court personnel,
from the Presiding Justice of the Court of Appeals down to the lowest municipal trial court
clerk. By virtue of this power, it is only the Supreme Court that can oversee the judges' and
court personnel's compliance with all laws, and take the proper administrative action
against them if they commit any violation thereof. No other branch of government may
intrude into this power, without running afoul of the doctrine of separation of
powers.
The Ombudsman cannot justify its investigation of petitioner on the powers granted to it by
the Constitution, for such a justification not only runs counter to the specific mandate of the
Constitution granting supervisory powers to the Supreme Court over all courts and their
personnel, but likewise undermines the independence of the judiciary.
Supreme Court’s referral. the Ombudsman should first refer the matter of petitioner's
certificates of service to this Court for determination of whether said certificates reflected
the true status of his pending case load, as the Court has the necessary records to make
such a determination. The Ombudsman cannot compel this Court, as one of the three
branches of government, to submit its records, or to allow its personnel to testify on this
matter, as suggested by public respondent Abiera in his affidavit-complaint.
The rationale for the foregoing pronouncement is evident in this case. Administratively. the
question before Us is this: should a judge, having been granted by this Court an extension of
time to decide cases before him, report these cases in his certificate of service? As this
question had not yet been raised with, much less resolved by, this Court. how could the
Ombudsman resolve the present criminal complaint that requires the resolution of said
question?
In fine, where a criminal complaint against a Judge or other court employee arises from
their administrative duties, the Ombudsman must defer action on said complaint and refer
the same to this Court for determination whether said Judge or court employee had acted
within the scope of their administrative duties.
Dolalas v. Ombudsman (1996)
Facts:
Petitioners Judge Ana Dolalas, Evelyn Obido and Wilberto Carriedo - Presiding Judge, Clerk
of Court and Clerk II, respectively of the Municipal Circuit Trial Court of Zamboanga del Sur
were charged administratively by private respondent Villarente before the Office of the
Ombudsman-Mindanao for miscarriage of justice, dishonesty, gross neglect of duty,
unnecessary delay in the administration of justice and for failure to prosecute a criminal
case for an unreasonable length of time.
Petitioner-judge contends that the Office of the Ombudsman has no jurisdiction to initiate
an investigation into the alleged undue delay in the disposition of the case as said charge
relates to a judges performance of her official duties over which the Supreme Court has
administrative control and supervision, as mandated under Section 6, Rule VIII of the 1987
Constitution.
Public respondent Ombudsman-Mindanao, however, contends that referral to the Supreme
Court is not essential in this case as what is sought to be determined by the investigation is
whether or not any undue delay in the disposition of the alarms and scandals case resulted
in injury to private respondent through manifest partiality, evident bad faith or gross
inexcusable negligence and/or undue advantage to any party, in violation of the Anti-Graft
and Corrupt Practices Act.
Issue:
W/N the Office of the Ombudsman may take cognizance of the complaint against petitioner
for purposes of investigation and possible prosecution in accordance with its mandate
under Section 13 (1) and (2) of Article XI of the 1987 Constitution for alleged violation of
the Anti-Graft and Corrupt Practices Act.
Ruling:
This Court agrees with petitioner-judge. The complaint against petitioner-judge before the
Office of the Ombudsman is basically administrative in nature. It must be borne in mind
that the resolution of the administrative charge of unduly delaying the disposition of the
said criminal case involves the determination of whether, in resolving the alarms and
scandals case, petitioner-judge acted in accordance with the guidelines provided in the
Rules of Court and in the Administrative Circulars in pursuance of the ideals embodied in
the Code of Judicial Conduct. Such is clearly an administrative matter. Unquestionably, this
Court is mandated under Section 6, Article VIII of the 1987 Constitution to assume
administrative supervision over all courts and the personnel thereof.
This Court, in the case of Sanz Maceda v. Vasquez, , held that:
Article VIII, Section 6 of the 1987 Constitution exclusively vests in the Supreme
Court administrative supervision over all courts and court personnel, from the
Presiding Justice of the Court of Appeals down to the lowest municipal trial court
clerk. By virtue of this power, it is only the Supreme Court that can oversee the
judges and court personnels compliance with all laws, and take the proper
administrative action against them if they commit any violation thereof. No other
branch of government may intrude into this power, without running afoul of the
doctrine of separation of powers.
Public respondent Ombudsman cannot justify its investigation of petitioner on the
powers granted to it by the Constitution, for such a justification not only runs
counter to the specific mandate of the Constitution granting supervisory powers to
the Supreme Court over all courts and their personnel, but likewise undermines
the independence of the judiciary.
De Vera v. Pelayo
Facts:
Salvador De Vera instituted with the Regional Trial Court, Pasig City a special civil action to
enjoin the municipal trial court from proceeding with a complaint for ejectment against
him. Judge Pelayo denied the petition as well as his motion for reconsideration which
prompted De Vera to file with the Office of the Ombudsman an affidavit-complaint against
Judge Pelayo, accusing him of violating Articles 206 and 207 of the Revised Penal Code and
Republic Act No. 3019.
However, guided by the decision in Maceda v. Vasquez, the Office of the Ombudsman
referred the case to the Court Administrator of the Supreme Court.
Petitioner criticizes the jurisprudence cited by the Office of the Ombudsman as erroneous
and not applicable to his complaint. He insists that since his complaint involved a criminal
charge against a judge, it was within the authority of the Ombudsman not the Supreme
Court to resolve whether a crime was committed and the judge prosecuted therefor.
Issue: W/N Ombudsman has jurisdiction to entertain criminal charges filed against a judge
of the regional trial court in connection with his handling of cases before the court.
Ruling:
The petition can not succeed.

There was no grave abuse of discretion committed by the Ombudsman. The Ombudsman
did not exercise his power in an arbitrary or despotic manner by reason of passion,
prejudice or personal hostility. There was no evasion of positive duty. Neither was there a
virtual refusal to perform the duty enjoined by law.
We agree with the Solicitor General that the Ombudsman committed no grave abuse of
discretion warranting the writs prayed for. The issues have been settled in the case of In
Re: Joaquin Borromeo. There, we laid down the rule that before a civil or criminal action
against a judge for a violation of Art. 204 and 205 (knowingly rendering an unjust
judgment or order) can be entertained, there must first be "a final and authoritative judicial
declaration" that the decision or order in question is indeed "unjust." The pronouncement
may result from either:
(a).....an action of certiorari or prohibition in a higher court impugning the
validity of the judgment; or
(b).....an administrative proceeding in the Supreme Court against the judge
precisely for promulgating an unjust judgment or order.
Likewise, the determination of whether a judge has maliciously delayed the disposition of
the case is also an exclusive judicial function.
"To repeat, no other entity or official of the Government, not the prosecution
or investigation service of any other branch, not any functionary thereof, has
competence to review a judicial order or decision -- whether final and
executory or not -- and pronounce it erroneous so as to lay the basis for a
criminal or administrative complaint for rendering an unjust judgment or
order. That prerogative belongs to the courts alone (underscoring ours)."

Fuentes v. Ombudsman

Facts:
The dispute of the case was culled from the trial court’s order of writ of execution in favour
of Tessie Amadeos against DPWH for the unpaid expropriated property of the former
which was used for the construction of fly-over project in Davao City. The writ was served
by respondent Sheriff Paralisan to the DPWH-Region XI.
Subsequently, Judge Fuentes granted the ex-parte urgent motion for the issuance of a break
through in favour of Alex Bacquial (winning bidder of the auctioned DPWH properties),
which enable him to effect the withdrawal of the auctioned properties. Together with
Sheriff Paralisan, Bacquial returned to the depot, armed with the lower courts order and
succeeded in hauling off the scrap iron/junk equipment in the depot, including the
repairable equipment within the DPWH depot.
However, DPWH through the Solicitor General, filed an administrative complaint against
Sheriff Norberto Paralisan and was held by the Supreme Court guilty of conduct prejudicial
to the best interest of the service and accordingly, dismissed him from the service. On the
other hand, Director Valenzuela filed with the Office of the Deputy Ombudsman for
Mindanao a criminal complaint charging Judge Renato A. Fuentes with violation of Republic
Act No. 3019, Section 3 (e)

Petitioner alleged that the respondent Ombudsman-Mindanao committed a grave abuse of


discretion amounting to lack or excess of jurisdiction when he initiated a criminal
complaint against petitioner for violation of R.A. No. 3019, Section 3 [e]. And he conducted
an investigation of said complaint against petitioner. Thus, he encroached on the power of
the Supreme Court of administrative supervision over all courts and its personnel.
Issue: W/N the Ombudsman may conduct an investigation of acts of a judge in the exercise
of his official functions alleged to be in violation of the Anti-Graft and Corrupt Practices Act,
in the absence of an administrative charge for the same acts before the Supreme Court.
Ruling:
The Ombudsman may not initiate or investigate a criminal or administrative complaint
before his office against petitioner judge, pursuant to his power to investigate public
officers. The Ombudsman must indorse the case to the Supreme Court, for appropriate
action.

Article VIII, Section 6 of the Constitution exclusively vests in the Supreme Court
administrative supervision over all courts and court personnel, from the Presiding Justice
of the Court of Appeals to the lowest municipal trial court clerk. Hence, it is the Supreme
Court that is tasked to oversee the judges and court personnel and take the proper
administrative action against them if they commit any violation of the laws of the
land. No other branch of government may intrude into this power, without running afoul of
the independence of the judiciary and the doctrine of separation of powers.
Whether or not such order of execution was valid under the given circumstances, must be
inquired into in the course of the judicial action only by the Supreme Court that is tasked to
supervise the courts. No other entity or official of the Government, not the prosecution or
investigation service of any other branch, not any functionary thereof, has competence to
review a judicial order or decision--whether final and executory or not--and pronounce it
erroneous so as to lay the basis for a criminal or administrative complaint for rendering an
unjust judgment or order. That prerogative belongs to the courts alone.
Caoibes v. Ombudsman
Facts:
Judge Florentino Alumbres filed a criminal charge before the Office of the Ombudsman
against Judge Jose Caoibes for unlawfully punching him at the hallway of Las Pinas City Hall
of Justice when he requested to return the executive table Judge Caoibes borrowed from
him, which result damage to his eyeglasses and rendered it unserviceable. Subsequently,
Judge Alumbres, lodged an administrative case against Judge Caoibes, with the Supreme
Court, docketed as Adm. Case No. 97-387-RTJ, praying for the dismissal of petitioner from
the judiciary on the ground of grave misconduct or conduct unbecoming a judicial officer.
Said complaint is based on the same facts as those in the complaint filed earlier with the
office of the Ombudsman.

Petitioner filed an Ex-Parte Motion for Referral to the Honorable Supreme Court, praying
that the Office of the Ombudsman hold its investigation of Case No. OMB-0-97-0903 in
abeyance, and refer the same to the Supreme Court which, through the Office of the Court
Administrator, is already investigating what transpired on May 20, 1997. Petitioner
contended that the Supreme Court, not the Office of the Ombudsman, has the authority to
make a preliminary determination of the respective culpability of petitioner and
respondent Judge who, both being members of the bench, are under its exclusive
supervision and control.
However, the Office of the Ombudsman issued an Order denying the motion for referral to
the Supreme Court. Invoking Section 15 (1) of Republic Act No. 6770, and held that it is
within its jurisdiction to investigate the criminal charges ofJudge Alumbres against
petitioner.
Issue: W/N the Office of the Ombudsman should defer action on case No. OMB-0-97-0903
pending resolution of Adm. Case
No. 97-387-RTJ.

Ruling:
The powers granted in Section 15 (1) of R.A. 6770 to the Office of the Ombudsman is not
tantamount to giving it exclusive authority thereon.

Under Section 6, Article VIII of the Constitution, it is the Supreme Court which is vested
with exclusive administrative supervision over all courts and its personnel. Prescinding
from this premise, the Ombudsman cannot determine for itself and by itself whether a
criminal complaint against a judge, or court employee, involves an administrative matter.
The Ombudsman is duty bound to have all cases against judges and court personnel filed
before it, referred to the Supreme Court for determination as to whether and
administrative aspect is involved therein.

This rule should hold true regardless of whether an administrative case based on the act
subject of the complaint before the Ombudsman is already pending with the Court. For,
aside from the fact that the Ombudsman would not know of this matter unless he is
informed of it, he should give due respect for and recognition of the administrative
authority of the Court, because in determining whether an administrative matter is
involved, the Court passes upon not only administrative liabilities but also other
administrative concerns, as is clearly conveyed in the case of Maceda vs. Vasquez.
The Ombudsman cannot dictate to, and bind the Court, to its findings that a case before it
does or does not have administrative implications. To do so is to deprive the Court of the
exercise of its administrative prerogatives and to arrogate unto itself a power not
constitutionally sanctioned. This is a dangerous policy which impinges, as it does, on
judicial independence.

Adajar v. Develos
Facts:
The dispute of herein case took place within the premises of the RTC, Branch 8, Malaybalay
City, Bukidnon during office hours (around 10:45 a.m.) on February 6, 2002, and that the
bone of contention are the unpaid accounts which complainant seeks to collect by reason of
her business of selling jewelry to court personnel. It is not also disputed that respondents
Develos and Celsa bought jewelry from complainant when the latter was in their office in
October 2000, and that Develos helped complainant by listing the names of the employees
who bought items from her.

Ruling:
Pursuant to the settled rule in Maceda v. Vasquez, the Office of the Ombudsman, Mindanao
should have referred the instant complaint to this Court for appropriate action, instead of
resolving the same. Hence, we agree with the OCA that the Decision rendered by the Office
of the Ombudsman, Mindanao in OMB-M-A-02-126-E does not have any force and effect on
the present administrative case before us.

SC found Teresita Develos and Celsa Ellorin guilty of Administrative Circular Nos. 1-99 and
09-99 while the complaint against Cyrus A. Ellorin was dismissed for insufficiency of
evidence.
CSC v. Andal
Facts:
Herminigildo Andal, a security guard II in Sandiganbayan, was found guilty for dishonesty
by CSC-NCR thereby imposed upon him the penalty of dismissal from the service. It was
discovered upon verification that impersonation was committed when the picture attached
to the Picture Seat Plan was unmatched to the Andal’s picture in his identification card.

Denied with his motion for reconsideration, Respondent elevated the case to the Appellate
Court which set-aside CSC-NCR’s decision and ordered referral of the said administrative
case to the Office of the Court Administrator, Supreme Court, for appropriate action.

Court of Appeals denied the motion for reconsideration of the CSC.

Issue: W/N CSC has disciplinary jurisdiction to try and decide administrative cases against
court personnel.

Ruling:
While respondent may have filed his Answer to the formal charge of dishonesty after
having been directed to do so, he denied having taken the civil service examination and did
not even appear at the formal investigation conducted by the CSC-NCR. He appealed to the
CSC after the adverse decision of the CSC-NCR was rendered but raised the issue of lack of
jurisdiction over his person. He argued that as an employee in the Judiciary, the jurisdiction
to hear disciplinary action against him vests with the Sandiganbayan or the Supreme Court.
It cannot therefore be said that he was estopped from assailing the jurisdiction of the CSC.

This notwithstanding, we reiterate that we will not and cannot tolerate dishonesty for the
judiciary expects the highest standard of integrity from all its employees. The conduct and
behavior of everyone connected with an office charged with the dispensation of justice is
circumscribed with a heavy burden or responsibility. The Court will not hesitate to rid its
ranks of undesirables.

Garcia v. Miro
Facts: Rodolfo Garcia, then Presiding Judge of the MCTC of Negros Occidental was charged
with the crime of murder and the administrative offenses of grave misconduct and abuse of
authority. The Office of the Ombudsman conducted an investigation of the case for
prosecutors cannot do such since they regularly appear before the sala of the petitioner.
Issue:
Ruling:
Indeed, supervision over all inferior courts and court personnel, from the Presiding Justice
of the Court of Appeals to the lowest ranked court employee, is vested by the Constitution
in the Supreme Court. However, that prerogative only extends to administrative
supervision. As such, the Ombudsman cannot encroach upon this Courts task to oversee
judges and court personnel and take the proper administrative action against them if they
commit any violation of the laws of the land.

However, in the case at bar, the criminal case filed against petitioner was in no way related
to the performance of his duties as a judge. Contrary to petitioners allegation, the
administrative aspect of the case against him was endorsed by the Ombudsman-Visayas to
the OCA for appropriate action. In addition, an administrative complaint against petitioner
involving the same facts was filed by Julieta Ortega with the OCA. Petitioner cannot feign
ignorance of this fact considering that he filed a Comment and Answer to the Complaint-
Affidavit of Mrs. Julieta Ortega.

To reiterate, the case filed against petitioner before the MCTC is a criminal case under its
own jurisdiction as prescribed by law and not an administrative case. To be sure, trial
courts retain jurisdiction over the criminal aspect of offenses committed by judges of the
lower courts.
Section 9. The Members of the Supreme Court and judges of the lower courts shall be
appointed by the President from a list of at least three nominees prepared by the
Judicial and Bar Council for every vacancy. Such appointments need no confirmation.

For the lower courts, the President shall issue the appointments within ninety days
from the submission of the list.
Chavez v. JBC
Facts:
The case is in relation to the process of selecting the nominees for the vacant seat of
Supreme Court Chief Justice following Renato Corona’s departure.
Originally, the members of the Constitutional Commission saw the need to create a
separate, competent and independent body to recommend nominees to the President.
Thus, it conceived of a body representative of all the stakeholders in the judicial
appointment process and called it the Judicial and Bar Council (JBC).
In particular, Paragraph 1 Section 8, Article VIII of the Constitution states that “(1) A
Judicial and Bar Council is hereby created under the supervision of the Supreme Court
composed of the Chief Justice as ex officio Chairman, the Secretary of Justice, and a
representative of the Congress as ex officio Members, a representative of the Integrated
Bar, a professor of law, a retired Member of the Supreme Court, and a representative of the
private sector.” In compliance therewith, Congress, from the moment of the creation of the
JBC, designated one representative from the Congress to sit in the JBC to act as one of the ex
officio members.
In 1994 however, the composition of the JBC was substantially altered. Instead of having
only seven (7) members, an eighth (8th) member was added to the JBC as two (2)
representatives from Congress began sitting in the JBC – one from the House of
Representatives and one from the Senate, with each having one-half (1/2) of a vote. During
the existence of the case, Senator Francis Joseph G. Escudero and Congressman Niel C.
Tupas, Jr. (respondents) simultaneously sat in JBC as representatives of the legislature.
It is this practice that petitioner has questioned in this petition.
The respondents claimed that when the JBC was established, the framers originally
envisioned a unicameral legislative body, thereby allocating “a representative of the
National Assembly” to the JBC. The phrase, however, was not modified to aptly jive with the
change to bicameralism which was adopted by the Constitutional Commission on July 21,
1986. The respondents also contend that if the Commissioners were made aware of the
consequence of having a bicameral legislature instead of a unicameral one, they would have
made the corresponding adjustment in the representation of Congress in the JBC; that if
only one house of Congress gets to be a member of JBC would deprive the other house of
representation, defeating the principle of balance.
The respondents further argue that the allowance of two (2) representatives of Congress to
be members of the JBC does not render JBC’s purpose of providing balance nugatory; that
the presence of two (2) members from Congress will most likely provide balance as against
the other six (6) members who are undeniably presidential appointees
Supreme Court held that it has the power of review the case herein as it is an object of
concern, not just for a nominee to a judicial post, but for all the citizens who have the right
to seek judicial intervention for rectification of legal blunders.
Issue:
Whether the practice of the JBC to perform its functions with eight (8) members, two (2) of
whom are members of Congress, defeats the letter and spirit of the 1987 Constitution.
Ruling:
No. The current practice of JBC in admitting two members of the Congress to perform the
functions of the JBC is violative of the 1987 Constitution. As such, it is unconstitutional.
One of the primary and basic rules in statutory construction is that where the words of a
statute are clear, plain, and free from ambiguity, it must be given its literal meaning and
applied without attempted interpretation. It is a well-settled principle of constitutional
construction that the language employed in the Constitution must be given their ordinary
meaning except where technical terms are employed. As such, it can be clearly and
unambiguously discerned from Paragraph 1, Section 8, Article VIII of the 1987 Constitution
that in the phrase, “a representative of Congress,” the use of the singular letter “a”
preceding “representative of Congress” is unequivocal and leaves no room for any other
construction. It is indicative of what the members of the Constitutional Commission had in
mind, that is, Congress may designate only one (1) representative to the JBC. Had it been
the intention that more than one (1) representative from the legislature would sit in the
JBC, the Framers could have, in no uncertain terms, so provided.
Moreover, under the maxim noscitur a sociis, where a particular word or phrase is
ambiguous in itself or is equally susceptible of various meanings, its correct construction
may be made clear and specific by considering the company of words in which it is founded
or with which it is associated. Every meaning to be given to each word or phrase must be
ascertained from the context of the body of the statute since a word or phrase in a statute is
always used in association with other words or phrases and its meaning may be modified
or restricted by the latter. Applying the foregoing principle to this case, it becomes
apparent that the word “Congress” used in Article VIII, Section 8(1) of the Constitution is
used in its generic sense. No particular allusion whatsoever is made on whether the Senate
or the House of Representatives is being referred to, but that, in either case, only a singular
representative may be allowed to sit in the JBC
Considering that the language of the subject constitutional provision is plain and
unambiguous, there is no need to resort extrinsic aids such as records of the Constitutional
Commission. Nevertheless, even if the Court should proceed to look into the minds of the
members of the Constitutional Commission, it is undeniable from the records thereof that it
was intended that the JBC be composed of seven (7) members only. The underlying reason
leads the Court to conclude that a single vote may not be divided into half (1/2), between
two representatives of Congress, or among any of the sitting members of the JBC for that
matter.
With the respondents’ contention that each representative should be admitted from the
Congress and House of Representatives, the Supreme Court, after the perusal of the records
of Constitutional Commission, held that “Congress,” in the context of JBC representation,
should be considered as one body. While it is true that there are still differences between
the two houses and that an inter-play between the two houses is necessary in the
realization of the legislative powers conferred to them by the Constitution, the same cannot
be applied in the case of JBC representation because no liaison between the two houses
exists in the workings of the JBC. No mechanism is required between the Senate and the
House of Representatives in the screening and nomination of judicial officers. Hence, the
term “Congress” must be taken to mean the entire legislative department.
The framers of Constitution, in creating JBC, hoped that the private sector and the three
branches of government would have an active role and equal voice in the selection of the
members of the Judiciary. Therefore, to allow the Legislature to have more quantitative
influence in the JBC by having more than one voice speak, whether with one full vote or
one-half (1/2) a vote each, would “negate the principle of equality among the three
branches of government which is enshrined in the Constitution.”
It is clear, therefore, that the Constitution mandates that the JBC be composed of seven (7)
members only. Thus, any inclusion of another member, whether with one whole vote or
half (1/2) of it, goes against that mandate. Section 8(1), Article VIII of the Constitution,
providing Congress with an equal voice with other members of the JBC in recommending
appointees to the Judiciary is explicit. Any circumvention of the constitutional mandate
should not be countenanced for the Constitution is the supreme law of the land. The
Constitution is the basic and paramount law to which all other laws must conform and to
which all persons, including the highest officials of the land, must defer. Constitutional
doctrines must remain steadfast no matter what may be the tides of time. It cannot be
simply made to sway and accommodate the call of situations and much more tailor itself to
the whims and caprices of the government and the people who run it.
Notwithstanding its finding of unconstitutionality in the current composition of the JBC, all
its prior official actions are nonetheless valid. In the interest of fair play under the doctrine
of operative facts, actions previous to the declaration of unconstitutionality are legally
recognized. They are not nullified.

In Re: Villanueva (can’t search the full text)

De Castro v. JBC
Facts:
This is a consolidated case which assails the constitutionality of the action of former
President Gloria Macapagal Arroyo by appointing a Chief Justice 7 days after the
Presidential election in 2010.

After the compulsory retirement of former Chief Justice Reynato Puno, the position of Chief
Justice was left vacant. Section 4 (1), in relation to Section 9, Article VIII of the Constitution
states that, "vacancy shall be filled within ninety days from occurrence thereof," from a, "List
of nominees prepared by the Judicial Bar Council for every vacancy" furthermore, Section 15,
Article VII was also taken into consideration which prohibits the President or the Acting
President from making appointments within two (2) months immediately before the next
Presidential elections and up to the end of his term, except temporary appointments to
executive positions when continued vacancies therein will prejudice public service or
endanger public safety.
The JBC agreed that the vacant position must be filled and there were five (5) candidates
for the position from the most senior of the Associates of the court and one of them is
Associate Justice Reynato C. Corona who was chosen by the President and was appointed
for the position of Chief Justice.

Office of the Solicitor General (OSG) contends that the incumbent President may appoint
the next Chief Justice since the Constitution do not apply to the Supreme Court. If the
framers of the Constitution intended the prohibition to apply in the Supreme Court then it
should have expressly stated it in the Constitution.

Issue: W/Nthe President can appoint the successor of the Chief Justice..

Ruling:
Yes, the President can appoint the successor of Chief Justice as the prohibitions in the
Constitution.
Prohibition under Section 15, Article VII does not apply to appointments to fill a vacancy in
the Supreme Court or to other appointments to the Judiciary.

Two constitutional provisions are seemingly in conflict.

The first, Section 15, Article VII (Executive Department), provides: Section 15. Two months
immediately before the next presidential elections and up to the end of his term, a
President or Acting President shall not make appointments, except temporary
appointments to executive positions when continued vacancies therein will prejudice
public service or endanger public safety. The other, Section 4 (1), Article VIII (Judicial
Department), states: Section 4. (1). The Supreme Court shall be composed of a Chief Justice
and fourteen Associate Justices. It may sit en banc or in its discretion, in division of three,
five, or seven Members. Any vacancy shall be filled within ninety days from the occurrence
thereof.

Had the framers intended to extend the prohibition contained in Section 15, Article VII to
the appointment of Members of the Supreme Court, they could have explicitly done so.
They could not have ignored the meticulous ordering of the provisions. They would have
easily and surely written the prohibition made explicit in Section 15, Article VII as being
equally applicable to the appointment of Members of the Supreme Court in Article VIII
itself, most likely in Section 4 (1), Article VIII. That such specification was not done only
reveals that the prohibition against the President or Acting President making appointments
within two months before the next presidential elections and up to the end of the
President’s or Acting President’s term does not refer to the Members of the Supreme Court.

Had the framers intended to extend the prohibition contained in Section 15, Article VII to
the appointment of Members of the Supreme Court, they could have explicitly done so.
They could not have ignored the meticulous ordering of the provisions. They would have
easily and surely written the prohibition made explicit in Section 15, Article VII as being
equally applicable to the appointment of Members of the Supreme Court in Article VIII
itself, most likely in Section 4 (1), Article VIII. That such specification was not done only
reveals that the prohibition against the President or Acting President making appointments
within two months before the next presidential elections and up to the end of the
President’s or Acting President’s term does not refer to the Members of the Supreme Court.

Section 14, Section 15, and Section 16 are obviously of the same character, in that they
affect the power of the President to appoint. The fact that Section 14 and Section 16 refer
only to appointments within the Executive Department renders conclusive that Section 15
also applies only to the Executive Department. This conclusion is consistent with the rule
that every part of the statute must be interpreted with reference to the context, i.e. that
every part must be considered together with the other parts, and kept subservient to the
general intent of the whole enactment. It is absurd to assume that the framers deliberately
situated Section 15 between Section 14 and Section 16, if they intended Section 15 to cover
all kinds of presidential appointments. If that was their intention in respect of
appointments to the Judiciary, the framers, if only to be clear, would have easily and surely
inserted a similar prohibition in Article VIII, most likely within Section 4 (1) thereof.

Sec. 10 Salary

Nitafan v. Commissioner
Facts:
Petitioners, the duly appointed and qualified Judges presiding over Branches 52, 19 and 53,
respectively, of the Regional Trial Court, NCR, seek to prohibit and/or perpetually enjoin
the Commissioner of Internal Revenue and the Financial Officer of the Supreme Court, from
making any deduction of withholding taxes from their salaries. They submit that "any tax
withheld from their emoluments or compensation as judicial officers constitutes a decrease
or diminution of their salaries, contrary to the provision of Section 10, Article VIII of the
1987 Constitution mandating that "during their continuance in office, their salary shall not
be decreased," even as it is anathema to the Ideal of an independent judiciary envisioned in
and by said Constitution."

Ruling:
The debates, interpellations and opinions expressed regarding the constitutional provision
in question until it was finally approved by the Commission disclosed that the true intent of
the framers of the 1987 Constitution, in adopting it, was to make the salaries of members of
the Judiciary taxable.

it is plain that the Constitution authorizes Congress to pass a law fixing another rate of
compensation of Justices and Judges but such rate must be higher than that which they are
receiving at the time of enactment, or if lower, it would be applicable only to those
appointed after its approval. It would be a strained construction to read into the provision
an exemption from taxation in the light of the discussion in the Constitutional Commission.

Section 11. The Members of the


Supreme Court and judges of De la Llana v. Alba
lower courts shall hold office
during good behavior until they
reach the age of seventy years or
become incapacitated to discharge Facts:
the duties of their office. The
Supreme Court en banc shall have In 1981, Batas Pambansa Blg. 129, entitled “An Act
the power to discipline judges of Reorganizing the Judiciary, Appropriating Funds Therefor and
lower courts, or order their
dismissal by a vote of a majority of
the Members who actually took
part in the deliberations on the
issues in the case and voted
thereon.
for Other Purposes”, was passed. Gualberto De la Llana, a judge in Olongapo, was assailing
its validity because, first of all, he would be one of the judges that would be removed
because of the reorganization and second, he said such law would contravene the
constitutional provision which provides the security of tenure of judges of the courts. He
averred that only the Supreme Court can remove judges NOT the Congress.

Issue: W/N a judge like Judge De La Llana can be validly removed by the legislature by
such statute (BP 129).
Ruling:
Yes. The SC ruled the following way: “Moreover, this Court is empowered “to discipline judges
of inferior courts and, by a vote of at least eight members, order their dismissal.” Thus it
possesses the competence to remove judges. Under the Judiciary Act, it was the President who
was vested with such power.
• Removal is, of course, to be distinguished from termination by virtue of the
abolition of the office. There can be no tenure to a non-existent office. After the
abolition, there is in law no occupant. In case of removal, there is an office with
an occupant who would thereby lose his position. It is in that sense that from
the standpoint of strict law, the question of any impairment of security of
tenure does not arise. Nonetheless, for the incumbents of inferior courts abolished,
the effect is one of separation. As to its effect, no distinction exists between removal
and the abolition of the office. Realistically, it is devoid of significance. He ceases to be
a member of the judiciary. In the implementation of the assailed legislation,
therefore, it would be in accordance with accepted principles of constitutional
construction that as far as incumbent justices and judges are concerned, this Court be
consulted and that its view be accorded the fullest consideration. No fear need be
entertained that there is a failure to accord respect to the basic principle that this
Court does not render advisory opinions.
• No question of law is involved. If such were the case, certainly this Court could not
have its say prior to the action taken by either of the two departments. Even then, it
could do so but only by way of deciding a case where the matter has been put in issue.
Neither is there any intrusion into who shall be appointed to the vacant positions
created by the reorganization. That remains in the hands of the Executive to whom it
properly belongs.
• There is no departure therefore from the tried and tested ways of judicial power.
Rather what is sought to be achieved by this liberal interpretation is to preclude any
plausibility to the charge that in the exercise of the conceded power of reorganizing
the inferior courts, the power of removal of the present incumbents vested in this
Tribunal is ignored or disregarded. The challenged Act would thus be free from any
unconstitutional taint, even one not readily discernible except to those predisposed to
view it with distrust. Moreover, such a construction would be in accordance with the
basic principle that in the choice of alternatives between one which would save and
another which would invalidate a statute, the former is to be preferred.”
People v. Gacott

Facts: For failure to check the citations of the prosecution, the order of respondent RTC
Judge Eustaquio Gacott, Jr. dismissing a criminal case was annulled by the SC. The
respondent judge was also sanctioned with a reprimand and a fine of P10,000.00 for gross
ignorance of the law. The judgment was made by the Second Division of the SC.

Issue: Whether or not the Second Division of the SC has the competence to
administratively discipline respondent judge
Ruling:
To support the Court’s ruling, Justice Regalado relied on his recollection of a conversation
with former Chief Justice Roberto Concepcion who was the Chairman of the Committee on
the Judiciary of the 1986 Constitutional Commission of which Regalado was also a member.

The very text of the present Sec. 11, Art. VIII of the Constitution clearly shows that there are
actually two situations envisaged therein. The first clause which states that “the SC en banc
shall have the power to discipline judges of lower courts,” is a declaration of the grant of
that disciplinary power to, and the determination of the procedure in the exercise thereof
by, the Court en banc. It was not therein intended that all administrativedisciplinary cases
should be heard and decided by the whole Court since it would result in an absurdity.

The second clause, which refers to the second situation contemplated therein and is
intentionally separated from the first by a comma, declares on the other hand that the
Court en banc can “order their dismissal by a vote of a majority of the Members who
actually took part in the deliberations on the issues in the case and voted therein.” In this
instance, theadministrative case must be deliberated upon and decided by the full Court
itself.

Pursuant to the first clause which confers administrative disciplinary power to the Court en
banc, a decision en banc is needed only where the penalty to be imposed is the dismissal of
a judge, officer or employee of the Judiciary, disbarment of a lawyer, or either
the suspension of any of them for a period of more than 1 year or a fine exceeding P10,
000.00 or both.

Indeed, to require the entire Court to deliberate upon and participate in


alladministrative matters or cases regardless of the sanctions, imposable or imposed,
would result in a congested docket and undue delay in theadjudication of cases in the
Court, especially in administrative matters, since even cases involving the penalty of
reprimand would require action by the Court en banc.

Section 13. The conclusions of the Supreme Court in any case submitted to it for
decision en banc or in division shall be reached in consultation before the case is
assigned to a Member for the writing of the opinion of the Court. A certification to
this effect signed by the Chief Justice shall be issued and a copy thereof attached to
the record of the case and served upon the parties. Any Member who took no part, or
dissented, or abstained from a decision or resolution, must state the reason therefor.
The same requirements shall be observed by all lower collegiate courts.

Consing v. CA

Facts:
Private respondent Caridad Santos and the Consings entered into an agreement
denominated as a "Contract of Sale" whereby the latter agreed to sell, transfer and convey
to the former a house and lot. However, Santos defaulted in her payments. Consing sent her
several letters of demand to which she did not reply which prompted the counsel for the
Consings to send a final demand letter to Santos asking her to settle her obligations,
otherwise, they shall be constrained to resort to court litigation.
Santos, represented by a lawyer, manifested her willingness to settle her obligations on the
condition that the Consings comply with all the laws and regulations on subdivisions and
after payment to her of damages as a consequence of the use of a portion of her lot, more or
less 168 sq.m., as a subdivision road.
Subsequently, the Consings filed an ejectment case against Santos. After trial, on November
4, 1974, judgment was rendered by Judge Gregorio de la Paz of the Municipal Court of
Marikina in favor of the Consings.

However, with the ejectment case still pending, Santos filed with the then Court of First
Instance a complaint for specific performance with damages against the Consings. CFI
rendered judgment finding that although the Consings may have "corrected the
irregularities and/or [have] complied with the legal requirements for the operation of their
subdivision, they cannot escape their liability to Santos for having sold to her portions of
the roads or streets denominated as right-of-way. On this ground alone, this Court believes
that Santos was fully justified in refusing to pay further her monthly amortizations. In the
interest of justice, fair play and equity, this Court believes that there shall be a
proportionate reduction of the purchase price of the two lots corresponding to the area of
168 square meters, more or less, used as a right of way.

The Consings interposed an appeal to the Court of Appeals which affirmed the decision of
the CFI

Issue: W/N the decision rendered by the respondent Court of Appeals in this case does not
comply with the requirements of Article VIII, section 13, of the 1987 Constitution
Ruling:
The certification requirement, is a new provision introduced by the framers of the 1987
Constitution. Its purpose is to ensure the implementation of the constitutional requirement
that decisions of the Supreme Court and lower collegiate courts, such as the Court of
Appeals, Sandiganbayan and Court of Tax Appeals, are reached after consultation with the
members of the court sitting en banc or in a division before the case is assigned to a
member thereof for decision-writing. The decision is thus rendered by the court as a body
and not merely by a member thereof. This is in keeping with the very nature of a collegial
body which arrives at its decisions only after deliberation, the exchange of views and ideas,
and the concurrence of the required majority vote.
The absence, however, of the certification would not necessarily mean that the case
submitted for decision had not been reached in consultation before being assigned to one
member for the writing of the opinion of the Court since the regular performance of official
duty is presumed [Sec. 5 (m) of Rule 131, Rules of Court]. The lack of certification at the
end of the decision would only serve as evidence of failure to observe the
certification requirement and may be basis for holding the official responsible for
the omission to account therefor]. Such absence of certification would not have the
effect of invalidating the decision.
Estrada v. Desierto
Facts:
After Estrada’s impeachment proceedings were aborted and his resignation from the
Presidential post, a cluster of legal problems started appearing. Several cases previously
filed against him in the Office of the Ombudsman were set in motion including among
others, bribery and graft and corruption, plunder, perjury, serious miscounduct,
malversation of public funds, illegal use of public funds. A special panel of investigators was
forthwith created by the respondent Ombudsman to investigate the charges against the
petitioner.Petitioner filed with this Court a petition for prohibition with a prayer for a writ
of preliminary injunction. It sought to enjoin the respondent Ombudsman from
“conducting any further proceedings in any other criminal complaint that may be filed in
his office, until after the term of petitioner as President is over and only if legally warranted
Petitioner also contends that the respondent Ombudsman should be stopped from
conducting the investigation of the cases filed against him due to the barrage of prejudicial
publicity on his guilt. He submits that the respondent Ombudsman has developed bias and
is all set to file the criminal cases in violation of his right to due process.
Issue: W/N the prosecution of petitioner Estrada should be enjoined due to prejudicial
publicity
Ruling:

No. Then and now, we now rule that the right of an accused to a fair trial is not incompatible
to a free press. To be sure, responsible reporting enhances an accused’s right to a fair trial
for, as well pointed out, a responsible press has always been regarded as the handmaiden
of effective judicial administration, especially in the criminal field x x x. The press does not
simply publish information about trials but guards against the miscarriage of justice by
subjecting the police, prosecutors, and judicial processes to extensive public scrutiny and
criticism.

Pervasive publicity is not per se prejudicial to the right of an accused to fair trial. The mere
fact that the trial of appellant was given a day-to-day, gavel-to-gavel coverage does not by
itself prove that the publicity so permeated the mind of the trial judge and impaired his
impartiality. Our judges are learned in the law and trained to disregard off-court evidence
and on-camera performances of parties to a litigation. Their mere exposure to publications
and publicity stunts does not per se fatally infect their impartiality.

At best, appellant can only conjure possibility of prejudice on the part of the trial judge due
to the barrage of publicity that characterized the investigation and trial of the case. To
warrant a finding of prejudicial publicity, there must be allegation and proof that the judges
have been unduly influenced, not simply that they might be, by the barrage of publicity. In
the case at bar, the records do not show that the trial judge developed actual bias against
appellant as a consequence of the extensive media coverage of the pre-trial and trial of his
case. The totality of circumstances of the case does not prove that the trial judge acquired
a fixed opinion as a result of prejudicial publicity which is incapable of change even by
evidence presented during the trial. Appellant has the burden to prove this actual bias and
he has not discharged the burden. The court recognizes that pervasive and prejudicial
publicity under certain circumstances can deprive an accused of his due process right to
fair trial.However,petitioner needs to show more weighty social science evidence to
successfully prove the impaired capacity of a judge to render a bias-free decision. Thus the
petition was dismissed.

Chavez v. PEA
Facts:
In July 2002, Amari Coastal Bay Development Corporation lost a case (PEA-Amari Scandal)
before the Supreme Court involving certain reclaimed lands. Upon receipt of the adverse
decision, Amari filed a Motion for Inhibition asking the ponente of said case, Justice Antonio
Carpio, to inhibit from the case on the ground that before Justice Carpio was appointed to
the Supreme Court, he wrote a column in the Manila Times newspaper where he
questioned the legality of the agreement between the Public Estates Authority and Amari
regarding the said reclaimed property (PEA-Amari deal). Amari insists that Justice Carpio
already prejudged the issue as his bias and prejudice were already apparent. Amari also
prays for a re-deliberation after Justice Carpio inhibits.
Issue: W/N Justice Carpio should inhibit from the case by reason of the said Manila Times
column.
Ruling:
No. In the first place, the decision was already promulgated when Amari filed its motion
requesting Justice Carpio to inhibit. The rule is that a motion to inhibit must be denied if
filed after a member of the Court had already given an opinion on the merits of the case.
Reason: a litigant cannot be permitted to speculate upon the action of the Court (only to)
raise an objection of this sort after a decision has been rendered.
Second, judges and justices are not disqualified from participating in a case just because
they have written legal articles on the law involved in the case.
Third, looking at Justice Carpio’s Manila Times article, his article questioned the legality of
the PEA Amari deal on the basis of the lack of public bidding. In this particular case before
the Supreme Court, the issue of the absence of public bidding was not raised by any of the
parties involved hence, Justice Carpio’s write up had nothing to do with the very merits of
the case.
Pedragoza v. Comelec
Facts:
Petitioner Artemio Pedragoza and respondent Francisco Sumulong, Jr. were among the
candidates for Punong Barangay of De La Paz, Antipolo City in the 15 July 2002
Sangguniang Kabataan and Barangay elections. Petitioner was proclaimed winner by a
margin of 39 votes. Claiming that irregularities marred the elections, respondent filed an
election protest in the Municipal Trial Court in Cities, Antipolo City. Respondent sought a
recount of ballots from 25 out of De La Paz's 103 precincts. petitioner denied respondent's
claim and filed a counter-protest, contending that he was the one prejudiced by election
irregularities. The trial court revised the contested ballots. Respondent appealed to the
Comelec which then granted. Petitioner sought reconsideration with the COMELEC En
Banc, listing the ballots he wanted re-examined. However, in the per curiam Resolution, the
COMELEC En Banc denied petitioner's motion and affirmed the First Division's findings. All
the three incumbent signed the Resolution while Commissioners Sadain and Tuason took
no part, without, however, indicating the reasons for their inhibition.

Issue: W/N the failure of Commissioners Sadain and Tuason to indicate their reasons for
taking no part in the case annuls the Resolution and, if in the negative.

Ruling:
To begin with, even if the votes of Commissioners Sadain and Tuason are disregarded (for
whatever reason), a quorum still remains, with three of the then five8 COMELEC
Commissioners voting to deny petitioner's motion for reconsideration. The more important
question is whether, despite such quorum, the 30 September 2005 Resolution should be
invalidated for failure of the two Commissioners to state the reasons for their inhibition.
While there is no extant record of the COMELEC's proceedings in adopting Section 1, Rule
18 of the COMELEC Rules, the parallel deliberations of the framers of the 1987 Constitution
on Section 13, Article VIII shed light on the purpose of the rule requiring a member of this
Court and all lower collegiate courts to state his reason for taking no part in a case. Because
of the exact identity of the rule in question as stated in Section 1, Rule 18 and Section 13,
Article VIII, these deliberations apply here by analogy.
Being a devise to dissuade members of this Court and all lower collegiate courts (or in this
case, the members of the COMELEC) from not taking part in the deliberation of cases, the
requirement has nothing to do with the ruling involved but concerns the judge himself.
Thus, non-compliance with the rule does not annul the ruling in which a judge takes no part
but may be basis for holding him responsible for the omission.
Indeed, the omission involved here is akin to the failure of the head of a collegiate court to
issue the certification under Section 13, Article VIII that "The conclusions of the x x x Court
in any case submitted to it for decision en banc or in division [was] reached in consultation
before the case [was] assigned to a Member for the writing of the opinion of the Court," a
requirement also imposed on the Chairman or the Presiding Commissioner of the
COMELEC, as the case may be, under Section 1, Rule 18. We held in Consing v. Court of
Appeals that such omission does not invalidate the questioned ruling but "may be basis for
holding the official responsible for the omission to account.
Accordingly, we hold that the failure of Commissioners Sadain and Tuason to state the
reasons for their inhibition from the 30 September 2005 Resolution does not affect the
validity of that ruling.

Section 14. No decision shall be rendered by any court without expressing therein clearly and distinctly the facts and the
law on which it is based.
No petition for review or motion for reconsideration of a decision of the court shall be refused due course or denied
without stating the legal basis therefor.

a. Must state the facts and the law on which the decision is based

People v. Bugarin
Facts:
Accused-appellant in a rape case questions the trial court’s decision on the ground that the
decision of the trial court does not state the facts and law upon which it was based. In a
two-page decision, promulgated on February 11, 1993, the trial court, after giving a
summary of the testimonies of the complainant and accused-appellant, laconically ruled:
The issue is simple. Is the private complainant credible in her story of how she was raped? The
answer of this Court is an undoubtful and a definite yes.

Ruling:
The Constitution provides in part in Art. VIII, 14 that No decision shall be rendered by any
court without expressing therein clearly and distinctly the facts and the law on which it is
based. This requirement is reiterated and implemented by the 1985 Rules of Criminal
Procedure which provides in Rule 120, 2:
Sec. 2. Form and contents of judgment. - The judgment must be written in the official
language, personally and directly prepared by the judge and signed by him and shall
contain clearly and distinctly a statement of the facts proved or admitted by the accused
and the law upon which the judgment is based.
Conviction. If it is of conviction, the judgment shall state (a) the legal qualification of the
offense constituted by the acts committed by the accused, and the aggravating or mitigating
circumstances attending the commission thereof, if there are any; (b) the participation of
the accused in the commission of the offense, whether as principal, accomplice, or
accessory after the fact; (c) the penalty imposed upon the accused; and (d) the civil liability
or damages caused by the wrongful act to be recovered from the accused by the offended
party, if there is any, unless the enforcement of the civil liability by a separate action has
been reserved or waived.
Acquittal. In case of acquittal, unless there is a clear showing that the act from which the
civil liability might arise did not exist, the judgment shall make a finding on the civil liability
of the accused in favor of the offended party.
Application. The decision of the trial court falls short of this requirement in at least three
respects.
a) First, it does not contain an evaluation of the evidence of the parties and a
discussion of the legal questions involved. It does not explain why the trial court
considered the complainants testimony credible despite the fact that, as accused-
appellant points out, complainant could not remember the time of the day when
she was allegedly raped. It does not explain why accused-appellants licking of
complainants genital constituted attempted rape and not another crime.

b) Second, the complainant testified that she had been raped five times, to wit, in
November 1989, on December 24, 1989, in June 1990, on March 14, 1991, and on
December 23, 1991, and that once, on January 17, 1992, she was molested by her
father who licked her private part, for which reason six informations were filed
against him, but the decision found the accused-appellant guilty of only four counts
of rape (which the trial court erroneously said three counts) and one count of
attempted rape, without explaining whether accused-appellant was being acquitted
of one charge of rape.

c) Third, the decision is so carelessly prepared that it finds the accused-appellant


guilty of three counts of consummated rape but sentences him to suffer the penalty
of reclusion perpetua for each of the four counts of rape.

The victim claimed she had been raped on December 24, 1989, but the information in
Criminal Case No. Q-92-31160 is for rape allegedly committed in May 1990. It must be for
this reason that the trial court convicted accused-appellant of only four counts of rape,
instead of five. But the trial court should have explained so, if this was really the reason,
and expressly acquitted the accused-appellant of the charge under this information.
The requirement that the decisions of courts must be in writing and that they must set
forth clearly and distinctly the facts and the law on which they are based serves many
functions. It is intended, among other things, to inform the parties of the reason or reasons
for the decision so that if any of them appeals, he can point out to the appellate court the
findings of facts or the rulings on points of law with which he disagrees. More than that, the
requirement is an assurance to the parties that, in reaching judgment, the judge did so
through the processes of legal reasoning. It is, thus, a safeguard against the impetuosity of
the judge, preventing him from deciding by ipse dixit. Vouchsafed neither the sword nor the
purse by the Constitution but nonetheless vested with the sovereign prerogative of passing
judgment on the life, liberty or property of his fellowmen, the judge must ultimately
depend on the power of reason for sustained public confidence in the justness of his
decision. The decision of the trial court in this case disrespects the judicial function.
Hernandez v. CA (can’t search the full text)

Saballa v. NLRC
Facts:
Herein petitioners were among the 52 employees who were subjects to a forced leave of
office which purportedly part of the cost-saving measures instituted to enable the
Camarines Sur III Electric Cooperative Inc., to meet its financial obligations especially with
NPC and NEA. Instead of reinstatement, Camarines Sur III Electric Cooperative Inc., applied
to the Regional Office for the retrenchment of the employees on forced leave and directed
the cooperative department heads, area supervisors, division and sections chiefs not to
accept any of the 52 employees who might try to report back to work.

Affected employees filed illegal dismissal cases against the private respondent. The Labor
Arbiter ruled in favor of the complainants, saying that the Labor Code does not authorize
any employer to declare any employee under forced leave, temporary or otherwise.

However, Private respondent appealed the arbiters decision to the NLRC, which modified
the arbiters ruling and held that complainants separation is valid, due to retrenchment.
Petitioners argue that while the NLRC claimed to disagree with the factual
findings/conclusions of the arbiter, it did not state what particular findings and conclusions
it could not go along with; and while the Decision purports to apply the requisites for a
valid retrenchment, the public respondent did not specify what those were.

Ruling:
This Court has previously held that judges and arbiters should draw up their decisions
and resolutions with due care, and make certain that they truly and accurately
reflect their conclusions and their final dispositions. A decision should faithfully
comply with Section 14, Article VIII of the Constitution which provides that no decision
shall be rendered by any court without expressing therein clearly and distinctly the facts of
the case and the law on which it is based.
If such decision had to be completely overturned or set aside, upon the filing of a motion
for reconsideration, in a subsequent action via a resolution or modified decision, such
resolution or decision should likewise state the factual and legal foundation relied upon.
The reason for this is obvious:
a) aside from being required by the Constitution, the court should be able to justify
such a sudden change of course;
b) it must be able to convincingly explain the taking back of its solemn conclusions
and pronouncements in the earlier decision.
The same thing goes for the findings of fact made by the NLRC, as it is a settled rule that
such findings are entitled to great respect and even finality when supported by substantial
evidence; otherwise, they shall be struck down for being whimsical and capricious and
arrived at with grave abuse of discretion. It is a requirement of due process and fair
play that the parties to a litigation be informed of how it was decided, with an
explanation of the factual and legal reasons that led to the conclusions of the court. A
decision that does not clearly and distinctly state the facts and the law on which it is based
leaves the parties in the dark as to how it was reached and is especially prejudicial to the
losing party, who is unable to pinpoint the possible errors of the court for review by a
higher tribunal.
Based on the foregoing considerations, we find the assailed Decision arbitrary in its naked
assertion that: Applying the requisites (for valid retrenchment) to the instant case, we lent
credence to respondents evidence supporting the fact that it was suffering financial
reverses. Hence, complainants separation is valid, due to retrenchment.
The Decision does not indicate the specific bases for such crucial holding. While it
enumerated some of the factors that supposedly weighed in favor of private respondents
position, public respondent nevertheless did not bother to explain how it came to the
conclusion that private respondent was experiencing business reversals, nor did it specify
which particular data and document it based such conclusion upon. This can only be
because the private respondent failed to show convincingly by substantial evidence the fact
of its failing financial health, and that such retrenchment was justified
b. Memorandum Decisions

Francisco v. Permskul
Facts:
Victorino Francisco leased his apartment to Winai Permskul who deposited the amount of
P9,000.00 in accordance to the lease contract. However, after a short stay, Permskul
vacated the property and thereby requested Francisco for a refund of his deposit minus the
sum of P1,000.00. The petitioner rejected this request and said that the lessee still owed
him for other charges, including the electricity and water bills and the sum of P2,500.00 for
repainting of the leased premises to restore them to their original condition.
Francisco sued Permskul in the Metropolitan Trial Court of Makati which ruled in favour of
Permskul.
The decision was appealed to the Regional Trial Court of Makati and was affirmed by Judge
Jose C. de la Rama on January 14, 1987. This was done in a memorandum decision reading
in full as follows:
MEMORANDUM DECISION
After a careful and thorough perusal, evaluation and study of the records of
this case, this Court hereby adopts by reference the findings of fact and
conclusions of law contained in the decision of the Metropolitan Trial Court of
Makati, Metro Manila, Branch 63 and finds that there is no cogent reason to
disturb the same.
WHEREFORE, judgment appealed from is hereby affirmed in toto.
When the defendant went to the Court of Appeals, his petition for review was denied, as
well as his motion for reconsideration. He is now before us to fault the respondent court,
principally for sustaining the memorandum decision of the regional trial court. His
contention is that it violates Article VIII, Section 14 of the Constitution.

Issue: W/N the memorandum decision authorized under Section 40 of B.P. Blg. 129 in the
light of Article VIII, Section 14 of the Constitution

Ruling:
** It cannot be too strongly emphasized that just as important as the intrinsic validity of a
decision is the perception by the parties-litigants that they have been accorded a fair
opportunity to be heard by a fair and responsible magistrate before judgment is rendered.
It is this perception, coupled with a clear conscience, which enables the members of the
judiciary to discharge the awesome responsibility of sitting in judgment on their
fellowmen.
The PURPOSE of the law in authorizing the memorandum decision is to expedite the
termination of litigations for the benefit of the parties as well as the courts themselves.
The law does not define the memorandum decision and simply suggests that the court may
adopt by reference the findings of fact and the conclusions of law stated in the decision,
order or resolution on appeal before it. No particular form is prescribed; the conditions for
its use are not indicated. In fact, B.P. Blg. 129 does not even employ the term
"memorandum decision" in Section 40 or elsewhere in the rest of the statute. This phrase
appears to have been introduced in this jurisdiction not by that law but by Section 24 of the
Interim Rules and Guidelines, reading as follows:
Sec. 24. Memorandum decisions. — -The judgment or final resolution of a
court in appealed cases may adopt by reference the findings of fact and
conclusions of law contained in the decision or final order appealed from.
The DISTINCTIVE FEATURES of the memorandum decision are,
a) first, it is rendered by an appellate court; and
b) second, it incorporates by reference the findings of fact or the conclusions of law
contained in the decision, order or ruling under review.
Most likely, the purpose is to affirm the decision, although it is not impossible that the
approval of the findings of fact by the lower court may lead to a different conclusion of law
by the higher court. At any rate, the reason for allowing the incorporation by reference is
evidently to avoid the cumbersome reproduction of the decision of the lower court, or
portions thereof, in the decision of the higher court. The Idea is to avoid having to repeat in
the body of the latter decision the findings or conclusions of the lower court since they are
being approved or adopted anyway.
What is questioned about the Section 40 of B.P. Blg. 129 is the permission it gives for the
appellate court to merely adopt by reference in its own decision the judgment of the lower
court on appeal. It is easy to understand that this device may feed the suspicion feared by
Justice Feria that the court has not given the appeal the attention it deserved and thus
deprived the parties of due process. True or not, this impression is likely to undermine
popular faith in the judiciary as an impartial forum which hears before it decides and bases
its decision on the established facts and the applicable law.
Requirement for Validity. The memorandum decision, to be valid, cannot incorporate the
findings of fact and the conclusions of law of the lower court only by remote reference,
which is to say that the challenged decision is not easily and immediately available to the
person reading the memorandum decision. For the incorporation by reference to be
allowed, it must provide for direct access to the facts and the law being adopted, which
must be contained in a statement attached to the said decision. In other words, the
memorandum decision authorized under Section 40 of B.P. Blg. 129 should actually
embody the findings of fact and conclusions of law of the lower court in an annex
attached to and made an indispensable part of the decision.
The Court finds it necessary to emphasize that the memorandum decision should be
sparingly used lest it become an addictive excuse for judicial sloth. It is an additional
condition for its validity that this kind of decision may be resorted to only in cases where
the facts are in the main accepted by both parties or easily determinable by the judge and
there are no doctrinal complications involved that will require an extended discussion of
the laws involved. The memorandum decision may be employed in simple litigations
only, such as ordinary collection cases, where the appeal is obviously groundless and
deserves no more than the time needed to dismiss it.
Despite the convenience afforded by the memorandum decision, it is still desirable
that the appellate judge exert some effort in restating in his own words the findings
of fact of the lower court and presenting his own interpretation of the law instead of
merely parroting the language of the court a quo as if he cannot do any better. There
must be less intellectual indolence and more pride of authorship in the writing of a
decision, especially if it comes from an appellate court.
Oil and Natural Gas v. CA
Facts: Oil And Natural Gas Commission, a foreign corporation owned and controlled by the
Government of India, filed before the RTC of Surigao City a complaint for the enforcement
of Indian Court’s decision against its contract partner, Pacific Cement Company
Incorporated, a private corporation duly organized and existing under the laws of the
Philippines. The complaint was filed when private respondent refuse to comply with the
order and several demands of the petitioner.

Ruling:
The pertinent portion of the judgment of the foreign court reads:: Award dated 23.7.88,
Paper No. 3/B-1 is made Rule of the Court. On the basis of conditions of award decree is
passed. Award Paper No. 3/B-1 shall be a part of the decree. The plaintiff shall also be
entitled to get from defendant ( US$ 899, 603.77 (US$ Eight Lakhs ninety nine thousand six
hundred and three point seventy seven only) alongwith 9% interest per annum till the last
date of realisation.

SC: As specified in the order of the Civil Judge of Dehra Dun, Award Paper No. 3/B-1 shall
be a part of the decree. This is a categorical declaration that the foreign court adopted the
findings of facts and law of the arbitrator as contained in the latters Award Paper. Award
Paper No. 3/B-1, contains an exhaustive discussion of the respective claims and defenses of
the parties, and the arbitrators evaluation of the same. Inasmuch as the foregoing is
deemed to have been incorporated into the foreign courts judgment the appellate court
was in error when it described the latter to be a simplistic decision containing literally, only
the dispositive portion.
The constitutional mandate that no decision shall be rendered by any court without
expressing therein clearly and distinctly the facts and the law on which it is based does not
preclude the validity of memorandum decisions which adopt by reference the findings of
fact and conclusions of law contained in the decisions of inferior tribunals. In Francisco v.
Permskul, this Court held that the following memorandum decision of the Regional Trial
Court of Makati did not transgress the requirements of Section 14, Article VIII of the
Constitution:
MEMORANDUM DECISION
After a careful perusal, evaluation and study of the records of this case, this Court hereby
adopts by reference the findings of fact and conclusions of law contained in the decision of
the Metropolitan Trial Court of Makati, Metro Manila, Branch 63 and finds that there is no
cogent reason to disturb the same.
WHEREFORE, judgment appealed from is hereby affirmed in toto. (Underscoring supplied.)
This Court had occasion to make a similar pronouncement in the earlier case of Romero v.
Court of Appeals, where the assailed decision of the Court of Appeals adopted the findings
and disposition of the Court of Agrarian Relations in this wise:
We have, therefore, carefully reviewed the evidence and made a re-assessment of the same,
and We are persuaded, nay compelled, to affirm the correctness of the trial courts factual
findings and the soundness of its conclusion. For judicial convenience and expediency,
therefore, We hereby adopt by way of reference, the findings of facts and conclusions of the
court a quo spread in its decision, as integral part of this Our decision.

c. Where rule is not applicable


Dadubo v. CSC
Facts:
Petitioner Lolita A. Dadubo, Senior Accounts Analyst of DBP, who was designated once as
an acting teller, found guilty by the DBP of dishonesty for embezzlement of bank funds.
Petitioner appealed to the CSC which reversed the order of DBP. However, DBP filed a
motion for reconsideration which the commission granted.

Issue: W/N CSC fails to comply with the constitutional requirement to state clearly and
distinctly the facts of the law in its decision.

Ruling:

The petitioner's challenges are mainly factual. The rule is that the findings of fact of
administrative bodies, if based on substantial evidence, are controlling on the reviewing
authority is settled that it is not for the appellate court to substitute its own judgment for
that of the administrative agency on the sufficiency of the evidence and the credibility of
the witnesses. Administrative decisions on matters within their jurisdiction are entitled to
respect and can only be set aside on proof of grave abuse of discretion, fraud or error of
law. None of these vices has been shown in this case.

We must also dismiss the petitioner's complaint that CSC Resolution No. 92-878 failed to
comply with the constitutional requirement to state clearly and distinctly the facts and the
law on which a decision is based. We have held that this provision applies only to courts of
justice and not to administrative bodies like the Civil Service Commission.

In any event, there was an earlier statement of the facts and the law involved in the
decision rendered by the MSPB dated February 28, 1990, which affirmed DBP's decision to
dismiss the petitioner. In both decisions, the facts and the law on which they were based
were clearly and distinctly stated.

Padua v. Ranada
Facts:
Eduardo Zialcita, as a taxpayer and as Congressman of Paranaque City alleged that the TRB
violated the Constitution when it did not express clearly and distinctly the facts and the law
on which Resolution No. 2001-89 (authorizing provisional toll rate adjustments at the
Metro Manila Skyway) was based.

Issue: W/N Resolution No. 2001-89 issued by the Toll Regulatory Board (TRB) is valid.

Ruling:
Obviously, the laws and the TRB Rules of Procedure have provided the remedies of an
interested Expressways user. The initial proper recourse is to file a petition for review of
the adjusted toll rates with the TRB. The need for a prior resort to this body is with reason.
The TRB, as the agency assigned to supervise the collection of toll fees and the operation of
toll facilities, has the necessary expertise, training and skills to judiciously decide matters
of this kind. As may be gleaned from the petition, the main thrust of petitioner Zialcitas
argument is that the provisional toll rate adjustments are exorbitant, oppressive, onerous
and unconscionable. This is obviously a question of fact requiring knowledge of the formula
used and the factors considered in determining the assailed rates. Definitely, this task is
within the province of the TRB.
We take cognizance of the wealth of jurisprudence on the doctrine of primary
administrative jurisdiction and exhaustion of administrative remedies. In this era of
clogged court dockets, the need for specialized administrative boards or commissions with
the special knowledge, experience and capability to hear and determine promptly disputes
on technical matters or intricate questions of facts, subject to judicial review in case of
grave abuse of discretion, is indispensable. Between the power lodged in an
administrative body and a court, the unmistakable trend is to refer it to the former.
In Industrial Enterprises, Inc. vs. Court of Appeals, we ruled:

x x x, if the case is such that its determination requires the expertise, specialized skills and
knowledge of the proper administrative bodies because technical matters or intricate
questions of facts are involved, then relief must first be obtained in an administrative
proceeding before a remedy will be supplied by the courts even though the matter is within
the proper jurisdiction of a court.

Moreover, petitioner Zialcitas resort to prohibition is intrinsically inappropriate. It bears


stressing that the office of this remedy is not to correct errors of judgment but to prevent
or restrain usurpation of jurisdiction or authority by inferior tribunals and to compel them
to observe the limitation of their jurisdictions. G.R. No. 151108, while designated as a
petition for prohibition, has for its object the setting aside of Resolution No. 2001-89 on the
ground that it was issued without prior notice, hearing and publication and that the
provisional toll rate adjustments are exorbitant. This is not the proper subject of
prohibition because as long as the inferior court, tribunal or board has jurisdiction
over the person and subject matter of the controversy, the writ will not lie to correct
errors and irregularities in procedure, or to prevent an erroneous decision or an
enforcement of an erroneous judgment. And even in cases of encroachment, usurpation,
and improper assumption of jurisdiction, the writ will not issue where an adequate and
applicable remedy by appeal, writ or error, certiorari, or other prescribed methods of
review are available. In this case, petitioner Zialcita should have sought a review of the
assailed Resolution before the TRB.
d. Denial of Due Course

German v. Endaya

Facts:
Eddie Endaya was an employee of German Machineries Corporation who was unduly
dismissed from service in the same
Company. Labor Arbiter rendered judgment in favor of herein respondent which then
affirmed by the NLRC. Agrieved by the judgment, petitioner filed a petition for a temporary
restraining order and/or preliminary injunction with the Court of Appeals assailing
decision and resolution of the NLRC. However, Appellate Court issued a resolution
dismissing the petition.

Issue: W/N CA violated the constitutional provision that no decision shall be rendered by
any court without expressing clearly and distinctly the facts and the law on which it is
based.

Ruling:
The assailed resolution is not the decision contemplated under Section 14, Article VIII of
the Constitution. The mandate embodied in this constitutional provision is applicable only
in cases submitted for decision i.e., given due course and after the filing of briefs or
memoranda and/or other pleadings, but not where a resolution is issued denying due
course to a petition and stating the legal basis thereof. Thus, when the court, after
deliberating on a petition and subsequent pleadings, decides to deny due course to
the petition and states that the questions raised are factual or there is no reversible
error in the respondent courts decision, there is sufficient compliance with the
constitutional requirement.

In the present case, the Court of Appeals denied due course and outrightly dismissed the
petition for certiorari filed by herein petitioner on the grounds that the factual issues had
already been passed upon by the NLRC, and since its factual findings are in agreement with
the findings of the labor arbiter, the same are binding and conclusive upon the Court of
Appeals; and that the questions raised are too unsubstantial to require consideration. We
find these legal bases in conformity with the requirements of the Constitution.

Protacio v. Laya

Facts:
Zayber John Protacio resigned from his position as Tax Principal in KPMG Laya
Mananghaya & Co. Upon resignation, he demanded the immediate payment of his 13th
month pay, the cash commutation of his leave credits and the issuance of his 1999
Certificate of Income Tax Withheld on Compensation. However, Respondent firm failed to
act upon the demand letters, thereby Protacio filed a complaint before the NLRC which
thereafter granted the same.

Respondents elevated the matter to the Court of Appeals via a petition for certiorari, which
reduced the total money award to petitioner. Petitioner sought reconsideration but was
denied through a resolution.

Issue: w/n CA’s summary denial of petitioner’s motion for reconsideration violates the
constitutional requirement that court decisions must state the legal and factual basis

Ruling:
Obviously, the assailed resolution is not a "decision" within the meaning of the
Constitutional requirement. This mandate is applicable only in cases "submitted for
decision," i.e., given due course and after filing of briefs or memoranda and/or other
pleadings, as the case may be. The requirement is not applicable to a resolution denying a
motion for reconsideration of the decision. What is applicable is the second paragraph of
the above-quoted Constitutional provision referring to "motion for reconsideration of a
decision of the court." The assailed resolution complied with the requirement therein
that a resolution denying a motion for reconsideration should state the legal basis of
the denial. It sufficiently explained that after reading the pleadings filed by the
parties, the appellate court did not find any cogent reason to reverse itself.
Next, petitioner argues that the Court of Appeals erred in giving due course to the petition
for certiorari when the resolution thereof hinged on mere evaluation of evidence.
Petitioner opines that respondents failed to make its case in showing that the Labor Arbiter
and the NLRC had exercised their discretion in an arbitrary and despotic manner.

As a general rule, in certiorari proceedings under Rule 65 of the Rules of Court, the
appellate court does not assess and weigh the sufficiency of evidence upon which the Labor
Arbiter and the NLRC based their conclusion. The query in this proceeding is limited to the
determination of whether or not the NLRC acted without or in excess of its jurisdiction or
with grave abuse of discretion in rendering its decision. However, as an exception, the
appellate court may examine and measure the factual findings of the NLRC if the same are
not supported by substantial evidence. The Court has not hesitated to affirm the appellate
court’s reversals of the decisions of labor tribunals if they are not supported by substantial
evidence

Section 15. (1) All cases or matters


filed after the effectivity of this Re: Problems of Delays in cases before the
Constitution must be decided or Sandiganbayan.
resolved within twenty-four
months from date of submission Facts:
for the Supreme Court, and, unless
Integrated Bar of the Philippines through a resolution of its
reduced by the Supreme Court,
twelve months for all lower Board of Governors, recommending an inquiry into the causes
collegiate courts, and three months of delays in the resolution of incidents and motions and in the
for all other lower courts. decision of cases pending before the Sandiganbayan.
(2) A case or matter shall be
deemed submitted for decision or
Issue:
resolutionWhat is filing
upon the the ofreglementary
the period within which the Sandiganbayan must
decide/resolve
last pleading, cases
brief,falling
or within its jurisdiction?
memorandum required by the
Rules of Court or by the court
Ruling:
itself.
(3) Upon the expiration of the
Period To Decide/Resolve
corresponding period, a
Cases. There are two views.
certification to this effect signed by
☝The first
the Chief view
Justice is that
or the from the time a case is submitted for decision or resolution, the
presiding
Sandiganbayan has
judge shall forthwith twelve
be issued and(12) months to decide or resolve it.
a copy thereof attached to the
☝The second view is that as a court with trial function, the Sandiganbayan has three (3)
record of the case or matter, and
served upon the parties. The
months to decide
certification the case
shall state why from
a the date of submission for decision.
decision or resolution has not been
rendered or issued within said
period.
Article VIII, Section 15 (1) and (2), of the 1987 Constitution provides:
(4) Despite the expiration of the
"Sec. mandatory
applicable 15. (1) All casestheor matters filed after the effectivity of this Constitution must be
period,
court, without prejudice to such
decided or resolved
responsibility as may have been within twenty-four months from date of submission to the Supreme
Court,
incurred and, unlessthereof,
in consequence reduced by the Supreme Court, twelve months for all lower collegiate
shall courts,
decide orand three
resolve the months
case or for all other lower courts.
matter submitted thereto for
determination, without further
delay.
(2) A case or matter shall be deemed submitted for decision or resolution upon the filing
of the last pleading, brief or memorandum required by the Rules of Court or by the court
itself.

The above provision does not apply to the Sandiganbayan. The provision refers to regular
courts of lower collegiate level that in the present hierarchy applies only to the Court of
Appeals. The Sandiganbayan is a special court of the same level as the Court of Appeals and
possessing all the inherent powers of a court of justice, with functions of a trial court.
Thus, the Sandiganbayan is not a regular court but a special one. The Sandiganbayan
was originally empowered to promulgate its own rules of procedure. However, on March
30, 1995, Congress repealed the Sandiganbayans power to promulgate its own rules of
procedure and instead prescribed that the Rules of Court promulgated by the Supreme
Court shall apply to all cases and proceedings filed with the Sandiganbayan.

Special courts are judicial tribunals exercising limited jurisdiction over particular or
specialized categories of actions. They are the Court of Tax Appeals, the Sandiganbayan,
and the Sharia Courts. Under Article VIII, Section 5 (5) of the Constitution Rules of
procedure of special courts and quasi-judicial bodies shall remain effective unless
disapproved by the Supreme Court.

In his report, the Court Administrator would distinguish between cases which the
Sandiganbayan has cognizance of in its original jurisdiction, and cases which fall within the
appellate jurisdiction of the Sandiganbayan. The Court Administrator posits that since in
the first class of cases, the Sandiganbayan acts more as a trial court, then for that
classification of cases, the three (3) month reglementary period applies.

The law creating the Sandiganbayan, P.D. No. 1606 is clear on this issue. It provides:
Sec. 6. Maximum period for termination of cases As far as practicable, the trial of cases
before the Sandiganbayan once commenced shall be continuous until terminated and the
judgment shall be rendered within three (3) months from the date the case was submitted
for decision.

On September 18, 1984, the Sandiganbayan promulgated its own rules, thus:
Sec. 3 Maximum Period to Decide Cases The judgment or final order of a division of the
Sandiganbayan shall be rendered within three (3) months from the date the case was
submitted for decision.
Given the clarity of the rule that does not distinguish, we hold that the three (3) month
period, not the twelve (12) month period, to decide cases applies to the Sandiganbayan.
Furthermore, the Sandiganbayan presently sitting in five (5) divisions, functions as a trial
court. The term trial is used in its broad sense, meaning, it allows introduction of evidence
by the parties in the cases before it. The Sandiganbayan, in original cases within its
jurisdiction, conducts trials, has the discretion to weigh the evidence of the parties, admit
the evidence it regards as credible and reject that which they consider perjurious or
fabricated.
Licaros v. Sandiganbayan
Facts:
Abelardo Licaros was involved in Legaspi City Branch of the Central Bank robbery case as
an accessory which was then filed at Sandiganbayan. Licaros filed several motion for the
disposition of the case and praying that judgment be rendered acquitting him of the offense
charged. However, Sandiganbayan deffered the same.

Issue: W/N the unexplained failure of the SANDIGANBAYAN to render the Decision for
more than ten (10) years violated herein petitioners constitutional right to due process and
to a speedy disposition of the case.

Ruling:

The unreasonable delay of more than ten (10) years to resolve a criminal case, without fault
on the part of the accused and despite his earnest effort to have his case decided, violates the
constitutional right to the speedy disposition of a case. Unlike the right to a speedy trial, this
constitutional privilege applies not only during the trial stage, but also when the case has
already been submitted for decision.
Under Section 6 of PD 1606 amending PD 1486, the Sandiganbayan has only 90 days to
decide a case from the time it is deemed submitted for decision. Considering that the
subject criminal case was submitted for decision as early as June 20, 1990, it is obvious that
respondent court has failed to decide the case within the period prescribed by law. Even if
we were to consider the period provided under Section 15(1), Article III of the 1987
Constitution, which is 12 months from the submission of the case for decision, the
Sandiganbayan would still have miserably failed to perform its mandated duty to render a
decision on the case within the period prescribed by law. Clearly then, the decision in this
case is long overdue, and the period to decide the case under the law has long expired.
Even more important than the above periods within which the decision should have been
rendered is the right against an unreasonable delay in the disposition of ones case before
any judicial, quasi-judicial or administrative body. This constitutionally guaranteed right
finds greater significance in a criminal case before a court of justice, where any delay in
disposition may result in a denial of justice for the accused altogether. Indeed, the
aphorism justice delayed is justice denied is by no means a trivial or meaningless concept

In Abadia v. Court of Appeals, the Court had the occasion to rule on the nature and the extent as well as the broader protection afforded by
the constitutional right to the speedy disposition of a case, as compared with the right to a speedy trial. Thus, it ratiocinated as follows:
The Bill of Rights provisions of the 1987 Constitution were precisely crafted to expand substantive fair trial rights and to protect citizens
from procedural machinations which tend to nullify those rights. Moreover, Section 16, Article III of the Constitution extends the right to a
speedy disposition of cases to cases before all judicial, quasi-judicial and administrative bodies. This protection extends to all citizens, x x x
and covers the periods before, during and after the trial, affording broader protection than Section 14(2) which guarantees merely the right
to a speedy trial.
It has been held that a breach of the right of the accused to the speedy disposition of a case may have consequential effects, but it is not
enough that there be some procrastination in the proceedings. In order to justify the dismissal of a criminal case, it must be established
that the proceedings have unquestionably been marred by vexatious, capricious and oppressive delays.
In the case before us, the failure of the Sandiganbayan to decide the case even after the lapse of more than ten years after it was
submitted for decision involves more than just a mere procrastination in the proceedings. From the explanation given by the
Sandiganbayan, it appears that the case was kept in idle slumber, allegedly due to reorganizations in the divisions and the lack of logistics
and facilities for case records. Had it not been for the filing of this Petition for Mandamus, petitioner would not have seen any development
in his case, much less the eventual disposition thereof. The case remains unresolved up to now, with only respondent courts assurance that
at this time work is being done on the case for the preparation and finalization of the decision

that can be taken for granted by those who are tasked with the dispensation of justice.
In Dela Pea v. Sandiganbayan, penned by Chief Justice Hilario G. Davide Jr., the Court laid down certain guidelines to
determine whether the right to a speedy disposition has been violated, as follows:
The concept of speedy disposition is relative or flexible. A mere mathematical reckoning of the time involved is not
sufficient. Particular regard must be taken of the facts and circumstances peculiar to each case. Hence, the doctrinal rule is
that in the determination of whether that right has been violated, the factors that may be considered and balanced are as
follows: (1) the length of the delay; (2) the reasons for the delay; (3) the assertion or failure to assert such right by the
accused; and (4) the prejudice caused by the delay.

More than ten years has lapsed since the subject case has been deemed submitted for
decision. The delay cannot at all be attributed to petitioner, who has neither utilized
dilatory tactics nor undertaken any procedural device to prolong the proceedings. As a
matter of fact, he has been continuously pushing for the resolution of his case even during
the early stages of the prosecution. Moreover, it is undeniable that such delay has caused
much prejudice, distress and anxiety to herein petitioner, whose career as bank executive
and businessman has suffered the stigma of being shackled to an unresolved criminal
prosecution, virtually hanging like a Damocles sword over his head for more than a decade.
We need not stress the consequences and problems inherent in this pending litigation
and/or criminal prosecution which include the prospects of unrealized business
transactions, stagnant professional growth, hampered travel opportunities and a
besmirched reputation. Furthermore, it is worth noting that petitioner has been charged
merely as an accessory after the fact due to his being a senior executive of the bank where
the principal accused tried to deposit the stolen money. Clearly then, the dismissal sought
by herein petitioner is justified under the circumstances and in accordance with the
guidelines set forth in the above-cited case.

Re: Report on the Judicial Audit and Physical Inventory of cases conducted in the
Regional Trial Courts of Davao City

Facts:
Audit team discovered that in Branches 8 to 17 RTC, Davao City, several cases were left
undecided/resolved within the 90-day reglementary period, thereby prompted the
Supreme Court to release a resolution stating: 1) to note the compliance of nine (9) out of
the ten (10) Regional Trial Court Judges, and 2) to require Judge William Layague, RTC,
Branch 14, Davao City, to explain why no administrative sanction should be imposed on
him for his continued failure to comply with the November 24, 1994 resolution and to
submit his compliance thereto within five days from notice.
On the backlog of cases, Judge Layague explained that when he assumed office in Branch 14
in 1987 he inherited so many cases submitted for decision and there were more than sixty
(60) cases not reflected in the docket which was discovered only after he conducted an
inventory. Moreover, the Team also observed that Judge Layague pens his decisions as if
each one of them is a masterpiece which causes further delay.
However, with regard to Judge Layagues compliance, the Supreme Court is not fully
satisfied with the explanation of Judge Layague.
Ruling:
The situation should not be allowed to continue indefinitely.

In Diputado-Baguio vs. Torres, 211 SCRA 1, the judge was fined P20,000.00 for his failure to
decide three (3) criminal cases within 90 days, even though the court found four (4)
mitigating circumstances in his favor.
In the case at bar, while we find such factors as congestion of docket, lack of necessary
personnel and the judges illness to be fairly adequate to mitigate the latters liability, the
cases unresolved beyond the 90-day period number more than 100. A fine of Twenty-Five
Thousand (P25,000.00) Pesos would thus be reasonable.

Goodlucksa Exam! Please use at your own risk. –KC ☺

You might also like